January 6 edit

Life or death determination edit

 
Death following loss of blood.

When is a potato dead? Edison 02:13, 6 January 2007 (UTC)

It is definitely dead when it is cooked, completely rotten or dried up. Death as a concept is not easily applied to plants as most plants can regenerate from a few cells. Animals can be cloned but it is not the same as regeneration and so cloned animals are not considered a continuation of the life but a new life. --- Skapur 02:21, 6 January 2007 (UTC)
Remarkable. The Wikipedia Google Queen 02:29, 6 January 2007 (UTC)
I would say a plant is only dead when all of it's cells are dead. Also note that each potato is only a small part of the root system of the plant, which may very well still be alive (depending on the farming method). StuRat 02:33, 6 January 2007 (UTC)
I agree that a plant is only dead when the cells die. Even when plucked the cells in a plant are alive, although, in the case of leaves they will begin to enter the senescence program that leads to death. For a potato it is alive until it rots since it is sustaining meristems in the eyes that can form new shoots and roots. It rots, due to bacterial infection i.e. Erwinia carotovora. In fact, the potatoes are not attached to the root system but a stolon. David D. (Talk) 19:16, 6 January 2007 (UTC)
So then that's like asking when an apple is dead. Once you "pick" it, so to speak. Ƶ§œš¹ [aɪm ˈfɻɛ̃ⁿdˡi] 05:27, 6 January 2007 (UTC)
Hmmm, I'm not so sure. You say if you pluck a potato from its plant, the rest of the plant may still be alive, but the potato is dead. But if that potato happens to have an "eye" on it, and you slice off that "eye" and put it in water, it will begin to grow. So it must still be alive. Or not? — Michael J 17:32, 6 January 2007 (UTC)
If a cut rose is placed in water and a root system develops and the plant is perpetuated as a new rose bush, then was the cut flower dead? If cells from a dead animal are cloned and the clone lives, then was the animal still alive? Edison 19:44, 6 January 2007 (UTC)
This is an interesting point. You are now bringing in the distinction between brain dead and physiologically dead. This does not apply to plants but is clearly relevant to animals. Check out the following links Clinical_death and Brain_death. David D. (Talk) 19:47, 6 January 2007 (UTC)
By a dead animal I mean one which has experienced Clinical_death and Brain_death and has perhaps been run over, sliced and diced. Edison 21:22, 6 January 2007 (UTC)
I think you now entering the realm of science fiction. But you may be interested to read about Craig Venter's new company. http://www.syntheticgenomics.com/about.htm David D. (Talk) 21:45, 6 January 2007 (UTC)

I would define death as "the point at which the original can no longer be reproduced, naturally, by what remains". A potato, and many plants species, therefore remain alive as long as it can grow a reasonable facsimile of the original. This isn't true for most small parts of animals, however, which can't regrow a copy of the original. There are exceptions, however, such as the planarian. Cloning doesn't count, since it isn't natural. Also, in the case of animals, it doesn't restore the memories in the brain. StuRat 15:59, 8 January 2007 (UTC)

My cat is neutered, so it "can no longer be reproduced, naturally, by what remains." But it seems pretty alive. This reproductive definition does not seem very satisfying. Did you include cloning as "natural?" Edison 21:41, 10 January 2007 (UTC)

Struggling Butterflies edit

A friend on MySpace posted this story on my bulletin:

"A man found a cocoon for a butterfly. One day a small opening appeared, he sat and watched the butterfly for several hours as it struggled to force its body through the little hole. Then it seemed to stop making any progress. It appeared as if it had gotten as far as it could and could go no farther. Then the man decided to help the butterfly.

He took a pair of scissors and snipped the remaining bit of the cocoon. The butterfly then emerged easily. Something was strange. The butterfly had a swollen body and shriveled wings. The man continued to watch the butterfly because he expected at any moment, the wings would enlarge and expand to be able to support the body, which would contract in time. Neither happened. In fact, the butterfly spent the rest of its life crawling around with a swollen body and deformed wings. It was never able to fly.

What the man in his kindness and haste did not understand, was that the restricting cocoon and the struggle required for the butterfly to get through the small opening of the cocoon are God`s way of forcing fluid from the body of the butterfly into its wings so that it would be ready for flight once it achieved its freedom from the cocoon. Sometimes struggles are exactly what we need in our life.

If God allowed us to go through all our life without any obstacles, that would cripple us. We would not be as strong as what we could have been. Not only that, we could never fly."

Is this true? I ask for the sake of spiritual enrichment, because as a student of "A Course in Miracles" I tend to believe that suffering is unnecessary for spiritual development, but because this idea is so implanted in the collective conciousness, we "make" it. It probably doesn't matter, because the natural world does not reflect the spiritual in many cases, but I am curious. Thank you for your time.

24.88.54.247 03:40, 6 January 2007 (UTC) Sarah Ray

Hi, Sarah. First, you should know that the WP Reference Desks are not the ideal location for a spiritual/philosophical discussion, but welcome. Second, you said the word 'suffering', but the story you told used the word 'struggle'. Who says that something that is hard is also a source of suffering? I am familiar with The A Course in Miracles, and (sorry to be giving unsolicited advice) I think that as long as you are conflating challenge with suffering you will be stalled in your spiritual development. Anchoress 03:48, 6 January 2007 (UTC)
If we had no obstacles in life, we would lack the skills necessary to solve problems and get ourselves out of emergency situations. However, if there were no problems or emergency situations to begin with, we won't need those skills. So if there is no suffering in the world, the world would be better, not worse.
In your case of the butterfly, is it not possible for God to distribute the fluid while the puppa is still in the cocoon, so that it doesn't need to struggle to come out? --Bowlhover 04:28, 6 January 2007 (UTC)
Possible. But God made them animals, not puppets. BenC7 07:57, 6 January 2007 (UTC)
But God can change the life cycle of the butterfly to eliminate the need for struggle. He doesn't have to personally help every single butterfly. --Bowlhover 10:34, 7 January 2007 (UTC)
Consider reading Letter to a Christian Nation by Sam Harris. It's short and highly illuminating. This whole issue is actually moot. Religion cannot be constructively applied to science, hence the confusion. Richard Dawkins has much to say on this subject as well, though most religious people consider him infamous. Good luck! Jeeves 08:40, 6 January 2007 (UTC)

Personally I agree with the OP. If there were no struggles everything would be easy and noone would acchieve anything. And if everything was done for us, we would never learn from our mistakes. Also if I was like the butterfly, I wouldn't want any gods to help me and make everything better, as then I wouldn't be me any more. Unless it was a very serious problem, of course.Hidden secret 7 21:03, 6 January 2007 (UTC)

But that's not what the original poster is saying. She is saying that in her belief system suffering is not a factor. Anchoress 05:28, 7 January 2007 (UTC)

Butterfly technical question edit

Reading this has sparked my interest. Don't suppose there's anyone around here knowledgable about butterflies and would like to tell me whether the bit about the butterfly in the story is true? As in the bit about the fluid going from the body of the butterful to the wings? because it all sounds....a bit suspicious to me. --`/aksha 10:08, 6 January 2007 (UTC)

I've split this off into a separate section. I too am curious about the science – if any – behind this, and I'm hoping that the entomologists in our readership will come out of the woodwork to comment. TenOfAllTrades(talk) 22:33, 6 January 2007 (UTC)

Well the article on butterflies says this: "A newly-emerged butterfly needs to spend some time inflating its wings with blood and letting them dry." From the sound of it, most of the inflation occurs after it leaves the cocoon. In fact, if the cocoon is really that hard to get out of, it would make sense that the wings should be kept as small and uninflated as possible. Strad 00:18, 7 January 2007 (UTC)
Note this is unreferenced WP:OR, but the story interested me:-): I have just been over to our local butterfly breeder (not an trained entomologist, but 20 years breeding experience) to ask. Her comments: She thought the story was a joke, a fanciful fable thought up by some one who has never seen a butterfly emerge from a pupa. A normal butterfly does not struggle to get out of the chrysalis (only moths have silk cocoons). The splitting and emerging from the pupa takes "shorter than it takes you to skin a banana" - quick, simple, easy, no struggle. The pupa bursts it is not "gradually worked open". An animal that cannot expand enough to split the pupa skin is sick. An animal that cannot move sufficiently to exit from a split pupa is sick. Infection apparently does this, the animal gets stuck and dies halfway in the pupa, simply because it is too weak to move normally. She doubts if a healthy normal butterfly can get stuck in a pupa, the covering bursts for a good distance from the bottom almost to the top, and there is no "struggle to get out", rather a clumsiness in executing the first "baby steps" out of a patently open shell, to get going. She has "helped" only one specimen when she happened to be there to see it struggling (tweezers), but would not do it again, since she believes a struggle is a sign that there is something wrong with the butterfly. The "swollen body and shriveled wings" is normal; they always look like that on just emerging; that fluid (probably hemolymph, not blood) helped split the pupa, and will be pumped into the wings to expand them, and then returned to the butterfly's body when the wings have dried. As far as cocoons go (moths), she does not breed moths, but she makes the pertinent point that silkworm moths emerge normally from cocoonless pupae, so cutting the cocoon should make no difference. I would love to see a trained entomologist's angle on this. --Seejyb 05:21, 7 January 2007 (UTC)
In Annie Dillard's A Pilgram at Tinker Creak she describes a moth that emerges from metamorphasis inside of a glass jar. however the glass jar is not large enough to strecth its wings and then become stuck deformed by the shape of the jar. This would suggest that a wing "hardens" as upon emerging so its possible that the butterflies wings could have "hardened" in cocoon. Beckboyanch 06:25, 9 January 2007 (UTC)
I have read that butterfly collectors sometimes let the little creatures emerege from thei cocoon and pump up their winds to full glory, then they kill the butterflys and mount them in specimen cases. Edison 21:43, 10 January 2007 (UTC)

Love and P A I N edit

I'm in love with someone I probably can't get. I'm very curious as to why this psychological state causes actual, physical pain. Specifically, a burning feeling in my chest when I think about that special someone. I guess it's pretty mild compared to, say, cutting a finger open, but still. Has anyone figured out why this happens? Pain means nerves must be being stimulated, yet my body is wholly intact. I imagine this is why love historically came to be associated with the "heart", though the blood-pumping musculature is obviously not directly involved. What gives? Thanks. 24.95.48.112 07:12, 6 January 2007 (UTC)

Thats gives me an IDEA (not IKEA). Love Sickness can be Cured, lol. Sorry for my jokes but this is the best question I have ever seen on the Reference Board. Oh yeah and I have the same symptoms too, the anti-depressants: Celexa, Wellbutrin don't seem to be working for me. --Judged 08:35, 6 January 2007 (UTC)
It resembles the same pain as for anxiety. --Zeizmic 14:42, 6 January 2007 (UTC)
In addition to non-cardiac chest pain associated with emotional stress there is also research on Pathophysiological processes underlying emotional triggering of acute cardiac events. --JWSchmidt 23:12, 6 January 2007 (UTC)
The question, I believe, is why is depression associated with the heart? --Judged 04:18, 7 January 2007 (UTC)
I would guess, neurotransmitters trigger hormonal stress that triggers raised blood pressure and adrenaline rushes and general tension, and increased heartrate. That could all trigger acute cardiac events. Though the above links will be much more informative and authoritative than this guess of mine, which should be seen as a way to explain things in simpler terms. Oh, broken heart might help as well. Also acute coronary syndrome and panic attacks. Carcharoth 06:10, 7 January 2007 (UTC)

:From the above analysis you all relate it to stress, which acts directly on the Heart? How can you compare that with sadness in which there is no stress? --Jones2 11:20, 7 January 2007 (UTC)

  • Let me break it down for you, the neurotransmitters affect the whole body not just the brain. For example lack of neurotransmitter Dopamine, causes movement disorders eg. Parkinsons disease. So these neurotransmitters affect the chest region some how, I am guessing this part though, someone want to back me up? --Foundby 17:14, 7 January 2007 (UTC)
Or maybe, its just something that cant be explained by Science, and I think that we are figuring that out right now. Its our feelings, and our thoughts. No, the Heart that pumps blood is not where these feelings come from. It was believed in ancient times that the heart was the brain, not your actual brain. People thought that everything was caused by the heart, and they believed that the brain was useless. This could lead to the saying "it comes from the heart". Your brain and thoughts, however, have a kind of "different" heart; your feelings and thoughts. Its just that that makes you care, and makes you responsive to this person. When you get down to it, however, it is your brain causing these feelings. It is your brain causing your pain, love, and thoughts. (Sorry, I'm not the best at Wiki Editing, if I made any mistakes in my formatting, feel free to fix them. )--- (No Username...) Jan 11, 2007 at 8:29AM —The preceding unsigned comment was added by 207.69.138.12 (talk) 16:28, 11 January 2007 (UTC).

It is related to hormones, which are carried around in blood, so therefore it is blood, not the heart or the brain that controls peoples emotions &c :)Hidden secret 7 20:23, 11 January 2007 (UTC)

Most of people is controlled by their blood, the brain is just there to process information:)Hidden secret 7 20:23, 11 January 2007 (UTC)

Scientifically speaking Alcohol edit

Scientifically speaking Alcohol makes a person sick? --Delma1 08:30, 6 January 2007 (UTC)

In excess, yes it can. Ethanol#Metabolism_and_toxicology --Joel 11:02, 6 January 2007 (UTC)
Anything in excess can, but that's where the definition of "in excess" is. Vitamin C won't though! X [Mac Davis] (DESK|How's my driving?) 03:42, 7 January 2007 (UTC)
In response to Vitamin C, that is not completely true. For example, there is a case report about a guy with acute renal failure taking 5,000 mg of Vitamin C per day to fight off an upper respiratory tract infection. It resulted in the guy going to the hospital and was found to have tubular necrosis with massive oxalate deposition. The report goes on to say, "Our patient’s course and our review of the literature again support the contention that vitamin C ingestion may cause significant morbidity and mortality. Vitamin C should therefore not be viewed as a benign, water-soluble drug but rather as a drug that is potentially toxic, not only for diseased kidneys, but also for normal ones given the proper circumstances." - Dozenist talk 15:42, 7 January 2007 (UTC)
Then why does alcohol make you vomit? --Delma1 04:03, 7 January 2007 (UTC)
Vomiting is a "reflex" that eliminates ethanol from your stomach so your liver does not have to metabolize it. -Michael 04:55, 7 January 2007 (UTC)

Well how is this reflex activated due to alcohol consumption? --Delma1 14:03, 7 January 2007 (UTC)

I can't say with any certainty, but I'd assume that the body reacts to the presence of the toxin in sufficient levels by activating the vomiting center of the medulla, either by the vagal system and enteric nervous system being stimulated by the presence of ethanol or via the action of ethanol on the brain's chemoreceptors (by the action of the ethanol metabolite acetaldehyde perhaps?) This is an educated conjecture, but I'm not an expert, so I can't say with 100% certainty. Wintermut3 22:46, 7 January 2007 (UTC)

It is my understanding that alcohol is a tissue irritant and by irritiating the stomach (when taken in excess) the nausea and vomiting reflex is engaged protectively by the body to remove the irritant.Canticle 10:15, 14 January 2007 (UTC)

Type of Science edit

What the name of the type of science or job that looks for ways to help people from natural resources? Such as the African clawed toad, it has healing abilities. Or using snakes venom to create anti-venom.

Natural medicine? But it is not typically classified as a science, since it includes a number of areas that are pseudo-scientific. Science is also based on repeatable experiments (well, mostly) - and I doubt any experiements on a toad would show that people are healed by it. Possibly some chemical that it produces may be synthesized and be used for beneficial purposes. But then that's a chemical, not the toad itself. BenC7 12:23, 6 January 2007 (UTC)

Maybe... Bioprospecting --Zeizmic 13:24, 6 January 2007 (UTC)

Medicinal chemistry and pharmacology. The majority of new drugs are still being derived from natural substances, even if the molecules are altered to improve the kinetics. alteripse 15:06, 6 January 2007 (UTC)

Anaerobic respiration edit

I understand that the anaerobic pathway of respiration produces harmful waste products such as lactic acid. This causes fatigue in humans. How, then, can an anaerobic organism survive having lots of these waste products inside them? 86.147.210.9 12:31, 6 January 2007 (UTC)

Anaerobic waste products are excreted by anaerobic organisms, just like any other waste product. As you can read in the article on anaerobic organisms, there are many anaerobic chemical pathways, but they all result in at least one waste product. For instance, yeast, which undergoes fermentation to produce the waste product of ethanol, excretes the alcohol to get it out of its system. In fact, if you put yeast in a bottle with some water and carbohydrates, it keeps producing alcohol until it dies because there's too much alcohol around. Then you have some nice grain whiskey, or whatever. --Bmk 17:48, 6 January 2007 (UTC)
Actually, on second thought, I dont' think the yeast population dies because of the concentration of alcohol - I think the yeast population dies because it runs out of sugars to ferment. Sorry! --Bmk 17:48, 6 January 2007 (UTC)
It depends on the initial concentration of sugar, and on the type of yeast. With a lower concentration of sugar, fermentation proceeds until the sugar runs out; with high sugar concentrations, the alcohol concentration will get high enough to kill the yeast. (In its most extreme form, this comes into play in the making of certain late harvest and icewines, where extremely concentrated grape juices are used to generate a very sweet final product.) To produce alcoholic beverages more potent than about 15% to 20% alcohol by volume, one needs to concentrate the beverage after fermentation using a physical process like distillation. TenOfAllTrades(talk) 15:18, 6 January 2007 (UTC)

What is sleep apnea ? edit

What is sleep apnea ?

Hi - we have a large article on that subject called sleep apnea (link provided). Please try using the search box before asking here - it will be much quicker. Also, the instructions at the top of this page should be useful - they will tell you how to add new questions. --Bmk 17:48, 6 January 2007 (UTC)

Herpes? edit

I am a 22 year old male, and the other day I noticed this purple thing on my thigh. It is was like a blister bigger than my thumb-nail, and I put a needle through it and a lot of blood came out, and then i squeezed and all this white stuff (the same stuff that comes out of pimples) came out, a lot of it too. Now Ive read about herpes before, and I saw pictures, but it looks different, it doesnt look the same at all. But i'm worried, is it just a pimple? Thanks

Please direct medical questions to your doctor. In the mean time, you may want to read boil. -- mglg(talk) 21:19, 6 January 2007 (UTC)
Pimples aren't purple! "The same stuff that comes out of pimples" would be sebum. X [Mac Davis] (DESK|How's my driving?) 03:41, 7 January 2007 (UTC)
Accurate but incomplete information. Pimples also contain pus, and sometimes blood plasma or lymphatic fluid. Anchoress 10:51, 14 January 2007 (UTC)

sugar edit

What would happen if someone was on a diet consisting almost entirely of sugar, and very sugary food.Hidden secret 7 21:06, 6 January 2007 (UTC)

Start first with Sugar. Then, here's an article. [1] Seems that you'll rot your teeth and get diabetes. I've known of many young girls that go 'vegetarian', but end up as sugar-aholics. --Zeizmic 22:45, 6 January 2007 (UTC)
  • You'd also get serious deficiencies in the foodstuffs you weren't eating like fats, vitamins, aminoacids, fluids, etc, etc. - Mgm|(talk) 23:38, 6 January 2007 (UTC)
  • My Answer was kind of long so I put it on the user's talk page. See the answer here. Sifaka talk 00:31, 7 January 2007 (UTC)
Now that's a good thorough answer. X [Mac Davis] (DESK|How's my driving?) 03:43, 7 January 2007 (UTC)
  • If they didn't eat any protein with their carbohydrates/sugar, they might get kwashiorkor. --Uthbrian (talk) 09:16, 7 January 2007 (UTC)

Immunizations edit

Does Mexico require that school children be immunized before entry to school? What immunizations are required? What percentage of school age children are immunized? Thank you, pb

Here's an article from 2002 that says "Mexico more effective than U.S. at immunizing children: Mexico's paternalistic approach has led to a 96% vaccination rate for children ages 1 to 4, compared with 79% of American 2-year-olds." You can see Mexico's immunization schedule here (it's a .pdf file). - Nunh-huh 22:10, 6 January 2007 (UTC)

Do cats and dogs understand tv? edit

Can they understand that the 2D image represents a 3D scene? I've heard that they often watch a front-loading washing machine, what about tv?

Dogs really can't see very well, but they can detect motion. [2] It looks like 3d is just a dream to them. --Zeizmic 23:55, 6 January 2007 (UTC)
FWIW, my cat seems very interested in ball sports on TV, such as soccer, but ignores all other programmes. Grutness...wha? 00:03, 7 January 2007 (UTC)
Another anecdotal data point, my dog will bark when it sees other dogs on TV (but never normally on the tv just because of motion). Other people describe this too, however I've not studied it closely enough to decide perhaps if perhaps the dogs bark as well and this is what are setting them off. But I'm pretty sure if you just showed a dog running on TV with no sound he does bark. (One Man and His Dog being an example) 217.43.184.59 00:30, 7 January 2007 (UTC)
Does the different persistence of vision (I assume it is different for dogs than that of humans) make it difficult for the dog to see the image on TV? -- WikiCheng | Talk 14:08, 8 January 2007 (UTC)
As others have said, dogs do not actually have very good eye sight (although they have big eyes and can even see behind them a bit), so I wouldn't have thought their persistence of vision is better than ours. TV is 25 frames per second, though interlaced at 50 half frames a second. Humans are perfectly comfortable with much lower refresh rates, so I can't imagine dogs have a problem. 217.43.184.59 20:59, 8 January 2007 (UTC)
According to A.F. Chalmers in "What is this thing called Science" page 6, "the results of experiments on members of Africans tribes whose culture does not include the custom of depicting 3d objects by 2d perspective drawings perspective drawings, nor staircases for that matter indicate that members of those tribes would not see figure 1 (of a staircase) as a staircase at all". Does anyone know if similar experiments have been done on how said tribes perceive e.g. photos and TV? If humans don't necessarily perceive photos or TV in the same way then I would say animals probably don't. N.B. For anyone interested in this he also recommends Hanson, N.R; 1958; Patterns of discovery. Nil Einne 15:00, 13 January 2007 (UTC)
I've semi owned 4 cats and none of them have ever reacted to TV altho I did read once way back in 2000 or so about someone developing a TV ads for pets (Cats and dogs I think). A pet food company. My cats do sometimes respond to recorded meows and also barks etc. I would make sure your pets aren't responding to sound not the images if you think they're responding to TV Nil Einne 15:00, 13 January 2007 (UTC)
Anecdotes: I've got three cats. I've got a Cat TV video; it's all squirrels and birdies and so on. One of the cats totally ignores it. One of the cats is captivated, and will try to crawl into the TV screen (regardless of whether the sound is on.) The third tries to pretend she doesn't care, but eventually gets sucked in. We've also got a dog, Stella. A few years ago we were watching a SF Giants game. A squirrel made its way into the outfield. Stella went just as nutso as when she encounters a live one -- yip, yip, buck, buck. We were startled; we had no idea she was watching at all. But then when we put the event on replay, she looked at us with something like "Oh, you think I'll fall for that again?" and henceforth ignored it. So, yeah, dogs and cats sometimes see TV as real. (My mother-in-law too, but that's a different story.) --jpgordon∇∆∇∆ 15:11, 13 January 2007 (UTC)

Every breath very probably has an air atom also breathed by Julius Caesar edit

I recall reading that this is true. If that's the case, does the breath I'm breathing now contain an air atom breathed by every human who's ever lived (allowing for the time it takes for air to travel across the world and so on), plus the dinosaurs? Just how many atoms are there in a lungfull of air? Most atoms must have been breathed over and over again by many different people. Does anyone have the calculations that give rise to this belief?

I don't have the calculations, however here is guidance for them: the number of molecules in a certain volume can be calculated by the ideal gas law. You can also, by the information (volume per breath, breath frequency) in the article breathing calculate the total number of molecules breathed during the duration of a human life. Then you need the number of molecules in the atmosphere. There is several ways to get this number - the obvious idea is to try to find the volume of the atmosphere, and use that together with the ideal gas law. However, this solution is non-trivial (the atmosphere has different pressure on different heights). I would use the information on pressure at sea level in the article earth's atmosphere and the information on surface area in the article earth to calculate an approximate mass of the atmosphere instead. Combined with a weighted average molecule mass (calculable from the data in earth's atmosphere), you'd get the total number of molecules in the atmosphere.
Possibly you'll get really obvious numbers from this, but preferably you'd use some statistical hypothesis testing to hopefully confirm the hypothesis. TERdON 00:02, 7 January 2007 (UTC)
Didn't somebody ask a question similar to this a while back? Splintercellguy 00:21, 7 January 2007 (UTC)
Yeah, they did. This very issue is discussed on the book Innumeracy by John Allen Paulos, along with the calculations demonstrating the high probability of it. It is, however, highly unlikely that atoms of the air would travel half the world in, say, a year. — Kieff 00:37, 7 January 2007 (UTC)

It's more impressive than that. From what I've heard (and yes, shame on me for not working it out), each breath we take contains, on average, two atoms of each breath that everyone has taken. (If this turns out to be very wrong, I reserve the right to remove my sig). Bunthorne 00:41, 7 January 2007 (UTC)

I don't see how that would be possible. Atoms don't move around the earth that quickly. Chickenflicker--- 07:18, 7 January 2007 (UTC)


Okay, let's have a go. For purposes of this rough estimate we can ignore other substances and pretend the atmosphere is all nitrogen with a molecular weight of 28 -- oxygen is only a bit heavier and other gases are in small quantities). According to air, the total mass of the atmosphere ignoring water vapor is 5.14e18 kg. One mole of air (really nitrogen) is 28 g = 0.028 kg, so there are 5.14e18/0.028 = 1.84e20 moles of air. Each mole contains Avogadro's number of molecules, or 6.02e23, so there are 1.11e44 molecules of air altogether -- rounding to 1e44 will be close enough.

Now, breath says adults breathe 500 to 700 ml at a time. Say 600 ml or 0.6 L. One mole of any gas occupies the molar volume of 22.4 liters at standard pressure, so one breath is 0.6/22.4 = .027 mole, which is 1.6e22 molecules -- again, rounding to 1e22 will be close enough. (That's molecules; the original poster asked how many atoms in a breath, which will be twice that.)

So we have the coincidence that the total number of molecules in one breath is juts about the same as the number of breaths in the atmosphere. So yes, on average each breath you take will contain one molecule -- two atoms -- from Julius Caesar's last breath or any particular previous breath. (Assuming that there has been time for it to mix, of course; obviously it doesn't apply to a breath just taken by someone in a different city.) One complication I have ignored is that atoms get removed from the atmosphere and others added into it over time; I don't think this affects a substantial fraction of the atmosphere over a mere 2,000 years, but I could be wrong.

--Anonymous, January 7, 100% 2007, 06:52 (UTC).

600ml sounds like a lot for just one breath. Or I could just be breathing less than everyone else. Is there any way to find out?Hidden secret 7 11:34, 7 January 2007 (UTC)

Or you could just have no concept of how large your tidal volume is. You could get an actual value by taking a pulmonary function test, though it's probably going to be a bit expensive if taken just to satisfy your curiousity. - Nunh-huh 17:25, 7 January 2007 (UTC)
A plastic bag and a juice pitcher with a volume scale on the side would be cheaper. --mglg(talk) 22:10, 7 January 2007 (UTC)

January 7 edit

haploid/diploid edit

If chromosomal duplication happens twice before tetrad formation in spermatogenesis, while the other steps proceed normally, would the resulting 4 sperm cells be considered diploid because they each have 2 copies of a gene or haploid because the 2 copies are of the same homologue? In other words, does the term "diploid" in humans refer only to the total number of chromosomes (twice the number of the haploid state) or the state of having a pair of homologous chromosomes each with differing alleles for the same genes? thanks, N.S.

Binding energy edit

I'd like to make my own version of this graph. Where can I get a raw set of values for it? I tried googling a bit but all I found were fairly tedious formulae for computing the average binding energy per nucleon and all of that -- surely it is available somewhere more easily? I'm no physicist and would prefer not to have to try and fumble through the calcuations. Anybody have them at hand (or willing to do the calculations for me?)? I'll donate an SVG version of the resulting graph to Wikipedia if that's worth anything to you. --24.147.86.187 04:24, 7 January 2007 (UTC)

I'm afraid I don't have a terribly easy way to get these, but I can tell you the equations in a simple and friendly manner (I hope). Look at List of isotopes (actually a link to a page per element). For any isotope you consider interesting (probably the ones with natural abundance scores in the next-to-last column), take its isotopic mass (fourth column) and subtract 1.00779487489Z (second column) and 1.00866491979N (third column) to account for protons and electrons (first number) and neutrons (second number). You should get a small negative number (the mass defect); drop its sign. Multiply by 931.493581874 (this converts amus to MeV, which are the units in that plot) and divide by the mass number (the first column, or  ; this is also the abscissa in that plot). As an example, for oxygen-16 I get a mass defect of -0.136763737825 u, which corresponds to an ABEPN of 7.96215900105 MeV (remember to divide by 16) — good agreement with the existing plot.
Well. After writing all that, I've just noticed how to get the raw data from that list's reference; what you want is the left-hand column under Binding Energy/A. Note that it's in keV, unlike the plot we have. Hope this helps, even if the majority of it is just background. --Tardis 06:21, 7 January 2007 (UTC)
That's all very helpful, thank you. --24.147.86.187 15:55, 7 January 2007 (UTC)
OK. I tried to use a combination of that data set and your instructions. Here was my Excel equation:
=(([isotopic mass]-((1.00779487489*Z)+(1.00866491979*N)))*931.493581874)/(Z+N)
I have substituted column references with their meaningful variables, there, and I apologize for all of the nesting (I wanted to make sure it would do the order of operations correctly). Now this all looks pretty good except that I end up with what looks like exactly negative values of what I should be getting (oxygen-16 gives me -7.962159004). Now I know I can just multiple by -1 to reverse the sign, but why is it doing this? Have I made an error? --24.147.86.187 16:32, 7 January 2007 (UTC)
I mentioned that you would get negative numbers and should drop the sign on the mass defect. The reason is just semantics: is the energy (or mass) difference between free nucleons (heavy) and ones in a nucleus (slightly less heavy) positive because energy is released when you combine them or negative because the result (not counting emitted energy) has less mass than the raw materials?
As far as the order of operations goes, you can write that most succinctly as   (decimals abbrevd.) in any program.
However, it occurs to me that I omitted one effect -- the ionization energy of the atom shows up as a further mass defect. Ionization energies are much smaller than the nuclear binding energy, but it will cause an error which in extreme cases (heavy elements have much greater total ionization energy) may be as big as 2% or so. Ideally you need to add up all the ionization energies for an element (regardless of your isotope; this is still an approximation, but a very good one) in eV and so divide by a million to combine with your MeV number:   where I have done the sign flip on the mass defect explicitly. I realize, though, that this is rapidly getting complicated; it's probably best just to use the published data. --Tardis 21:02, 7 January 2007 (UTC)

Waterjets for teeth dangerous? edit

My dad said that Waterpik waterjets (used instead of flossing) are actually dangerous, because the force of the water pushes bateria and such deeper into the gums and even the bloodstream. Is this true, or even possible if used properly? --RockMaster 05:33, 7 January 2007 (UTC)

Follow the directions that come with the device. I looked around and found mostly good results. Excessive use can cause some gum damage, but nothing with the enamel. --Zeizmic 13:22, 7 January 2007 (UTC)
I do not think I have ever heard of a waterpick being dangerous. This is slightly off topic, but I thought I should mention that water picks are not be considered to have replace dental floss. Maybe someone that knows physics better than I would could explain the problem based on forces placed on fluids, but flossing and brushing are considered the most effective preventative measures. Around your teeth, the gums dip down slightly, forming a little trench that circle the teeth. This trench (called a sulcus) is filled with gingival fluid. Throwing water onto the sulcus is not going to be sufficient to wipe away food or bacteria/plaque present. Only physically disturbing the area, such as when using dental floss, can reduce or remove the stuck material. Waterpicks have their uses, which may be why your dentist suggested you to use it because of your particular situation. Your dentist would be able to recommend which locations a waterpick would benefit you, but dental floss will not be replaced by waterpicks anytime soon. - Dozenist talk 15:07, 7 January 2007 (UTC)
Err...it's Waterpik, as in the company. He mentioned a specific case about their pumps being too powerful. And, according to their site at least, their jets are intended to replace flossing. And shouldn't a concentrated jet of water be just as effective, if no more because it's a liquid? Industrial waterjets are very efficient and often just as effective as their solid conterparts. But, this is really neither here nor there. I was just wondering if there has been any actions against the company's jets. --RockMaster 22:44, 7 January 2007 (UTC)
I don't see how water could forcibly push bacteria that are supposadly inside the muscosa of the gums further in—there's a medium between the outside surface and the inside. Waterpiks are supposed to be the best possible form of dental care when used properly aren't they? Of course "talk to a dentist for dental advice." ;) X [Mac Davis] (DESK|How's my driving?) 05:02, 8 January 2007 (UTC)
While industrial waterjets can be quite effective, bear in mind their primary advantage AFAIK is they're easier to use and mechanise then solid action with significantly less wear/degradation (i.e. they cost a lot less). Also bear in mind they probably don't face quite the same limitations teeth water jets have with respect to safety requirements Nil Einne 15:03, 13 January 2007 (UTC)

Well, thank you all for your help; I think I've got just about everything I need. This should be enough to convince him. A good night to all, and to al a good night. --RockMaster 00:38, 9 January 2007 (UTC)

Tilt-shift Photography edit

So I've been wondering this since every time I've seen a photo taken with tilt-shift lenses, but I haven't ever found an explanation. What exactly makes a photo taken with a tilt-shift lens look like it was taken in miniature? What is responsible for the "miniaturization?" Maybe another way of putting it is what is the visual difference between a normal scene and a miniature version of that scene?

I think any discussion would be a great way to expand the stub on tilt-shift photography. For an example picture, see this picture. -Michael 05:42, 7 January 2007 (UTC)

A miniature version will all be effectively at one focus (that is, even a small depth of field can focus all of it) and will show very little perspective. Both of these effects arise because it is typically viewed from a distance much larger than the distance from "front" to "back" within the scene. The article on tilt-shift mentions a removal of perspective, so that makes sense; the linked article on faking mentions focus, so that makes sense too. Does that help? --Tardis 06:31, 7 January 2007 (UTC)
From what I can tell, the opposite appears to be the case; the focus is exaggerated, so that only a very small area of the image is in focus. This is a particularly good example that I found on Flickr. Because the trees in the foreground and background are both unfocused, the town in the middle looks very small. I think this effect occurs because focus is proportional to 1/distance to lens. Because the graph for 1/x is steepest for very small values of x, the difference between focus on very small objects is far more exaggerated than very large objects, where since x is often taken to be infinity in photography for objects the size of buildings, the focus is the same for the whole object. The brain notices this gradient of focus, and works out a scale for the object based on this. Laïka 13:39, 7 January 2007 (UTC)
The math in the second post makes sense for sure, but I think that parts of the first ring true as well. I'll digest this a little and see if i can't add anything to the tilf-shift page. Thanks for the answers. -Michael 06:14, 8 January 2007 (UTC)
Or it can simply because we know (through our eyes and macro photography) that at same focal length and same aperture, close focus will have smaller DOF than focused at infinity. Since landscape pictures like that can't possibly be taken with such a low DOF (which would require extremely big aperture), our brain thinks it's a miniature version shot as a close up instead. It has nothing to do with tilt-shift lens, although before the advent of digital image editing, the only way is to tilt your film in the opposite direction to your scene to reduce the DOF, and reverse to keep everything in focus without using very small apertures for specific scenes. --antilivedT | C | G 07:03, 9 January 2007 (UTC)
Also this is a lot more comprehensive than tilt-shift photography --antilivedT | C | G 07:27, 9 January 2007 (UTC)

What is type of this huge spider? Please let me know if you can figure out. edit

Here is my space, You can see the picture I took http://lionongshop.spaces.live.com — Preceding unsigned comment added by 211.30.34.134 (talkcontribs) 10:03 7 January 2007

I would guess that it is a member of the Argiope genus. BenC7 01:51, 8 January 2007 (UTC)

confidentiallity edit

'All that may come to my knowledge in the exercise of my profession or in daily commerce with men, which ought not to be spread abroad, I will keep secret and will never reveal.' This is a part of the Hippocratic oath, but can I still trust a doctor not to tell anyone anything, since apparently the hippocratic oath isn't used much any more?Hidden secret 7 11:56, 7 January 2007 (UTC)

Depends partly on where you live, but see physician-patient privilege.--Shantavira 14:02, 7 January 2007 (UTC)
This is taken as absolute in the US, especially if you make a point of reminding the doctor confidentiality is important to you. Breach is grounds for a malpractice suit. There are lots of flaws in the US system but we sacrifice much (including convenience and even quality of care sometimes) to maintain confidentiality. See HIPAA. alteripse 14:31, 7 January 2007 (UTC)
Although it is not legally binding, many MD graduates do take the hippocratic oath upon graduation. Indeed, to many (the current interpretations of) this oath are more important than the law. If you have embarrassment about something you are considering talking to a doctor about, consider that physicians see a lot people with a lot of different problems, and are likely to have seen whatever your problem is many times before. If you are considering disclosing illegal or illicit behavior, consider that a physician is bound by the privilege discussed above. Though a hospital may be able to hold an individual for a short amount of time (usually no more than 3 days) if that person is a threat to himself or others, the treatment and reason for treatment cannot, under most circumstances, be disclosed to police or prosecutors. Tuckerekcut 20:28, 7 January 2007 (UTC)

The end of time ? edit

I have been re-reading The End of Time by Julian Barbour. As far as I understand it, his thesis in summary is:

  • the most objective descriptions of the physical world are timeless descriptions in terms of geodesics in phase space;
  • the curve in phase space that represents our own universe happens to have one end in a rare, extremely low entropy state (the Big Bang);
  • this asymmetry in our cosmological curve gives rise to the second law of thermodynamics;
  • this asymmetry also gives rise to the psychological arrow of time, because a given state can only contain a record or memory of states which have a lower entropy (Barbour's "time capsules");
  • and so time is an illusion (although a pervasive and useful one).

Have I correctly understood Barbour's position ? Is it a respectable (although possibly minority) position in mainstream physics, or is it closer to pseudoscience ? Have any other physicists written about similar ideas in accessible publications ? Gandalf61 11:57, 7 January 2007 (UTC)

Hello, 61. What is not an illusion ? Our concepts are based upon representations derived from sensations, feelings ... and time, space, univs are useful concepts. Does it make more sense to tell that a given representation eliminates time ? Let's respect the idea and, if it becomes mainstream, why not ? But it may take time. -- DLL .. T 18:33, 7 January 2007 (UTC)
Albert Einstein said: "Reality is merely an illusion, albeit a very persistent one". JackofOz 04:52, 9 January 2007 (UTC)
Well, I struggled through that book as a sophomore a few years ago and your bullet points seem to be mostly what I understood from it, though I felt as though I didn't have a clear understanding of the text, so take that for what it is worth. Your first point and your second to last ones stand out the most for me, as I imagine a latice work of possibilities and the universe itself (or rather our experience in it) travels across that latice work. But my ability to even imagine that fails, because of course without time, there is no traveling, and the latice work does not exist in physical space, it is only a diagram. —The preceding unsigned comment was added by Muerteverde (talkcontribs) 04:44, 13 January 2007 (UTC).

Lactose and yeast fermentation edit

Hi i recently did a biological experiment investigation, involving different respiratory substrates for yeast cells Saccharomyces cerevisiae. I found that lactose hardly respired at all. Why would this be? I thought it could be due to the disaccharide formation of lactose, with galactose, having a sterio isomer which is not fermentable by yeast, but then surely i should get at least some respiration. Or perhaps another angle would be the fact thatthe yeast did not have the appropritate enzyme to break the bond between the saccharides? Any comments or opinions on this matter would be greatley appreciated. — Preceding unsigned comment added by 81.156.37.191 (talk) 08:47, January 7, 2007

Saccharomyces cerevisiae edit

Can Saccharomyces cerevisiae break down lactose? or does it not have the enzyme lactase?thanks — Preceding unsigned comment added by 81.156.37.191 (talk) 09:03, January 7, 2007

Saccharomyces cerevisiae has nearly no natural lactase activitiy. Genes which provide this activity, such as Aspergillus niger secretory beta-galactosidase, have been inserted into Saccharomyces cerevisiae via recombinent techniques, so there are experimental strains of Saccharomyces cerevisiae which can break down lactose, but no natural strains. ("Saccharomyces cerevisiae Fungemia: An Adverse Effect of Saccharomyces boulardii Probiotic Administration", Raoul Herbrecht and Yasmine Nivoix, Clinical Infectious Diseases, volume 40 (2005), pages 1635–1637: "Saccharomyces cerevisiae expresses significant sucrase and some isomaltase activity but no lactase activity") - Nunh-huh 17:19, 7 January 2007 (UTC)

liquid nitrogen in the tube edit

The tube deep lines have a problem with overheating. If they converted an old carriage to become a big tank of liquid nitrogen that went round squirting liquid nitrogen everywhere would it have any effect? Typo squotter 16:16, 7 January 2007 (UTC)

Okay, first let me explain the question. This person is talking about the London Underground (familiarly "the Tube") and specifically its deep-level tube lines (the only ones that are "tube lines" in strict usage). London Underground trains are not air-conditioned, and one reason often cited for this is that the deep tube tunnels are so small that there is no room to add air conditioners to the trains. (The fact that the subsurface lines are also not air-conditioned strongly suggests that this is not the only reason, but we're getting outside of science matters if we discuss that.) So this person is proposing a scheme to cool the tunnels instead.

My response to "would it have any effect" is: Yes, it would create a risk of suffocating the passengers by displacing too much air from the tunnels. This might prevent them from complaining about the heat, but is probably not the most desirable way to do so. :–) (Even if the amounts were properly controlled to avoid this, what if the tank developed a leak?) Using liquid air instead of nitrogen would avoid the suffocation hazard, but I believe it would substitute a fire hazard. In addition, squirting cryogenic liquid about the system could be damaging to any equipment (track, signals, wiring, etc.) that it landed on, due to thermal stress.

It's an interesting idea, though. It might be interesting to try to calculate the cooling available that way, ignoring the safety issues, but I'm not prepared to do it now. I do note that cooling the tunnels rather than the trains would only be helpful on the part of the journey when the trains actually are in tunnel; 5 of the 7 deep-level tube lines have long stretches of surface track as well, where the trains will heat up on a hot day.

You may be interested to know that the Channel Tunnel has a pipe carrying cold water through each tunnel, just for cooling. If a decision was made to cool the tunnels, this would probably be a safer approach, although there might not be enough cooling capacity in view of the fact that so much more of the cross-sectional area of the Tube tunnels is filled by the trains than is the case in the Channel Tunnel.

--Anonymous, January 07, '07, 20:52 (UTC).

Such an approach (water in pipes, not liquid nitrogen) is in fact being considered by the Tube sauce; since the River Thames soaks into the tunnels, pumps are constantly at work removing thus water from the tunnels. This water could then be used in a heat exchange system to cool the tunnels, but only at the riverside stations. Unfortunately, this means that your idea (basically the same concept) has come too late to win Ken Livingstone's £100,000 prize, though. Laïka 21:34, 7 January 2007 (UTC)

Scientific Question: Regarding Great Intelligence edit

Scientific Question Regarding Great Intelligence is asked at here, thought you might be interested, it's a little philosophical in nature though : Wikipedia:Reference_desk/Humanities#A_Person_with_Great_Intelligence. --Foundby 16:45, 7 January 2007 (UTC)

Price of Copper and Brass edit

Hi,

Does anyone know whether copper is more expensive than brass (because it's purer?) There's nothing about it on any of the wikipedia articles on either copper or brass or their talk pages. I couldn't find anything about it on the web.

Thanks, Bioarchie1234 17:40, 7 January 2007 (UTC)

ys, copper is more expensive than brass. You can get some values byt googling "copper cost per ton" and "brass cost per ton", but you'll find that the prices fluctuate widely. - Nunh-huh 18:00, 7 January 2007 (UTC)
Thanks very much! I really appreciate the help. Bioarchie1234 18:04, 7 January 2007 (UTC)

Sculpting a sphere edit

What are some good techniques for sculpting and measuring a smooth sphere? I'm pretty sure there must be an ingenious way to do it, I just can't think of anything that won't be eye-measured and prone to distortions.

The case in question is that I have a round shape already, and I want to use sandpaper to finish the sphere (by hand, unfortunately that's all I have). I need it as close to a sphere as possible. — Kieff 17:58, 7 January 2007 (UTC)

What is your sphere made of? Bioarchie1234 18:03, 7 January 2007 (UTC)
Wood. — Kieff 18:10, 7 January 2007 (UTC)
This is kind of eye-measured, but it might help:

Get two rectangular pieces of wood that are the same thickness as each other. Clamp them together to make a square(ish) shape. Make sure the sides are longer than the diameter of your sphere. Mark the centre of this square with a pencil, then, with pair of compasses, draw a circle the size of your sphere round this point. Take the two pieces of wood apart and cut out all the wood inside the circle, then file it or use sandpaper on it to get it perfectly circular. Glue the two pieces together and put your sphere in the circular hole. If you hold the square so it is parallel to a flat surface like a table, and so it passes through the diameter of the sphere, and then roll it around the table, you should be able to tell where it is bumpy because it will get slightly stuck. This might only work in principle, but it might work in practice too. I know you were looking for a more scientific method, but this is the best I could do! Hope it helps, if not, leave another message here or on my talk page. Bioarchie1234 19:04, 7 January 2007 (UTC)

I remember a television program (Like Mythbusters -- best guess is the Discovery Channel show "Debunked: Secrets of the Stone Spheres") which discussed the Stone spheres of Costa Rica. They came up with a relatively low tech way of getting the stones to be spherical, although I can't remember what it was. -- 22:32, 7 January 2007 (UTC)


This reminds me of when I learned how to grind a spherical mirror to use in a telescope. The strange thing being that it is a lot easier to grind a perfect sphere then you'd think, all that it involves, surprisingly, is randomness! Take for example a handful of clay, or mince, and roll them randomly between your hands, like you are making a meatball, the same principle applies, if you have a high enough degree of randomness you will end up with a perfect sphere. For a spherical mirror, all you have to do is rotate the mirror and randomly grind the surface, and as if by itself, the mirror will grind to a perfect sphere. Don't really know how to apply that to a wooden sphere but it may get you thinking, I'm thinking of two planks covered in sandpaper and the sphere rolled between them, but not sure how that would work practically, if the sandpaper removes enough material, if you could get it random enough, if it would create flat spots which then might cause bigger deviations… How big is the wooden sphere? Vespine 23:38, 7 January 2007 (UTC)
I saw this done for a wood sphere once - they lined a side-loading drier with sandpaper and put the block in it. After running for a day or so, they had a sphere. The same concept as a rock tumbler I believe. --Kainaw (talk) 04:40, 8 January 2007 (UTC)

[3] is the ball race method

channel form efficiency and hydraulic radius edit

Sorry if this doesn't come under science but I think it does. Anyway: Channel form efficiency and Hydraulic Radius, are they the same thing? I'm pretty sure they are but just need to double check. Thanks. 62.136.81.50 20:49, 7 January 2007 (UTC)

Light pollution edit

How do I measure how light pollution relates to the population of a certain area? And I'm doing this for a science project (hopefully you haven't seen me on another RD yet). —The preceding unsigned comment was added by Member (talkcontribs) 21:46, 7 January 2007 (UTC).

The population of humans, or some sort of fauna? The latter would be far more interesting, but you have to remember that correlation does not equal causation. Anyway, if it is humans, as I am suspecting, then the best way to do it would be get a light metre, and test several, random, spots within a certain area, then calculate the population/km2 (or similar measurement) of that area. Do this in many places, and see what you come up with. --liquidGhoul 00:00, 8 January 2007 (UTC)
Also be wary of the moon's phase. --liquidGhoul 00:01, 8 January 2007 (UTC)
Light is not the same as light pollution. I you use a light metre directly on the sky in the middle of night when it is new moon, once with a clear sky and once when it is fully overcast, the difference should be an indication of the local light pollution. Make sure no light from street lights and such can directly reach the light metre; put it in an upward pointing cardboard tube. If done correctly, the reading for the clear sky should be the same as in a totally dark room, since the light from the stars is negligeable. Of course, the exercise is somewhat meaningless if this is done for one place only. That would be as if I invented a new temperature scale and told you that where I am it is currently 714 degrees on the scale of Lambiam: that conveys no information.  --LambiamTalk 06:33, 8 January 2007 (UTC)

Biological night vision edit

Do animals with very good night vision detect colors in "more" brightness? From searching around, what I understand is that animals like owls have more sensitive rods that allow them to see more colors in the dark. But what does that really mean? Do owls see more colors but that the colors are still in low brightness/intensity, or do they somehow see the colors in more brightness (thus they can sense better contrast, etc.)? Thanks in advance 128.163.241.210 21:49, 7 January 2007 (UTC)

There was recently an article in New Scientist where they touched on this. Apparently, people used to think animals that saw in the dark saw only in black and white. However, they now find that nocturnal geckos definitely have colour vision at night, as do certain moths. Work is still needed as to which other animals have colour night vision. The cones in these animals are much longer than in diurnal animals, closer to rods, which is why people initially mistook them for rods. In case this was unclear, Rods are the sensitive, black-and-white cells, cones are usually less sensitive, but used for colour vision. Skittle 22:46, 7 January 2007 (UTC)
Many animals also have a tapetum lucidum at the very back of their eyes, which reflects light back to the retina to give it a second chance of being detected. If the tapetum lucidum can brighten the image enough for cone cells to function, the animal will have colour vision. --Bowlhover 22:54, 7 January 2007 (UTC)

Percentages regarding gas and water components in inspired and expired air edit

I have searched etc, and to no avail. I have to do a pie chart with % regarding the gas and water components in inspired air, and then in expired air. Do you have any ideas as to the answer, or where I could source it?

Munchkin MunchkinNZ 22:13, 7 January 2007 (UTC)

% gas and water in inspired air is just a factor of humidity isn't it? If you need percentages of the components of air then that article discusses them. As for Expired or exhalation, that's got to do with cellular respiration, but I'm not sure that article talks about % humidity of exhaled air.. Vespine 23:14, 7 January 2007 (UTC)
For inspiratory air Earth's atmosphere has data which I think comes from NASA. I can't find a Wikipedia reference right now, but you can use the following to get your own figures.You have to assume that the expiratory gas you are describing is end-expiratory gas, i.e. gas from the alveoli. Otherwise the effect of dead space ventilation becomes a problem (fortunately one which one cannot illustrate in a pie chart anyway). One also assumes that you ignore trace gases (e.g. Xenon). I'll give you the normal human end-expiratory PO2 and PCO2. End expiratory gas has the following characteristics: The PO2 is 100 mmHg, the PCO2 is 40 mmHg, the water vapour pressure would be the saturated water vapour pressure at 37°C, the rest of the expired gas would for practical purposes be N2. The total pressure of the gas would be 1 atmosphere. The partial pressure of a specifc gas divided by the total pressure of the gas mixture gives you the percentage of gas in the mixture (why is that?) So the figures you require for your graph would be: Atmospheric pressure (mmHg), Water vapour pressure at 37°C (mmHg), PCO2 40 mmHg, PO2 100 mmHg, PN2 is (Atmospheric - PO2 - CO2 - Water vapour at 37°C). Your whole pie would be 1 atmosphere, and the slices the proportion of each of the four gases. Make sure you understand why percentage of composition can be equated to partial pressure (it has to do with one Avogadro). --Seejyb 04:17, 8 January 2007 (UTC)

VSEPR Bonding edit

Doing some work with the VSEPR theory which is really not my field. Under VSEPR, what's the shape and bond angle of SCl2, and is it polar? I imagine that's a terribly simple question to you chemists out there, but I'd like to know. Thanks. Cool3 22:20, 7 January 2007 (UTC)

Sulfur dichloride should be very similar to water given the valences of the atoms involved. Two free pairs should give a roughly tetrahedral shape (thus bent for just the atoms) with a bond angle somewhat less than 109°. However, its polarity would be reversed and much reduced because the electronegativity difference between sulfur and chlorine is less than half that between oxygen and hydrogen and has the opposite sign. See also the reference on water's molecular structure linked from the article. (I also linked your theory and gave you the subscript you wanted.) Does that help? --Tardis 01:12, 8 January 2007 (UTC)
Thanks for the answer. That's what I wanted. Would the angle be about 105 degrees like water, or closer to 109 because of the different polarity? Thanks for the subscript too, I've been on wikibreak for a while and forgot the tag. Cool3 01:55, 8 January 2007 (UTC)
The article here on it gives it as 103°, even less than water's. Sorry, but I don't have any explanation for that real value, nor do I know whether VSEPR would predict it. --Tardis 04:25, 8 January 2007 (UTC)

What is oxidation? edit

I tried looking it up, but I couldn't understand it. Especially, I can't understand how aldehyde: R-CHO is derived from the oxidation of a primary alcohol. In that context, what does oxidation mean? Thank. —The preceding unsigned comment was added by Legolas52 (talkcontribs) 00:44, 8 January 2007 (UTC).

[An] oxidant removes electrons from another substance, and is thus reduced itself. And because it "accepts" electrons it is also called an electron acceptor. Do you understand this? X [Mac Davis] (DESK|How's my driving?) 05:41, 8 January 2007 (UTC)

Oxidation is the loss of electrons. I'm not an organic chemist, but I would say that the primary alcohol loses electrons, thereby allowing it to bind to the R-group. BenC7 11:06, 8 January 2007 (UTC)

This is a fairly high-school level chemistry problem. At that level, there are three mnemonics to keep in mind
  1. OIL RIG (Oxidation Is Loss of electrons, Reduction Is Gain)
  2. OIL RIG (Oxidation Is Loss of hydrogen, Reduction Is Gain)
  3. Oxidation is gain of Oxygen. To reduce, reduce the number of oxygen atoms
The one that applies here is the second OIL RIG, since the alcohol is losing hydrogen. I'm a third-year chemistry undergraduate, so yeah, I'm aware that the above is in absolutely no way rigorous. But it works for the purposes of revising for a high-school-level exam. GeeJo (t)(c) • 12:45, 8 January 2007 (UTC)

effects of solvents on styrofoam edit

On Wikipedia's "Polystyrene" article, I noticed that it mentions that there are U.S. regulations prohibiting the use of solvents on styrofoam because it "de-foams" it. What is doesn't mention is what specific kinds of non-aqueas solvents de-foam styrofoam and why. Can anyone tell me (very specifically) why solvents have such an effect on polystyrene? EDIT: Sorry, I found it.—The preceding unsigned comment was added by 68.190.7.99 (talk) 23:29, 7 January 2007 (UTC).

Probably the same reasons acetone defoams it! X [Mac Davis] (DESK|How's my driving?) 04:13, 8 January 2007 (UTC)

...why does Acetone dissolve it?

Architectural / structural terminology edit

I once lived in San Francisco's Tenderloin in a 1920s Edwardian apartment building. Looking down on it from above, it was shaped like a capital I (although on preview I see that the font here is giving me a straight, stick-shaped I, as opposed to one with a discernable top and bottom). Two store-fronts and a basement constitued the ground floor. Facing the building from the street, on the left there was a maintenance door. If you went through it, you first went through a short passageway before emerging into an open area (by open I mean exposed to the sky). All the buildings' windows that looked down onto this middle area saw an entrance into the basement, access to the boiler, and the tail end of the trash chute, among other things. I've been to the City's official home page, I've gone through the Architectural glossary and more here on Wikipedia, poured over myriad other sites, and I can not, to save my life, put a construction, building, or architectural term to this open area. Mere curiosity isn't fueling my interest; rather, it's work-related. I realize my description is poor, but I hope it will be enough. Guesses, suggestions, site referrals and more - all welcome. Thanks. Wolfgangus 00:02, 8 January 2007 (UTC)

Are you referring to a "well"? I have looked for something in the line of a "service well", but the overwhelming number of hits are for the adverb and adjective "well", and a very few as part of "stairwell". --Seejyb 16:55, 8 January 2007 (UTC)

Coming from a position of total ignorance, and in light of your efforts, I'm uncomfortable saying this, but according to what I'm reading as definitions for a "well", it doesn't seem to qualify. Service well actually has an attractive ring to me, but in usage it may prove misleading. A while back I'd consulted a construction-management friend who suggested the term draftway but this didn't fit either.

Of course, it's entirely possible that it has no name at all, it's just a structural feature, incidental or otherwise ... but even something so insignificant as a nook is honored with a title, and the space I'm trying to name was probably 10 x 30 ft.

Perhaps I can convince someone in the City offices to glance at the blueprints. Surely they're on file and in some fashion a matter of public record. At any rate, thank you so much for your efforts; I'm grateful. Wolfgangus 19:25, 8 January 2007 (UTC)

Think you're talking about an airshaft as it applies to a building. The quickest I could find an on-line reference was to this article, which, unfortunately, doesn't have a picture. 192.168.1.1 7:13pm, 8 January 2006 (PST)
Oh, wait, found one: Follow this link, and click on the link for the "Dumbell tenement air shaft" picture. 192.168.1.1 7:16pm, 8 January 2006 (PST)

Holy s**t. That's it. That's it! The diagram of the two tenements shows the general capital I shape, the opening, everything. Thank you so much; I can not adequately express my gratitude, but if I could, I could not possibly overstate it.Wolfgangus 03:39, 9 January 2007 (UTC)

Send me a big box full of money! 192.168.1.1 7:10pm, 9 January 2006 (PST)

January 8 edit

electron hotel edit

i need to know the answer to n=4

For the hotel californium n=42 is the answer.--JWSchmidt 02:38, 8 January 2007 (UTC)
Wouldn't that be n = 98 ? See the periodic table. You can check it out any time you like. StuRat 15:43, 8 January 2007 (UTC)
Ah, that would be for californium. JWSchmidt was talking about Hotel Californium, which require deep thought to understand. School-boy error. Skittle 02:43, 9 January 2007 (UTC)

Your question isn't clear. First off, what is an "electron hotel" ? StuRat 15:49, 8 January 2007 (UTC)

Hi Stu! googled to find this "Complete electron hotel assignment: Create quality representation of electron orbitals from n=1 to n=4, include filling rules (DUE: Mon, 01/08)." I expect we won't be too late! As for the hotel, it may be taken as an analogy, electrons filling orbitals as customers fill rooms (?) -- DLL .. T 18:13, 8 January 2007 (UTC)
Hmmm, and the s-orbital electrons can be "charged based on double occupancy", I suppose ? StuRat 05:25, 9 January 2007 (UTC)

Diffuse/Lambertian reflection edit

The Lambertian reflectance article says that it is used as a model for diffuse reflection, and for most of its mathematics links to the Lambert's cosine law article which says that blackbodies obey it (the Stefan-Boltzmann law article corroborates this). However, while diffuse reflection provides equal intensity in every direction, and I was under the impression that black-body radiation carried that property (see for instance the end of the linked section: "the radiation is the same in all directions"), the cosine law does not appear to do so. What gives between these three concepts, each of which is attached to the other two in inconsistent ways? --Tardis 02:21, 8 January 2007 (UTC)

What gives is foreshortening. Read the second paragraph of Lambert's cosine law, and the "Details of equal brightness effect" section, again carefully. —Ilmari Karonen (talk) 00:32, 9 January 2007 (UTC)
I actually understood that; I was just trying to figure out how the radiation could be "the same in all directions" as mentioned in the BBR article. I have since determined that the specific intensity (measured along a radius from the source) is isotropic although the irradiance is not (in terms of the normal to the surface with which it is measured). I wonder if the irradiance is isotropic (for the directions toward the source) for an infinite planar black body? --Tardis 19:27, 11 January 2007 (UTC)

Provisions for recharging electric vehicles from the grid by cities edit

What cities offer on street parking meters and off street parking spaces which are fitted with electric vehicle recharge stations (including area or spaces reserved for electric bikes under 750 watts) and ask a lower price for recharging than the equivalent energy cost of gasoline? -- Barringa 21:13, 7 January 2007 (UTC)

Note that you can't just do a straight comparison of the amount of electricity to drive X distance versus the amount of gasoline to travel the same distance. This is due to the relative efficiency of electrical power versus gasoline is higher for stop and go traffic than constant speed travel. This is because a gasoline engine still must burn fuel to idle, while an electric engine doesn't need to do so. Thus, the cost advantage of electricity will be higher (or the disadvantage lower) for city driving and rush hour driving. StuRat 15:33, 8 January 2007 (UTC)

Is meme a scientific concept? edit

Is meme a scientific concept? How does one find out? Is there a procedure to determine if a concept is scientific or not? 202.168.50.40 04:21, 8 January 2007 (UTC)

There is no procedure to determine if something is "scientific" or not, and this bothers some people, and it may or may not be good. There are however, a lot of qualifications that we like to say. I recommend reading: falsification, verifiability, Occam's razor, scientific method, logical fallacy, double-blind, skepticism, peer review, and scientific misconduct. Others are welcome to provide more attributes and related information. However, this is something a bit different. I would say this is a scientific concept, if I had the option of saying it was or it wasn't. This is based on its relation to science, although you could say anything really is related to science, since it can be described most likely somehow scientifically. X [Mac Davis] (DESK|How's my driving?) 05:18, 8 January 2007 (UTC)
I think the overall link you are looking for is demarcation problem, which is specifically about the difficulty in coming up with rigorous criteria for what is and isn't "scientific". --24.147.86.187 15:40, 9 January 2007 (UTC)

I would say that gene is much more 'scientific' than meme; the former has a huge number of ancillary terms surrounding it, while meme does not. Vranak

Gods, there's the problem of measuring how scientific words are? Scientifically related sounds better. "Tau meson" is more scientific than "polyethalane" though? There have got to be ranks in science, with something like, brane cosmology, superstring theory, spintronics, and particle physics being way up top. X [Mac Davis] (DESK|How's my driving?) 13:18, 8 January 2007 (UTC)
I'm just using my intuition, as usual. Vranak
In science we study many concepts that are not scientific of and by themselves: hunger, jealousy, pollution, taste, and so on. The scientific method involves hypothesis testing. The aim is that the process is reproducible, so that other scientist who independently test the same hypothesis will reach the same conclusions. This requires that concepts used in the hypothesis be defined in a very precise way (for instance by operationalization), so that the interpretation is not up to the subjective whim of the individual scientist. It is only in the context of this process that the concept becomes a scientific concept. For some concepts (temperature, energy) there is broad consensus among scientists what definitions to use, so you do not have to supply them each time; they are understood. For a concept like jealousy, no such consensus exists and you'd have to supply an operationalization if you want to make it a subject of scientific research. The same applies at present to the meme concept. One can study certain aspects of the reproduction of human cultural behaviour, where said aspects conform to the notion of memetic transmission. To do so in a scientific way, you probably need to be more precise about how you define the notions involved in your study than by saying "self-propagating cultural unit".  --LambiamTalk 13:50, 8 January 2007 (UTC)

People such as Ed Wilson have explained the biological basis for memes. The main problem is that while the physical basis of genes has been very well defined by molecular biologists the neural networks that allow memes to exist are still being investigated....it is a much more technically difficult problem. Secondarily, the idea that our genes have influences on our mental lives it not currently a popular idea within the soft sciences. Anyone who studies the links between genes, memes and culture is attacked by thought police who are not interested in the science. --JWSchmidt 18:24, 8 January 2007 (UTC)

From what I have read, it is not a hard scientific concept. I have read of criticism leveled at Wilson because of this (I believe in a New Yorker article earlier this year). It is an alluring concept and sounds convincing, but is not anywhere near the same as a gene. But a lot of sociology is not as cut and dry as the stuff of "hard science" so it can be held to a different standard if we don't put it side by side with genes, proteins, and hormones (for example). Also, give it more time, as it is a new concept still and if it has merit, we can hope to see it become more popular and eventually more accepted. Muerteverde 04:54, 13 January 2007 (UTC)

Atmospheric pressure edit

What would the atomospheric pressure be at an elavation euqual to the bottom of the Atlantic seafloor, but uncovered by water? In other words, if there was an open place on earth where the elevation was that low, would the pressure still be significantly different than sea level?

Klatue 06:23, 8 January 2007 (UTC)Klatue

(I fixed the spelling of the question title.)

This table is extracted from the 1986 edition of the "Rubber bible". The left-hand column is depth below sea level in meters, the right hand air pressure in millibars.

                   0   1013.25
                 500   1074.7
                1000   1139.3
                1500   1206.9
                2000   1277.8
                2500   1352.0
                3000   1429.7
                3500   1510.9
                4000   1595.9
                4500   1684.8
                5000   1777.6

The book does not say how the pressures were determined, and does not go to greater depths, but it's easy to extrapolate approximate values to moderately greater depths. Several of the Atlantic's deep basins have points between 5000 and 7500 meters deep, according to the atlas I checked, so the pressures you're askingabout could go to somewhat over 2000 mb, maybe up to around 2300 mb. --Anonymous, elevation 130 m (or so), January 8, 2007, 08:44 (UTC).

Aren't there mine shafts that deep ? In this case, wouldn't it simply be a matter of measuring the air pressure at the bottom of the shaft ? StuRat 15:21, 8 January 2007 (UTC)
I don't know the answer to the original question, but the deepest mine is about 3585 meters [4] or about 11,700 ft. Cheers Geologyguy 19:04, 8 January 2007 (UTC)
That source has the quote "...nobody has ever been able to drill down further than 15 km". So why don't they consider a 15 km deep hole to be a mine ? In any case, assuming the hole is filled with air, it seems possible to drop a probe to the bottom to measure the air pressure. StuRat 05:18, 9 January 2007 (UTC)
Why isn't it a mine? Because people don't go down it and extract minerals. --Anon, Jan. 9, 22:29 (UTC).
The minerals that were in the location which is now a hole were certainly "extracted" from the hole. So, is the fact that people don't fit down it what makes it no longer a mine ? StuRat 04:21, 10 January 2007 (UTC)

Hall effect sensor/switch identification edit

I'm trying to identify a Hall effect sensor or switch which I am pretty sure is a switch instead of a linear sensor because three of them are recessed into the stator to commutate a 3 phase DC brushless motor through a controller with what I judge to be a square wave from the amount of torque ripple the motor produces under load at startup. Once up to running speed, however, the motor purrs. The only reference I have is a barely visible designation on two of the sensor/switches which is: "F41.5Gc". The "G" in the "5Gc" could be an "0" or a "6" but I'm with 99% confidence sure it is a "G". It is most likely manufactured in Asia, probably China but possibly elsewhere. Allegro sales office says they are not able to identify it and Google produces no reference or clues. Thanks in advance. Barringa 08:27, 8 January 2007 (UTC)

This is a SS41 Honeywell equivalent bipolar switch with a hys. of about 30. 71.100.10.48 02:41, 12 January 2007 (UTC)

radio broadcasting edit

What is the name of the phenomenon in which a radio frequency transmitter transmitting with enough power, causes undesired sound to come from the speakers of and audio system. In every instance that I've heard of it occuring, it was caused when someone has audio system too close to a comercial radio station tower or an amateur transmitter that is using too much power.

Thank you for your help.146.53.3.4 08:38, 8 January 2007 (UTC)

See Audio feedback. Natgoo 10:46, 8 January 2007 (UTC)
I don't think the audio feedback article is relevant. There is no signal feedback from receiver to transmitter, either audio or otherwise, is there? The problem is to do with the signal being to strong, but I don't know of any name for the "undesired sound".--Shantavira 11:12, 8 January 2007 (UTC)
Whoops you are correct. I misread the question in my not-yet-caffeinated state. Sorry. Natgoo 11:59, 8 January 2007 (UTC)
Interference caused by Electro-Magnetic waves of Radio Frequency. In Wikipedia, Radio frequency interference (RFI) redirects to Electromagnetic Interference. Apart from its "usefulness" application in electronic warfare, are there beneficial applications of the phenomenon? It would be unusual if no one has found such a useful application, but I can not think of one. --Seejyb
You might be hearing the RF transmission because it is saturating an amplifier circuit, or because of inadequate shielding or a poor connection on a low level input circuit. I have heard a voice radio signal come out of the speaker of a stereo console, even with the unit turned off. In that case it was either Citizens Band or "Ham" radio, but no aspersions, it just happened he was nearby and a commercial transmitter wasn't. The speaker wiring was acting like a Crystal radio, picking up signal voltage from the radio transmitter, and rectifying it by an imperfect metal to metal contact of a loose speaker connection. There was apparently enough capacitance in the speaker or the splitter capacitor between the bass and treble speakers to serve the function of integrating the RF voltage to make the modulating voice signal audible. In times past, radio experimenters would make a detector out of a broken razor blade, and listen to AM signals through headphones, also with no amplification. They would have an inductance and capacitance combination to tune the reception, but if there is a powerful enough local transmitter, it may drown out the others avoiding the need for tuning. People have reportedly heard radio transmissions through fillings in their teeth. Sometimes a Public address amplifier system picks up radio stations or aircraft transmissions for similar reasons, and bypass capacitors are used to shunt out the RF signals from the microphone inputs. Edison 13:23, 8 January 2007 (UTC)
This reminds me of the time I was in the kitchen making breakfast when my boom box, turned off at the time, said to me (quite loudly) "How's it going, good buddy ?". I, of course, said, "It's going just fine, Mr. Radio, and how are you today ?". StuRat 15:17, 8 January 2007 (UTC)

Gravity edit

From The Core article:

  • Moving closer to the center of the Earth should result in a decrease in the force of gravity, but no such effect is shown in the movie.

Really? Why? And how would one observe such an effect—that is to say, what would happen? --210.246.30.238 10:21, 8 January 2007 (UTC)

Lets say you are in the middle of the earth. Half of the earth is above you and half below. They each attract you with equal and opposite force and cancel out. You feel no force. Something like that.WP 10:39, 8 January 2007 (UTC)
Gravity is caused by attraction of objects. When you bore towards the center of a solid sphere, there is less matter below you to pull you down. So you get lighter. At the center you get zero gravity (which way would it point anyway?) The force of gravity is as if all the mass above you didn't even exist; only the mass below you matters. For a mathematical formulation, see the "solid spheres" section of shell theorem. Weregerbil 10:42, 8 January 2007 (UTC)
(after ec) Great film, but dreadful science ! Our article on Earth's gravity says:
If the earth was of perfectly uniform composition then, during a descent to the centre of the earth, gravity would decrease linearly with distance, reaching zero at the centre. In reality, the gravitational field peaks within the Earth at the core-mantle boundary where it has a value of 10.7 m/s².
As Weregerbil says, the decrease in gravity inside a sphere is due to the "shell effect" - inside a spherically symmetric hollow shell, the gravitational attraction due to the shell is equal in all directions, so the net gravitational force on an object inside the shell is zero. The shell theorem article contains a detailed mathematical derivation of this effect. You could observe the effect because objects in the hypothetical capsule would weigh less as it approached the centre of the Earth. Gandalf61 11:03, 8 January 2007 (UTC)

For convenience the earth is considered to be a point mass, therefore in calculations like this all its mass is located just at the centre of the earth. Gravitiy is proportional to distance from the centre, and I have forgotted exactly what my point is, but I think I have just disproved it. Or I might not have.Hidden secret 7 20:57, 8 January 2007 (UTC)

The Earth's gravity resembles that of a point mass if you're not inside the Earth. As soon as you descend a significant depth, gravity decreases for the reasons explained above. --Anonymous, January 8, 00:22 (UTC).

Gravity of the moon? ...the sun? edit

The discussion above leads me to wonder: What is the value (in Gs, in the vicinity of the Earth) of the moon's gravitational attraction? ...of the sun's gravitational attraction? It's obviously non-trivial since it leads to tides on the Earth's surface, but I've never noticed the value being stated.

Atlant 15:24, 8 January 2007 (UTC)

Well, let's see ... the distance to the Moon is about 64 times the Earth's radius, and its mass is about 1/80 of the Earth's mass, so the force of the moon's gravitational attraction at the Earth's surface is about 1/(80x64x64) of the Earth's surface gravity, so about 3x10-6g. The distance to the Sun is about 25,000 times the Earth's radius, but its mass is about 33,000 times the Earth's mass, so the force of the Sun's gravitational attraction at the Earth's surface is about 33,000/(25,000x25,000) of the Earth's surface gravity, so about 5.3x10-4g. So the Sun's gravitational attraction on the Earth is much greater than the Moon's (which makes sense, otherwise the Earth would be in orbit around the Moon instead of around the Sun), but both are miniscule compared to the Earth's surface gravity.
However, the Moon has a much greater effect on the tides than the Sun does because the tides are caused by the gradient of the gravitational field, not its strength. As our article on tides says: Though the gravitational force exerted by the Sun on the Earth is almost 200 times stronger than that exerted by the Moon, the tidal force produced by the Moon is about twice as strong as that produced by the Sun. The reason for this is that the tidal force is related not to the strength of a gravitational field, but to its gradient. The field gradient decreases with distance from the source more rapidly than does the field strength; as the Sun is about 400 times further from the Earth than is the Moon, the gradient of the Sun's field, and thus the tidal force produced by the Sun, is weaker. Gandalf61 15:54, 8 January 2007 (UTC)
As an aside of sorts, the gravitational effects of the moon are actually great enough to produce earth tides, up and down motions in the rocks themselves. These are on the order of an inch or so, but are measurable with Gravimeters and must be corrected for in gravity surveys for oil exploration, for example. Cheers Geologyguy 16:03, 8 January 2007 (UTC)
Thanks! Yes, I read the Tide article, but I was interested in the absolute magnitude of the force rather than the tidal effects, so thanks for the calculations. And with regard to earth effects, I understand (per a recent article in New Scientist magazine) that scientists are starting to believe that these tidal forces may serve as part of the ultimate trigger for earthquakes.
Atlant 17:05, 8 January 2007 (UTC)
I was told that the moon has 3 times the gravitational tidal influence of the sun in my Nautical Science classes. —The preceding unsigned comment was added by 138.29.51.251 (talk) 17:37, 8 January 2007 (UTC).
If you want the absolute magnitude of the force, then the force that the Sun pulls on the Earth (and that the Earth pulls on the Sun by Newton's 3rd law), is around 3.5×1022 Newtons - roughly the amount of force needed to break a high-tensile steel cable around a mile or two in radius. The force that the Moon pulls on the Earth (and hence also that the Earth pulls on the Moon) is around 2.0×1020 Newtons. Richard B 13:34, 9 January 2007 (UTC)
Gandalf61, you're a bit light on the Sun's mass. The Sun is ~330,000 times the mass of the Earth, but somehow you've come to the right final answer for the gravitational field strength from the Sun. ;-) If you want to work out the tidal force, then you just need to work out the gradient of the field strength, which is proportional to the mass divided by distance cubed.
So the ratio between them is (using the numbers above);
Tidal force (Moon)/Tidal force (Sun) ~ Mass (Moon) * Distance (Sun)3 / (Mass (Sun) * Distance (Moon)3
Ratio ~ (1/80) * 25000^3 / (330000 * 64^3) ~ 2.3 Richard B 13:28, 9 January 2007 (UTC)

Work-Energy theorem edit

Is Work-Energy theorem valid in a non-inertial frame of reference?

Assuming you mean dEkin = F·ds , where F = dp/dt, I think the answer is no. See Special relativity, which gives formulas for kinetic energy and force.  --LambiamTalk 14:28, 8 January 2007 (UTC)
A reference frame is called non-inertial if it is undergoing some acceleration a relative to an inertial reference frame. The acceleration will give rise to an apparent gravitational field g=-a in the non-inertial reference frame. There will therefore be a gravitational term to the force, leading to a potential energy term in the energy equation: dEkin + dEpot = F·ds, where dEpot = m g dz, where dz is the displacement in the direction of the acceleration. This is just like the reference frame we live in on Earth, where there is gravity due to the nearby mass of the Earth. According to Einstein's general theory of relativity, such gravity has exactly the same effect as an acceleration of the reference frame. --mglg(talk) 21:12, 9 January 2007 (UTC)

work done by a force edit

I came across something which seemed peculiar to me - "No work is done by a force on an object if the object moves in such a way that the point of application of the force remains fixed."-Is this true?How? —The preceding unsigned comment was added by 59.93.76.73 (talk) 14:28, 8 January 2007 (UTC).

I'd say no, that's false. Consider two rollers with a tube between them being pulled through the rollers and raised in height. The location of the contact between the rollers and the tube remains constant, yet the tube ends up higher, and thus with more gravitational potential energy, showing that work was done. If they meant that "the point of application of the force remains fixed relative to the moving object", we could disprove that with the example of a rocket. The force remains at the end of the rocket during the entire burn, but still results in work being done on the rocket, again as shown by it ending up higher, with more gravitational potential energy. StuRat 15:06, 8 January 2007 (UTC)
Yes, the statement is true—provided that you're careful about your definitions. The 'point of application' of the force must remain fixed relative to two points: the object itself, and a fixed external frame of reference. In the 'roller' example above, this condition doesn't hold, as the point of contact on the tube is being moved. In the 'rocket' example, choosing the rocket itself as a frame of reference doesn't make sense.
From a technical (some might say semantic) standpoint It falls out of the definition of work (see mechanical work for details and formulae), that requires force to be applied over a distance for work to be done.
From a common sense perspective, the quoted rule applies to 'static' situations, like a brick sitting on a table. There's obviously a force being applied to the brick by the table, and vice versa :gravity pulls down on the brick, the table resists the brick falling through it. There isn't energy being created out of nothing by this interaction; the brick and table don't get warmer, the brick doesn't levitate, etc. There is a force being applied to the brick, but the point of contact is fixed, hence no work is done. TenOfAllTrades(talk) 18:22, 8 January 2007 (UTC)
But in your example the object is still, do you have an example with a moving object ? StuRat 04:47, 9 January 2007 (UTC)
That was troubling me too, would a brick on a table, falling off a cliff count? Vespine 05:40, 9 January 2007 (UTC)
In general, with an inertial frame and no relativistic velocities, it's quite simply that   at all times. (  can confusingly be the net force on the object or the force from one other object, depending on whether you want to say "No work is being done on this object." or "That object is doing no work on this one.".) This means that something in a circular orbit counts, because the force is in towards the planet or so and the velocity is around it. The trick is that it depends on your reference frame: if you are drifting by the planet in the plane of the orbit, you say it is doing work on the satellite, because the satellite is moving faster when it's moving the opposite way you are (relative to the planet) and slower the other way, so its kinetic energy is changing. It also of course works when  : if you watch a train go by, you say the tracks are doing work on it to make up for losses to air drag, but I on the train say that the train is not imparting energy to the air (merely diverting an oncoming wind) and is just clinging to the moving tracks to not be blown backwards. (I also say that since the train is pushing on the moving tracks, it is doing work on them!) Finally, in order for the "no work" condition to persist, it must also be the case that  , where   is acceleration. This means that you won't see it regardless of your frame for gravity's effects on things falling at small altitudes: the gravitational force is constant ( ) so the only term is  , and that would only be zero if   since they'll both point straight down. (You might argue that at terminal velocity this is the case, but there   in the first place.) Does that clarify it at all? --Tardis 18:08, 9 January 2007 (UTC)

What is the antonym of feral? edit

What is the word that describes a wild animal that appears domesticated. This usually happens on an individual basis, like a deer you can feed out of hand. There are stories about wild animals that seemed tame, but then just take off for the wild again.

The word isn't taming, nor is it domestication. It is a rare, technical term. I saw it once in 'New Scientist', but I can't remember what it is for the life of me. Can you help?

Meredith15:52, 8 January 2007 (UTC)15:52, 8 January 2007 (UTC)~

Habituated? —The preceding unsigned comment was added by 138.29.51.251 (talk) 17:38, 8 January 2007 (UTC).

That'll work! Thank you so much!

Gas Leak edit

What are some possible consequences of a city wide natural gas leak? Assuming also trace amounts of Mercaptan, other than the obvious smell of sulfur, which is unpleasent to say the least--66.65.156.162 16:03, 8 January 2007 (UTC)

A good place to start would be an MSDS for natural gas. (Here's one, in PDF.) This outlines some of the health and other hazards associated with natural gas. At moderate concentrations, natural gas is highly flammable; at very high concentrations it is an asphyxiant. TenOfAllTrades(talk) 18:26, 8 January 2007 (UTC)
Today's event bordered on mass hysteria. I have seen no proposed mechanism for a city-wide natural gas leak.
The temperatures were warm over the weekend and the river and sewers had that nice ripe smell they have in the summer. Normal, but a bit out of place in January. Evacuating office buildings and department stores and telling pregnant women to go home was absurd. -THB 04:09, 9 January 2007 (UTC)
I think something like that happened in 1992 in Guadalajara, Mexico, where gas leaked into the sewers and dispersed. I believe it detonated before it leaked out of the sewer throughout the town, but still killed around 200 people: [5]. It's not hard to imagine an even more serious event, especially if no odorant was added. StuRat 04:58, 9 January 2007 (UTC)

Bananas edit

I know that bananas have alot of pottasium. But, what are the benefits or dangers of pottasium? —The preceding unsigned comment was added by 216.253.128.27 (talk) 16:48, 8 January 2007 (UTC).

The article entitled potassium will tell you a lot about potassium; the section of that article entited "Potassium in nutrition and medicine" will probably be most helpful to you. DMacks 17:35, 8 January 2007 (UTC)

Clinical Physiologist. edit

I am awaiting notification as to whether I am successful in my job application for a position as a Trainee Clinical Respiratory Physiologist. I would like to know what kind of uniform i'd wear in the hospital. Would it be scrubs, normal clothes, white coat etc.

Thanks. Christopher. —The preceding unsigned comment was added by 85.210.129.119 (talk) 18:37, 8 January 2007 (UTC).

It is a matter of local custom. What did they tell you at the hospital? What were the other respiratory physiologists wearing? alteripse 23:13, 8 January 2007 (UTC)
Whatever is the norm at the facility where you are working. Generally, scrubs or street clothes with a lab coat. Trainees or students may have to wear distinguishing uniform such as a short lab coat or a different color of scrubs. -THB 04:12, 9 January 2007 (UTC)

Darwinism; the human race compared to other animals edit

As I understand it, Darwin explains the existance of so many species by a very slow process of evolution whereby the fittest survive. Did he, or any of his successors postulate any explanation for the fact that, although all species appear to have changed over time, the changes in homo sapiens and the capacities that have been developed as a result of them seem so much more spectacular? Put another way, why has no other species developed the use of language, artistic abilities etc etc beyond a rudimentary level?

This is science, so most of it is random chance. And no other animals have much chance to develop art &c, as they are too busy trying to survive.Hidden secret 7 19:39, 8 January 2007 (UTC)

Some other animals have some kind of language like: Birds, Dolphins and Whales. --helohe (talk) 19:55, 8 January 2007 (UTC)
Also, the development of culture is fairly recent in evolutionary time, and once it happened our species quickly achieved a huge advantage (advanced language and technology). In short, it seems spectacular because we were the first ones who got this far, and because it happened just a while back. — Kieff 21:14, 8 January 2007 (UTC)
The chance of two species simultaneously developing high intelligence is very low. Adding to what Kieff said, if another species were to be the first to develop language skills, don't you think someone from that species would be asking the exact same question? --Bowlhover 21:15, 8 January 2007 (UTC)

I think your question is this. Human development accelerate due to positive feedback so how come development of other animals did not enjoy this positive feedback? I believe that it is the development of language and writing that give humans the advantage of positive feedback. If other animal species develop language and writing then they will also "enjoy" positive feedback on their development.

As for why other animals did not develop language and writing, I believe it is because of their body structure, mainly their voicebox and the hands. It is a complete fluke of environment that lead to the development of a suitable type of voicebox and hands for human beings. 202.168.50.40 23:59, 8 January 2007 (UTC)

Why don't snails have wings? Why don't frogs have beaks? Why don't humans have eagle eyes? Why don't worms have four-chambered hearts? We can go on and on about why certain species don't have specific traits. I think the reason primates reign is because we've become so powerful that we've impeded whatever development other species have towards becoming more specialized. In other words, only one kind may prevail, us primates, at the expense of other animals. And who knows, if we let dolphins live they might develop high intelligence like ours in a few million years.

But these are only elementary speculations of mine; check out this part of Darwin's Descent of Man, where he talks about why humans develop complex language while other species lack it: http://books.google.com/books?vid=OCLC00550912&id=sgDHceAj22MC&pg=RA3-PR17&lpg=RA3-PR17&dq=descent+of+man#PRA3-PA87,M1 Read the entire chapter if you like to understand it better.128.163.241.210

Humans aren't the only artistic animals, others include bower birds. StuRat 04:42, 9 January 2007 (UTC)
Insects and fish are far more 'successful' without language, there are many more per species than humans. They are successful in terms of the number of copies of their genes are around. In evolution species fill a niche, i.e the working towards the best solution in their particular environment. There is no "perfect" ultimate goal of evolution, and if there is it isn't necessarily complexity and language. Just think for a minute what would happen if there was another species that had the same goals as us and the same intelligence and language skills. We have come close enough to wiping out our own species. Intelligence, art and language are successful in our environment (eating plants, animals, living in varied environments and making shelters etc), and there is no room for any other species to take over it. If an animal gained similar traits as us, they would compete directly with us, we would kill them before they had a chance to evolve. In this way evolution makes species that are not so similar as to compete that directly. 194.80.193.188 10:12, 9 January 2007 (UTC)
Darwin'sDescent of Man goes into just this issue at length. He primarily argues that you can see versions of any "uniquely human" trait in many other animals to lesser extents, and that nothing humans have are actually "unique" so much as exaggerated. An analogy—not Darwin's—might be that all animals have noses, though dogs can smell much better than most. We can recognize that the dog's nose evolved into a highly specialized organ due to some set of various evolutionary twists and turns; we don't really doubt that this is possible, since it seems pretty plausible. The human brain, then, would be similarly understood as a specialized organ, at least if we were not so 1. relatively clueless about how the brain works (even still), 2. vain about our own thinking abilities (which we elevate far higher than they probably ought to be, in the evolutionary scheme of things, and we also over-value brain power for a variety of cultural reasons, whereas evolution really does not care how you solve the fitness problem). --24.147.86.187 15:37, 9 January 2007 (UTC)

Just droppin' objects edit

I'm pretty ignorant. So I need help understanding what the deal is with falling objects all falling at the same rate. While I know and can see this to be true (doing some 'experiments' of my own), I wonder why. If all objects fall at the same rate without the influence of air resistance, when why are some of them heavier than others? If I put an object on the scales it might be 'pulled down' by Earth's gravity more or less than another object. So it would seem that the 'hardness' of the force on the object does not affect the speed that it falls at. That's pretty counterintuitive, as if I shove an object in a gravityless (?) environment hard it'll go faster than if I tap it gently (that'd probably a better analogy if I were tugging on an attached string hard or softly, but whatever). So what's the hole in my understanding? Vitriol 19:35, 8 January 2007 (UTC)

Well air resistance of course is a force that retards falling in an atmosphere, much as friction retards sliding of something sliding on the ground. Air resitance can be so great that a falling object reaches a terminal velocity, and its rate of fall does not increase any more. We do not really understand gravity. All we can do is measure it and form theories or laws about it, like Newton's law of gravity of Einstein's theory of gravity. The weight does not affect the speed in a vacuum, you are correct. Aristotle felt that the weight would affect the speed, and heavier objects would fall faster. Galileo tested this (by rolling objects on an inclined plane, not by dropping things from the Tower of Piza as the popular myth goes). Galileo found all objects fall at the same rate. So...there you have it. We do not know why really just that it happens. --Filll 19:41, 8 January 2007 (UTC)
The trick is that you have to account for mass. Recall one of the formulations of Newton's Second Law of Motion: F = ma (F = force, m = mass, a = acceleration). Rearrange and you get a = F/m; that is, to get the same acceleration of a large mass (compared to a smaller mass) requires a correspondingly larger force. Conveniently, the gravitational force scales precisely with the mass (inertia) of the object, so all objects accelerate at the same rate under the influence of gravity.
Now, there's a much more subtle underlying question that physicists haven't been able to answer satisfactorily here. You can measure mass in a couple of different ways. Gravitational mass is a measure of how strongly an object attracts other matter through the force of gravity. Inertial mass is essentially a measure of how strongly an object resists acceleration. Why is it that these two ways of measuring mass always work out to be equivalent? TenOfAllTrades(talk) 19:52, 8 January 2007 (UTC)
See, I don't think I'm understanding still as what I can glean from what you say a object with a larger mass is pulled on harder by gravity to get it moving in the first place... and gravity doesn't pull on some things harder than others. Doesn't it? Vitriol 20:02, 8 January 2007 (UTC)
Yes, gravity DOES "pull harder" on heavy objects than on lighter ones, according to Newton's Law of universal gravitation. The gravitational force exerted on one object by another is linearly proportional to the mass of the object. -sthomson06 (Talk) 20:53, 8 January 2007 (UTC)
Think about it like this: gravity pulls harder on objects with more mass. But objects with more mass are also harder to accelerate. From the equation F=ma, imagine if you multiplied m by 3 and also F by 3. 3F=3ma, so there's no need to change a. --Bowlhover 21:12, 8 January 2007 (UTC)
Actually, what Vitriol has asked about is a very beatiful and profound physical concept - the equivalence of 'inertial' mass and 'gravitational' mass. The fact that gravity pulls just hard enough on any mass to accelerate it at the same rate as all other masses is the profound insight that Einstein had which led him to realize that gravity is a geometrical phenomenon. Cool stuff. --Bmk
It's 'cool', eh? Thanks for the responses, forks. Vitriol 01:39, 9 January 2007 (UTC)
Great discussion; I don't think I've ever thought about things this way before. Does this all work out to be another way of expressing our lack of a unified field theory? Is it the fact that only gravity, of all the forces, depends on the very quantity that controls acceleration that both allows it to be interpreted geometrically and sets it apart from the other forces? --Allen 03:32, 9 January 2007 (UTC)

N.H. Horowitz edit

I have found paper after paper by this guy in biology and exobiology etc. He is a professor at Caltech. Is he famous ? Does he need his own biography?--Filll 19:37, 8 January 2007 (UTC)

This piece on Norman H. Horowitz should help you decide. Wolfgangus 21:40, 8 January 2007 (UTC)

I-V characteristics of Josephson junctions edit

Why do people investigate the I-V characteristics of Josephson Junctions, other than for the fact that the I-V curves are not what you would expect from a normal semiconductor junction? i.e. what kind of physical impact / application do the I-V characteristics have say, for example, in SQUIDs (Superconducting Quantum Interference Devices)? --RaGe 20:43, 8 January 2007 (UTC)

For a while, it was hoped that Josephson junctions might replace transistors as the basis for the high-speed electronics used in computers. IBM did a ton of research into this.
Atlant 13:50, 9 January 2007 (UTC)
cheers for the responce --RaGe 00:10, 12 January 2007 (UTC)

Looking for scientific journals edit

Today on new pages I ran across someone who has come up with a slightly different theory of the origin of the solar system. He was trying to use Wikipedia to publish his original thought. I prodded his article and explained to him why it doesn't belong on Wikipedia. Now he'd like me to suggest other places that might be more receptive. I'd like to suggest he try some scientific journals but don't know any specific journals to point him towards.

What are some names of journals where he might be able to try to get his theories published? ~ ONUnicorn(Talk|Contribs)problem solving 21:13, 8 January 2007 (UTC)

Try reading scientific journal? :) He can try ArXiv.org[6], or Physical Review D[7]. X [Mac Davis] (DESK|How's my driving?) 21:18, 8 January 2007 (UTC)
Thanks for the quick response. I'll suggest those. :) ~ ONUnicorn(Talk|Contribs)problem solving 21:23, 8 January 2007 (UTC)
You could also try spiked com 62.253.44.32 00:35, 9 January 2007 (UTC) NOTE: SPIKED COM APPEARS TO BE URL SPAM --Wjbeaty 00:37, 10 January 2007 (UTC)
And The Edge. AKA Edge Foundation. Anchoress 03:52, 9 January 2007 (UTC)
See List_of_scientific_journals#Astronomy_and_astrophysics and List of scientific journals in astronomy. StuRat 04:35, 9 January 2007 (UTC)

relation of the sun and wind edit

hello, i would like to know the role played by the sun on creation of wind on earth thank you, anu —The preceding unsigned comment was added by 59.144.29.242 (talk) 01:41, 9 January 2007 (UTC).

Have you tried reading the article wind? It has quite a bit of information in it. --18.214.1.125 02:08, 9 January 2007 (UTC)
I was going to go ahead and add a little capsule definition anyway, but I clicked on that article and the opening paragraph is very good! Vespine 02:41, 9 January 2007 (UTC)
I've just evaluated that article, and found it to be quite acceptable, as well. In other words, I've just passed wind. StuRat 04:32, 9 January 2007 (UTC)

Propranolol side effects edit

Is there any evidence that this Beta blocker can cause chronic itching all over the body in some patients suffering from hypertension?--Light current 01:56, 9 January 2007 (UTC)

Yes, itching Appears to be a known side-effect, especially upon exposure to light. DMacks 02:00, 9 January 2007 (UTC)
Ah no. My itching is where the sun dont shine (under my clothes). 8-|--Light current 02:02, 9 January 2007 (UTC)
There's your answer then, just head for the nearest nude beach. StuRat 04:28, 9 January 2007 (UTC)

Escherichia coli edit

Specifically how do virulent strains of E. coli cause the well-publicized complaints (bloody stools, diarrhea)? I was looking the question over, and couldn't really find any specific steps. bibliomaniac15 02:01, 9 January 2007 (UTC)

Have you read the e. coli article? This section seems to answer your question. Anchoress 02:09, 9 January 2007 (UTC)
This article is getting a bit old, but it describes some of the virulence factors such as shiga toxins that will damage and kill intestinal cells. --JWSchmidt 02:20, 9 January 2007 (UTC)

Horst Tube Submarine edit

Does any one know where I can get info on the Horst Tube, used in submaries by the Germans in ww11. This tube completely covered the propellor of the submarine, to increase speed or thrust. —The preceding unsigned comment was added by 203.118.142.58 (talk) 02:31, 9 January 2007 (UTC).

Are you maybe thinking of Kort nozzles? 192.168.1.1 7:05pm, 8 January 2006 (PST)

Sounds also like it could be the UW version of Ducted fan.--Wjbeaty 00:32, 10 January 2007 (UTC)

Electromagnetic Radiation from the end of coaxial cable edit

If an electromagnetic wave is travelling towards the open end of a coaxial cable, what proportion of the incident wave is radiated from the end. We may assume that the cable Z0 is 50R and the impedance of free space is 377R. THis is not a homework question as I finished my formal education many years ago. I try to learn from WP now.--Light current 02:46, 9 January 2007 (UTC)

Caps and Baldness edit

Can wearing hats/caps give you a higher chance of baldness in the long run? PitchBlack 02:46, 9 January 2007 (UTC)

See Fred Dibnah —The preceding unsigned comment was added by 88.110.24.62 (talk) 02:50, 9 January 2007 (UTC).
No, check out the baldness article. It says wearing a hat should not cause traction alopecia, which is the only cause of baldness through physically pulling the hair. Normal hair loss is genetic and caused by hormones (described well in the article). --liquidGhoul 03:06, 9 January 2007 (UTC)

A fungus among us edit

OK, thanks. Another question. Why is it that mold easily grows on bread when it's closed in the pantry, and not when it's in a open bag in my room?(I'm trying to grow mold and bacteria, but all it does is get hard) PitchBlack 03:13, 9 January 2007 (UTC)

When it's in a closed container, it retains the moisture needed to support the growth of mould. Mould is another term for fungus which is what actually forms visible mass on bread (green/white) rather than bacteria. FYI; it's customary to create a new heading for a new question or else a lot of people that may be drawn to mould questions (freaks!) might miss it, being put off over the apparent talk of 'caps and baldness'. :) Why are you trying to grow fungus anyway? Green fungus should be kept away from people with penicillin allergy in case its penicillium --Username132 (talk) 03:58, 9 January 2007 (UTC)
I've added the heading. StuRat 04:26, 9 January 2007 (UTC)

Yeah, my mistake. I wanted to post quickly to get an answer, but I also know you can't double post. PitchBlack 08:59, 9 January 2007 (UTC)

Weird science... edit

I eliminated strait glucose from my diet well over a year ago by switching to Sweet & Low and a store brand of Aspartame. I “discovered” that if I mix the two in a ration of 3 packets of Sweet & Low to 5 packets of Aspartame that I get sweetness even more intense than glucose except, but maybe not, pure cane syrup or HFCS. Its so intense that I can even mix it with plain sour yogurt and the sweetness takes over and rules. My question before I get side tracked is why is it that when I add this combination to coffee and then sprinkle some cinnamon in that I get a brown oozy slime in the bottom of the cup like extremely nasty sickening snot? What reaction causes this slime and why is there so much of it? I guess ouster lovers could suck it right down but to me it is disgusting. What is it??? -- Barringa 04:09, 9 January 2007 (UTC)

Is there by any chance any milk or cream in the coffee ? If so, perhaps you have caused it to curdle. In which case, I agree, that's whey cool. StuRat 04:19, 9 January 2007 (UTC)
No creamer or lactose derivative of any kind. Its Folger 100% Columbian coffee + Aspartame + Sweet & Low + cinnamon + less than 10 seconds and ...ha-choo! ...a couple of tablespoons of slimmy goo right in the old cup of coffee! -- Barringa 06:25, 9 January 2007 (UTC)
What color is your blob? Does that suggest it's mostly made of sweeteners, or that it has a lot of cinnamon (or coffee components) in it? What happens if you repeat your experiment with cinnamon but omit the sweeteners entirely? What happens if you use just hot water and cinnamon, with and without sweeteners? What order are you adding the sweeteners and cinnamon? If the cinnamon floats, then adding sweeteners after it could form a mixed-solids surface layer that sinks; try dissolving the sweeteners first. In my experience, powdered cinnamon does not dissolve in coffee, but floats in a thin layer on the surface at first and sometimes winds up as a gelatinous mass by the end. Same with powdered cocoa actually. DMacks 06:37, 9 January 2007 (UTC)
The dark brown color is obviously the wet cinnamon and possibly coffee. The goo is the part I'm trying to identify. Since ground cinnamon is nearly 100% dietary fiber I would assume it is for this reason sticking to whatever the goo might be. I can speculate that it is some sort of reaction of the sweetners with the fiber but was hoping there might be a chemist who already knew. Barringa 08:56, 9 January 2007 (UTC)
You have four ingredients for your goo recipe. Could you experiment and see whether perhaps some smaller subset already undergoes gooification (for example leaving out the coffee, or the Weet & Slow?).  --LambiamTalk 11:59, 9 January 2007 (UTC)
Sounds like a fine experiment. Try the process of elimination, and dump each batch through a coffee filter to extract the goo. Silly Putty , penicillin and saccharine were discovered in similar lab accidents. Cinnamon is believed to have medicinal properties. Be cautious about consuming the results of experiments. Edison 17:12, 9 January 2007 (UTC)
Teflon too was serendipitously discovered.  --LambiamTalk 22:39, 9 January 2007 (UTC)

Noodle soup edit

I once heard a counter-intuitive anecdote about making noodles for noodle soup. In Japan, a good noodle factory may add more water to the noodles if the noodles are for take-out use. Possibly, noodles with less water would absorb too much water and become soggy in less time. Is it true? If you have a pasta-making machine you may want to try it. -- Toytoy 04:35, 9 January 2007 (UTC)

If you're asking whether the practise makes sense, then I'd say it does to a certain extent. Uncooked pasta is usually covered in a fine layer of pasta "dust". When the noodles go in the water, some of the dust gets washed right off of course, but some will stick to the noodles. Nothing wrong with that - it helps sauce to stick to the noodles too - but if the noodles aren't being consumed right away, the now gooey dust will start to get everything soggy. Rinsing the noodles off can get rid of even more of the starchy dust, but will also usually chill the noodles, which will also make them sticky, defeating the purpose. Boiling the noodles in a larger quantity of water would be ideal because it will be able to absorb more of the pasta dust without adversely affecting the noodles. Matt Deres 16:49, 14 January 2007 (UTC)

Momentum edit

At school we frequently tackle momentum probllems using the general equation: mAuA + mbub = mava + mbvb.

To work out the result when two bodies collide using this equation, if they do not coalesce, you need to know something about the result of the collision, such as the velocity of one of the bodies. Knowing only the m and u values, is there a way of working out exactly what type of collision it is, and what speeds the two bodies will move off at?

Many thanks! --86.137.233.160 07:50, 9 January 2007 (UTC)

Generally not. The exception is for very small particles, such as atoms, where the collisions are elastic; as kinetic energy is conserved as well as momentum, using the formula for KE (1/2mv2) you can work out
1/2mAuA2 + 1/2mBuB2 = 1/2mAvA2 + 1/2mBvB2
simultaneously to find vA and vB. Unfortunately, this does not work for everyday macroscopic objects, because when they collide, they deform, which looses energy to the surroundings as heat. Laïka 08:05, 9 January 2007 (UTC)
...or you need to know the coefficient of restitution, which is the ratio of the difference in velocities after collision to the difference in velocities before collision. It is number between 0 and 1; it takes the value 1 for perfectly elastic collisions and 0 for perfectly inelastic collisions. Gandalf61 11:22, 9 January 2007 (UTC)
It works fairly well for objects that hardly deform under normal collisions, such as billiard balls when playing pool, or a penny "shot" at another penny. Here is an interesting, not well-known consequence of the two equations combined if the masses of the two objects are the same: ma = mb = m > 0. We can then divide out this common factor from the two equations:
ua + ub = va + vb;
1/2ua2 + 1/2ub2 = 1/2va2 + 1/2vb2.
Note that in the simplified momentum equation the velocities represent vectors. Squaring both sides (using inner product as the multiplication operation) gives us:
ua2 + 2uaub + ub2 = va2 + 2vavb + vb2.
Now take half of this and subtract the simplified energy equation, resulting in:
uaub = vavb.
So in an elastic collision between two objects of the same mass the inner product of their velocities is preserved. When one of the two objects was at rest, its initial velocity is zero, so uaub = 0, and therefore afterwards vavb = 0. So if a billiard ball hits another identical ball that is at rest, one of the two will be at rest afterwards, or they will take of in directions that form a right angle.  --LambiamTalk 11:52, 9 January 2007 (UTC)
Noone's mentioned it yet, but you should be aware that your "general equation" is merely a restatement of the Law of the Conservation of Momentum applied to interactions involving two "particles". -- 19:12, 12 January 2007 (UTC)

January 9 edit

How to find a reference...? edit

How do you find a reference for or locate the manufacture of a Hall effect switch or sensor like one designated as an F41.5Gc? -- Barringa 08:32, 9 January 2007 (UTC)

Try Honeywell SS41. -- 71.100.10.48 02:55, 12 January 2007 (UTC)

Standard taper ground glass joints (glassware): ISO vs. US edit

I'm trying to understand the difference between US and ISO standard taper (ST) ground glass joints. Are they more or less the same? Both using a xx/yy notation to describe taper diameter and length in mm? Would a ISO ST 29/42 and US ST 29/42 be identical? --Alf 08:02, 9 January 2007 (UTC)

It is possible to work with both at the same time. The US stuf is longer, but has the same diameter at the wider side. The people blive that this is better, but only in an ideal enviroment.--Stone 09:12, 9 January 2007 (UTC)
Thanks for the clarification. If you have the time, do you think you could incorporate some of your knowledge about the difference of these two standards into the Ground glass joint#Conically-tapered joints article? --Alf 09:44, 9 January 2007 (UTC)
Sorry this was for US-long the US is shorter than the ISO but still combinable, as long as the rest of the joint is not in conflict with some parts of the glas aperatus.--Stone 10:52, 9 January 2007 (UTC)


Importance of academic degrees edit

How important are academic degrees nowadays? Are Ivy league degrees worth the money? —The preceding unsigned comment was added by 132.231.54.1 (talk) 12:54, 9 January 2007 (UTC).

It depends on the field and location. For example, a degree from The Citadel doesn't mean much, unless you live near Charleston, South Carolina. If you do, it is an easy way to get a nice job at most of the local companies. I am a software engineer - which requires a degree to get any respect. I'm switching professions to become a college professor, which requires a graduate degree to even be considered. So, you can see that academic degrees are important in some areas and the college you go to is also important in some areas. --Kainaw (talk) 13:52, 9 January 2007 (UTC)
Having an undergrad from a very well known university can definitely get your foot in the door. Most employers respond very positively to "Yale", "Harvard", "Berkeley", etc. as your place of origin. Once you drop down a bit from a world-reknown place, I am not sure that the name means as much to most people. With more advanced degrees I think it depends on what your ambitions are. You are not going to get called up to Wall Street if your business degree is not from some place well-known, but if you weren't aiming at that then it probably matters less. If you are trying to break into academia, having a PhD from a very well known program will definitely make sure you are on the short-list of serious job candidates wherever you apply, which won't automatically get you a job but it sure does help. That's my perception of it, anyway. In most cases the degrees just act to impress upon others what sort of serious candidate you are — you've still got to perform to be successful. --24.147.86.187 15:28, 9 January 2007 (UTC)

If the whole idea of going to university or college for its own sake doesn't strike you as at least somewhat intriguing, you may want to consider your ultimate reasons for going in the first place. Money, a job, a house, a spouse, kids...? Vranak 16:25, 9 January 2007 (UTC)

I chose to go to university so I could meet new people, go somewhere different, spend some time away from home, and learn something I haven't studied at school. The slightly increased chance of getting a job is just an added benefit.Hidden secret 7 19:34, 9 January 2007 (UTC)

Are there some things you can not do if you do not have a degree? Going to university or college is like an operating system for your computer. Going to Havard will probably all a lot of bells and whistles even if you do not graduate like it did for Bill Gates who's parents socialized with the head of IBM, etc. Contacts might be equally or more important than a degree going by the number of millionaires with an 8th grade education or less. Instead of doing basic research in a university/government sponsored lab you could create such a lab of your own Jim Walters. Having a degree - even an incomplete one, is under any circumstance probably better than not having one at all. Barringa 21:25, 9 January 2007 (UTC)

If you are going to do any great work in your life, it will probably not hinge on your having a degree. You might get a degree in the course of doing great work, but it's not like you will say to yourself after four years of college "Ah, I have my degree! Now my life begins!". Vranak

Having a degree is like having a car... you can't walk or ride a bicycle on some roads or get to where you are going as fast. 71.100.10.48 03:00, 12 January 2007 (UTC)

Cosmic inflation edit

I'm attempting to write an essay on cosmic inflation, but am having a little trouble understanding the jargon in the article - I've interpretated the majority of it, but I don't see anything explaining why inflation ceased - did the decaying vacuum energy simply reach its new, more stable minimum?

Thanks, 130.209.6.40 13:40, 9 January 2007 (UTC)

Didn't it slow down because of gravity attracting everything towards everything else, and now they think the whole thing is going to collapse on itself.Hidden secret 7 19:31, 9 January 2007 (UTC)

Your guess is pretty much correct. The inflaton (no, I haven't missed an i there) field reached a lower, stable potential. What sort of level is this essay supposed to be? And are you versed in quantum field theory?

To be honest, I don't think I have to be that deep - it's for a group report on Cosmology, and the other topics are "easy" things like the expansion of the universe, and the CMBR, so I think it's more hand-wavey than anything else. I've got it pretty much done, with a summary of the process, some of the objections to the HBB model it explains, and now I'm just looking for criticisms. I've only been able to find two in my references, but the Cosmic inflation article doesn't really seem to have any. I've got that it predicts the wrong value of Ω0, and that the mechanism for why it stops isn't understood...but I'd really like more than that. Am I missing any crucial ones? I fear I'm in over my head... Icthyos 17:12, 10 January 2007 (UTC)

Other criticisms I can think of off the top of my head are: If inflation did happen why haven't we detected any cosmic strings? Why is there still a little bit of inflation going on today? Why is the energy of free space so much higher than that required by inflation? —The preceding unsigned comment was added by 86.145.254.48 (talk) 19:27, 11 January 2007 (UTC).

trivias about sounds... edit

Superscript text _could you please help me to find some of the most interesting trivias about sounds?,got a hard time seeking for some,;p 203.215.116.127 15:22, 9 January 2007 (UTC)heartbreaker203.215.116.127 15:22, 9 January 2007 (UTC)

The russians discovered that people can hear a much higher pitch of sound if whatever is producing the sound is touching the person hearing it.Hidden secret 7 19:28, 9 January 2007 (UTC)

The Mosquito is quite an interesting sonic device, and there's also ultrasound, used for medical tests. StuRat 04:08, 10 January 2007 (UTC)

Why does catnip have that kind of effect on cats? edit

Hi all. I was wondering why catnip has that kind of effect on cats? I was reading up in your article, but could find no mention of what the actual mechanism was. Maybe one of you biology-savvy individuals could explain it better? Much help appreciated ! Xhin 15:28, 9 January 2007 (UTC)

The mechanism actually isn't very well understood, though some aspects of it are. The cat's receptor for nepetalactone is in the vomeronasal organ, located above the feline palate, and the response is similar to a sexual response, but the exact nerve pathways involved are not, I think, known. - Nunh-huh 18:14, 9 January 2007 (UTC)
Following on from Nunh-huh, at the vomeronasal organ (VNO) the nepetalactone is hypothesized to bind to one or more G-protein coupled receptors on the surface of sensory neurons which are found in the sensory layer of the organ. Via a signal transduction pathway (probably involving a G-protein and a transient receptor potential channel) an influx of calcium ions occurs creates an action potential along the axon of the neuron. The sensory neurons of the VNO project to a region of the olfactory bulb called the accessory olfactory bulb where multiple neurons (each apparently expressing a single receptor type) synapse at special neuropil called glomeruli. Here the neurons synapse with mitral cells which, in turn, project to various brain loci, including the amygdala, where the signals are integrated into behavioural signals. It is at this point the pathway enters a bit of a black box. However, there is some evidence of projections to the hypothalamus, which in turn regulates a neuroendocrine response via the pituitary. These hormones would mediate the "sexual response" noted above. That is "how" it is thought to work, "why" is currently unknown. Though one might speculate that the chemical probably hijacks the pathway normally influenced by a cat pheromone. The fact that it only elicits such a response in a proportion of cats - and that it is such a dramatic response - suggests that a genetic element might be involved that is enriched for in domesticated breeds. Rockpocket 08:31, 10 January 2007 (UTC)
Mewow! Perhaps you could add that to the catnip article?--Shantavira 09:02, 10 January 2007 (UTC)

Evolution of organs edit

How do organs evolve when the intermediary step to the organ seem to bring no advantage to the animal? For example, take the elephant ear. I understand how useful their large ears are in present-day elephants, but I don't see how ancestral elephants with slightly bigger ears have advantage over others. For example, how can an ancestral elephant with an ear that's one milimeter larger in diameter be more successful than other elephants with a one milimeter smaller ear? I can understand how other organs, such as eyes, vertebrae ears, limbs, etc. would have advantageous intermediary forms, but not in organs that seemingly don't have such advantage. Is it genetic drift, or something else? Thanks in advance for any insight.128.163.224.198 19:02, 9 January 2007 (UTC)

An elephant's ears are an important cooling system for the animal. Consider larger ears==>better cooling==>better survivability. Ear size differences among different species correlates with climate. But also, how do you know the change was milimeter at a time? Perhaps the gene that regulates ear growth became virtually non-functional via a point mutation, leading to massive overgrowth? DMacks 19:39, 9 January 2007 (UTC)

Oh, to add more, a better example would be the evolution of bird wings, rather than elephant ears. How can a partially developed wing be useful to the animal?128.163.224.198 19:06, 9 January 2007 (UTC)

They now believe the appearance of bird wings was spontanious, caused by the spread of recessive alelles through a population of reptiles. I can't remeber what this is called now, but you might be able to find out more looking up evolution, or archaeopteryx.Hidden secret 7 19:26, 9 January 2007 (UTC)
One possible mechanism: Small wings are good for improving traction while running, larger wings are better, absurdly large wings give the ability to glide and later fly (birds).
Another mechanism: small skin flaps between limbs are good for longer jumps. Larger skin flaps improve on this (flying squirrel). Very large skin flaps give the ability to fly (bat). --Carnildo 19:52, 9 January 2007 (UTC)
Feathers#Evolution says that feature probably evolved for insulation, not for flight, and then got "hijacked" for an entirely different purpose. So they wouldn't have to suddenly be flight-worthy all at once. Clarityfiend 20:56, 9 January 2007 (UTC)

So the elephants genes were mangled and their ears suddenly went mad? Does the fossil record support this? I don't have a museum in my house, but the internet might know.Hidden secret 7 20:06, 9 January 2007 (UTC)

For example, how can an ancestral elephant with an ear that's one milimeter larger in diameter be more successful than other elephants with a one milimeter smaller ear?
That's easy to explain. Think about heatsinks for CPU in ordinary PC. The earliest CPU for PC have no heatsink. The problem is that those CPU cannot exceed a certain rate of heat release without failure. The first heatsinks are tiny (compare with today), but they give the those CPU evolutionary advantages over those without heatsinks because they can run "hotter". The next heatsinks are slightly bigger and so even more evolutionary advantages. Over the years, the heatsink got bigger and bigger. But they hit an evolutionary roadblock. So the next mutation are heatsinks with fans. The first heatsink fans are small and weak. Then they evolve bigger and faster. So you see, every small steps in evolution gives the animal advantages. Every millimeter counts!!! 202.168.50.40
NOTE: I'm the topic creator, just answering with a different computer/IP address here. Back to the question; I can understand how "every millimeter counts", but in the case of elephant ears I question its impact as a whole. There might be other environmental pressures that trump whatever advantage a milimeter brings (or not really "trump", but make them almost insignificant that the "big ear genes" don't get passed down that many more than small ear genes). For a trait to be passed down at more frequency than others, it must have absolute advantage over others. So, say that there are vicious predators of ancestral elephants. In this case, elephants with longer and stronger legs will survive better and increase its genes in the population. But, those with a milimeter larger ears will not pass more genes because the biggest pressure in the species will select those with better predator-evasion physique, not, say, temperature regulation that the large ears bring. And remember, the ear is only a milimeter larger; while it is an advantage to the animal, it doesn't have that big of an advantage for its genes to be passed down more frequently due to other environmental pressures.
Take another practical example, this time relating to us humans. We acknowledge that better vision makes a better organism. So even though we say that better vision is an advantage, how come we don't have everyone with "super eyes" compared to previous generations? (Our vision as a species probably has stayed more or less the same over the millenias. Pure speculation, but I don't see how it doesn't make sense). That's because, I think, there are other outside factors that selects other things, but not so much the vision. Early hominids with better communication skills, intelligence, and stronger legs had better chance to group together and evade predators; there are those with better vision, but the vision that they have don't offer too much of an advantage (if they don't have the three factors I listed in the previous sentence) that their genes become more frequent in future populations (again, because environmental pressures selected other things beside better vision).
So the only scenario I can think of that allows gradual enlargment of the elephant ear is if the environmental pressure demands larger ears and less everything else. (I guess I just answered my question there :-) ) Is what I'm saying a part of any specific evolutionary theory, so I can look it up? 128.163.80.164 22:23, 9 January 2007 (UTC)
"Environment" means more than just the physical environment, of course; it can also mean "mating pressures" and other things which come under the rubric of sexual selection. As to your question about the "less everything else," what you are really getting at, I think, is that the need for big ears needs to be evolutionary such an advantage that not having them becomes a disadvantage. There are rather complicated statistical models describing how strong these sorts of pressures need to be either way, if I recall. Additionally it is worth remembering that things need not evolve together at all—independent evolution of certain traits can come together in a favorable way as well, and some traits need not even evolve perfectly at all. --24.147.86.187 03:33, 10 January 2007 (UTC)
NOTE: I'm the topic creator, just answering with a different computer/IP address here. You hit my point right on the head there. So, would my analysis be correct? And is there any specific, named theory that describes what I talk about (by that I mean the evolution of a specific trait vs. relevant outside pressure)? From my limited knowledge of evolution, what I described seems to make the most sense to me in the evolution of organs, so if anyone can point out to me other sources regarding it, I'd appreciate it. 128.163.174.129 21:24, 10 January 2007 (UTC)
The questions you are asking are typically grouped into a school of thought known as Irreducible complexity. I think you will find the best answers to your questions in that article, as well as Evidence_of_evolution#Evidence from comparative anatomy, Evidence_of_evolution#Vestigial organis and especially Evolution of the eye. -- dpotter 21:19, 9 January 2007 (UTC)
NOTE: I'm the topic creator, just answering with a different computer/IP address here. Thanks, Dpotter. Just to note, however, I believe that evolution and common descent are the best scientific explanation of speciation. I'm not too fond of IC, ID, Creationism or anything like that in terms of providing the best scientific explanation of diversity. My question here is not so much to question evolution in general, but is about my curiosity in the specific mechanisms of evolution that may have occured.128.163.80.164 22:23, 9 January 2007 (UTC)
No one has yet mentioned the OBVIOUS ONE!:) Elephant ears of course would have started evolving for the same reason as human ears, for better hearing!! Which has very obvious advantages, hearing predators and such.. Vespine 21:34, 9 January 2007 (UTC)

Chemistry Trouble edit

Hey, I'm revising for Chemistry and have just done a past paper for my AQA Chemistry A-Level. I've just tried the AQA website, and it's not working! My friends have also found it down. I was hoping that you'd be able to confirm whether I'm write for this question.

You're given the formula for the decomposition of Lead(II)Nitrate

2Pb(NO3)2 → 2PbO + 4NO2 + O2

and then told to work out the number of moles using the Ideal Gas Equation, being told that the sample was heated until decomposition was complete. Using the information provided you get 0.00361 mol (to 3 s.f.) (500K, 100kPa, 1.50x10-4m3, T=8.31JK-1mol-1)

The followup question asks for the number of moles of NO2 and then the mass.

How would you tackle this followup. I'm pretty confident but want to check!

Thanks, --86.137.233.160 19:50, 9 January 2007 (UTC)


I think you are not giving us some info. You are looking for moles of what? You obtained .00361 mol of what? And what was the info provided? Maybe a more word-for-word description of the problem is in order. --Bennybp 21:55, 9 January 2007 (UTC)

unfitness test edit

This isn't about medical advice, I am just interested in knowing weather it is normal when my pulse rate gets over 220 that it feels like someone is squeezing my trachaea, my whole body is pulsating as blood flows through it and air is forced our of my lungs every time my heart beats. Is this beacuse I am very unfit, or just the usual side effects of a high pulse rate?Hidden secret 7 20:02, 9 January 2007 (UTC)

I would suspect that the latter is a symptom of the former. Vranak

Do you know how on all those fitness machines, they say "See doctor before commencing any fitness program!"? I think that is good advice. --Zeizmic 23:35, 9 January 2007 (UTC)

If your heart rate gets above 220 often, you're probably unfit. Regardless, try to exercise more than you do now, and see a doctor if you're worried. --Bowlhover 03:10, 10 January 2007 (UTC)

It has never gone above 180 before, and I have started exercizing more, which is why it was so high. I did the same thing again today and it seems slightly better. I hope.Hidden secret 7 13:46, 10 January 2007 (UTC)

Typically in exercise, your target heart rate should be 65-85% of your "maximal" heart rate. To calculate your maximal rate, take 220 minus your age. A heart rate over 200 in an adult is indicative of ventricular tachycardia, a potentially fatal heart rythm. You should probably talk to your doctor about your exercise regimin or about being tested with a Holter monitor. Andrewjuren(talk) 07:48, 11 January 2007 (UTC)

I don't think I have that, I hope not :( Is there any way to tell without seeing a doctor :? And I am not an adult yet :)Hidden secret 7 18:35, 11 January 2007 (UTC)

Galinstan? edit

Where can I find information on the liquid metal Galinstan? The wiki article did not have the physical properties I was looking for. (Viscosity, thermal conductivity, etc.) —The preceding unsigned comment was added by 216.182.144.119 (talk)

An MSDS is often a good source for physical properties. There's a link to one at the end of the galinstan article. DMacks 20:34, 9 January 2007 (UTC)

Two Hybrid Screening In Yeast edit

When talking about yeast cells used in two-hybrid screening, is it appropriate to call the yeast cell a 'host', 'model' or something else? --Username132 (talk) 21:06, 9 January 2007 (UTC)

Two-hybrid screening --JWSchmidt 04:26, 10 January 2007 (UTC)
Sorry but I don't think the article answers my question. As it happens I'm working on that article and that's why I asked the question (for a subheading). --Username132 (talk) 21:38, 10 January 2007 (UTC)
The plasmids are introduced into the yeast cells by a process that is generally called transfection. After the plasmids have entered the cells, the cells are called "tranfected cells". The tranfected cells are subjected to selection conditions: growth in a defined culture medium that only allows tranfected cells to survive if those cells can form a functional transcription complex formed by the physical interaction of two "transfected proteins". "Transfected proteins" meaning that they are proteins expressed from the plasmids that were transfected into the cells. --JWSchmidt 04:26, 11 January 2007 (UTC)

Elemental form edit

Is 3Co in its elemental form? 141.158.99.155 21:29, 9 January 2007 (UTC)

Do you mean Co3? A substance that's in its elemental form has only one element in its molecules; it's not a chemical compound. Co3 has 3 cobalt atoms, so the short answer to your question is yes. --Bowlhover 03:05, 10 January 2007 (UTC)
How would a molecule not be a chemical compound? A real question might be whether Co3 is a naturally-occurring allotrope of cobalt. DMacks 07:49, 11 January 2007 (UTC)

How's it done? edit

How was this done? http://ebaumsworld.com/2006/03/magnet.html Thanks!! Reywas92TalkSigs 21:52, 9 January 2007 (UTC)

That's an example of a superconductor. The black material is superconducting below a certain temperature. I assume that's liquid nitrogen poured onto it, this lowers the temperature and causes it to superconduct. When a material superconducts it does not admit any magnetic fields. The magnet will therefore not approach within a certain distance depending on its strength. This causes it to hover.
Contrary to popular belief this is not quite how bullet trains work. They mearly use very good conductors, not superconductors. I hope this helps. —The preceding unsigned comment was added by 81.153.52.250 (talk) 22:45, 9 January 2007 (UTC).
  • Clarification: The term "bullet train" is usually used for fast conventional trains. This person is talking about maglev trains. --Anon, Jan. 10, 07:42 (UTC).
How does the superconductor and the piece of metal stay a fixed different from eachother? —The preceding unsigned comment was added by 138.29.51.251 (talk) 23:09, 9 January 2007 (UTC).
Two answers: (1) The magnet settles downward until the upwards repulsive force from the superconductor becomes equal to the downward force of gravity, just like whenever a weight is placed on a spring, for example. If that were all that was going on, however, the magnet would probably slide off to the side as if it rolled off a ball. But it doesn't, so there is more going on: (2) You probably noted that the magnet appeared to become "stuck" at a particular point and orientation in space above the superconductor. This is because these are "type 2 superconductor|"type 2"]] superconductors, which do not exclude the magnetic flux completely, but instead allow a few thin tubes of magnetic flux ("flux vortices") to go through. These tubes tend to get stuck ("pinned") at defects inside the superconductor, and since the flux is connected to the magnet (in a sense) this makes the magnet "get stuck" too. --mglg(talk) 02:18, 10 January 2007 (UTC)
The article for more information is the Meissner effect. X [Mac Davis] (DESK|How's my driving?) 17:45, 10 January 2007 (UTC)

Reversible Gas Turbines edit

Kindly explain what is a reversible Gas Turbine —The preceding unsigned comment was added by 213.130.123.28 (talk) 10:33, 9 January 2007 (UTC).

Question moved here from the Mathematics reference desk.  --LambiamTalk 21:52, 9 January 2007 (UTC)
This extract from a research abstract considers one definition, where the turbine can turn in the opposite direction to normal -
Aircraft gas turbine engines, as now configured for ship propulsion, are unidirectional in output rotation and, therefore, require the added complexity of a reversing transmission or a reversible-pitch propeller. This study explores the feasibility of a novel reverse-turbine concept which is configured to adapt to existing free-power turbine engines without additional clutches or separate drive trains. This device, termed the 'isolated reverse turbine,' is sized for meeting that most demanding maneuver for a fixed-pitch propeller-driven frigate or destroyer, namely, the crash reversal maneuver. The reverse-turbine concept would replace the function of the reversing gear or the reversible-pitch propeller; it could also complement electrically actuated reverse transmissions by eliminating the need for braking resistors and switches.
A turbine operating in reverse would of course be a compressor, but the design characteristics of the two devices are somewhat different.81.153.220.80 12:56, 10 January 2007 (UTC)
Reversible implies that the turbine has an efficiency of 1; that is it loses no energy to frictional or viscous effects, and there is no entropy gain. Thus, the process (not the turbine) can be reversed without violating the second law of thermodynamics. It has nothing to do with running the turbine backwards. -anonymous6494 16:54, 10 January 2007 (UTC)
Based on Google hits on the term and the contexts in which the term appears, it doesn't seem that "reversible" is used in the sense of the word in thermodynamics. --71.175.23.226 18:08, 12 January 2007 (UTC)
I see hits for both, but for the sake of argument let's assume we are talking about running a turbine engine in reverse (i.e. as a compressor). The problem with such an engine is that the blades are designed with the direction of flow in mind, so simply running the turbine in reverse will not produce the desired result. An engine designed to do both could be built, of course at the expense of efficiency. It is likely that the reversible turbine would be less attractive (at such low efficiency) than another system, such as an electric motor and battery. anonymous6494 03:14, 13 January 2007 (UTC)
Google hits on the term suggest that reversible gas turbines are most commonly used for ship propulsion. At least in the context of this U.S. patent, a "reversible gas turbine" is one that can reverse the direction in which the output shaft rotates (however accomplished). It's not about running a turbine in reverse and using it as a compressor. --71.175.23.226 13:37, 13 January 2007 (UTC)

Measuring rainfall edit

Why is rain measured in mm than mL? The Updater 23:22, 9 January 2007 (UTC)

See Rain gauge. --Zeizmic 23:32, 9 January 2007 (UTC)
Kind of like how you wouldn't measure how much snow there is by its volume. I suppose one could measure rain and snow in terms of volume, but that wouldn't be practical. How high those things are from the ground lets us know, for example, how bad a flood or a blizzard is.128.163.80.164 00:33, 10 January 2007 (UTC)
More fundamentally, of course you could measure in litres say, and now you compare the amount of rainfall you had with your neighbour, who also collected rain, but you collected it in a can and your neighbour in a giant tub. Of course she would have more, if measured by volume. Your tiny can could not even hold one litre. So to make this comparable, you have to take the surface area into account over which the rain was collected. And then you get, for instance, as the measure an amount in litres per square metre, L/m2, and since 1 L = 0.001 m3, that unit of measurement equals 0.001 m3/m3 = 0.001 m = 1 mm.  --LambiamTalk 01:16, 10 January 2007 (UTC)
By the way, what Lambiam just did is called dimensional analysis, and it's a great way to figure why we use the units we use for several things. — Kieff 02:20, 10 January 2007 (UTC)

Like measuring pressure in mm too. :?Hidden secret 7 13:49, 10 January 2007 (UTC)

Pressure measured in terms of length is a reference to gravitational potential energy: height times the acceleration of gravity times the density of the fluid referenced (typically water or mercury) has units of pressure. So for mercury the conversion is (13.534 g/cm3)(9.81 m/s2)=133 Pa/mm. --Tardis 17:41, 10 January 2007 (UTC)
Most conventional pressure measurements are either in hPa (mbar) or in mmHg/inHg, for the reasons explained above. Titoxd(?!?) 17:44, 10 January 2007 (UTC)
Thanks for the info. The Updater

sideways in time edit

Thanks to such science fiction shows as Star Trek we have an understanding of the concept of back and forth in time. But what might "sideways" (left or right) or "diagonally" or "spirally" in time look like? Anyone care to speculate? —The preceding unsigned comment was added by 216.176.54.133 (talk) 23:40, 9 January 2007 (UTC).

"Sideways" doesn't exist in one dimension. If we consider time as some sort of timeline, it can only have "backward" and "forward" directions. — Kieff 23:50, 9 January 2007 (UTC)
Being a creature that occupies four dimensions, which I am aware of at least, I find it really difficult to imagine what one dimension would be like. ;) Have you seen the Back to the future movies?? I think they explain sideways in time fairly well, more the sequels then the original. Basically, time is like a tree, from any point in time, there are an almost infinite number of branches of possible 'futures' that could happen depending on chance and choice. going forward you can only travel down the branches that span from the trunk of the NOW you are in, travelling sideways, you could end up on a branch that branched off BEFORE the now you are in. For example, travel sideways in time to the 2007 where the allies didn't manage to develop nuclear weapons and the axis powers ultimately took over the world. Vespine 00:20, 10 January 2007 (UTC)
Imagining one dimension is easy, you can only go one way. Like time, in this universe, or driving through a tunnel. Imagining multidimensional time is much harder. Philc TECI 18:28, 10 January 2007 (UTC)
The way the OP asked the question, it seems he's interested in continuous movement in time, not "jumping". What you described is just jumping to a different timeline altogether. Assuming many timelines side by side, travelling completely "sideways" in time would just move you around the same instant in time, but in different alternative universes. Travelling diagonally, then, would make you pass through a different universe in each subsequent instant forwards (or backwards) in time. That wouldn't be very useful to observe. Unless, of course, alternative universes close to each other had a gradual change. If that was the case, you could see people walking in the streets slowly turning into intelligent chicken-like beings as you travelled diagonally along the alternative universes. — Kieff 00:35, 10 January 2007 (UTC)
Which raises the question, would it be unethical to catch these chicken-like beings, bring them back to our universe, and sell them (after some culinary processing) as Alternative Fried Chicken?  --LambiamTalk 01:21, 10 January 2007 (UTC)
From the realm of science fiction or modern physics, moving sideways in time would imply going to another of the nearly infinite number of alternate universes. At each instant, universes should branch out from each possible quantum event. Drop a piece of buttered bread, and it lands butter side up in one universe and butter side down in another. Drop 50 coins and there are universes where all 50 land heads up and every other permutation. Robert Heinlein wrote some fun sci-fi about such things. Edison 05:44, 10 January 2007 (UTC)
There is a classic science fiction story whose name escapes me now, about a Catholic priest who travels to another planet where he discovers a civilization of intelligent, peaceful dinosaurs which he comes to admire. But their entire existence begins to undermine his faith that only humans can have souls. User:Zoe|(talk) 18:16, 10 January 2007 (UTC)
Ah, it's A Case of Conscience, by James Blish. User:Zoe|(talk) 18:36, 10 January 2007 (UTC)
How about imaginary time? Also, Stephen Hawking wrote in his book Black Holes and Baby Universes about that I think. Something about time always moves forward, never backward, but we can go at an angle, as in the special theory of relativity. I suppose it just depends on what you put on the other axis, and if time is one dimensional or not. --Bennybp 06:17, 10 January 2007 (UTC)

This is only peripherally relevant, but watching the 2004 movie Sideways is an excellent way of spending time. JackofOz 02:34, 11 January 2007 (UTC)

January 10 edit

Joinery edit

If joinery is about joining bits of wood, what word describes joining metals? Brazing and welding and so forth. --Username132 (talk) 00:14, 10 January 2007 (UTC)

It's really welding as far as I know. — Kieff 02:16, 10 January 2007 (UTC)
I thought welding was just one technique, along with brazing and soldering. --Username132 (talk) 03:31, 10 January 2007 (UTC)
Perhaps you want metalworking. --Anonymous, January 10, 2007, 07:46 (UTC).
Or smithing.81.153.220.80 12:59, 10 January 2007 (UTC)
fabrication?82.38.197.184 20:04, 10 January 2007 (UTC)

Corrosive gallium edit

Is there a material with high thermal conductivity that does not react with gallium?

Gold, perhaps? -anonymous6494 16:58, 10 January 2007 (UTC)

Healing after a spinal injury/surgery edit

Hi, my dad had an accident (slip and fall) a few days ago and had to have surgery on his neck (c4 - c3 area). They inserted artificial bone into his neck. We were really scared he would be paralyzed, but he has shown a lot of improvement. He can move one hand and one leg with considerable strength and the other leg just a little but it sometimes spasms. His bad arm he can move but in the fingers he has mostly just thumb and index motion. This is at about 48 hours after the surgery.

We are hopefull that he will do a lot better, and with physio be able to walk. The problem is I really have no knowledge of how rehabilitation works outside hollywood movies (where people enter totally paralyzed and go onto win championships in football). My question is, how effective is rehabilitation at making movement better for a patient? Would it be common for someone who for example has feeling and some movement in both legs go onto to be able to walk?

I know this is a hard format to answer questions in, I am just looking for your general opinions.

Some people make remarkable recoveries, while others have only limited progress. Even for the doctors who treat the patients and know all the details it is hard to make a prognosis. The signs are hope-giving, but more than that can really not be said. You should really talk to the doctors on the case. All the best with your dad.  --LambiamTalk 01:30, 10 January 2007 (UTC)
After I broke my back in two places, I had two years of rehab. I gained full function (though I have places on my body where if I touch it I feel it somewhere else). In rehab, I noticed that the younger people recovered much faster than the older people. I was 10 at the time, so that is probably why I did well. I do not know if this is a universal rule or just an anomoly in the hospital I was in at the time. --Kainaw (talk) 06:06, 10 January 2007 (UTC)

Why do Acetone and other solvents dissolve polystyrene? edit

Someone please answer this for me: Why do Acetone and other solvents dissolve polystyrene? I've looked all over Net Library and other sources and have so far found no answer. Thank you! 68.191.137.57 01:24, 10 January 2007 (UTC)

Acetone is a solvent, maybe that article will help you find the answer you are after. Vespine 02:09, 10 January 2007 (UTC)
A chemical rule of thumb is that "like dissolves like", so it makes sense that an organic material like polystyrene should dissolve in organic solvents such as acetone, whereas an ionic material like table salt is better soluble in polar liquids like water. -- mglg(talk) 02:18, 10 January 2007 (UTC)

ELISA Reading edit

If I have to read an enzyme-linked immunosorbent assay microplate, is it ok to obtain optical densities with a spectrophotometer instead of a microplate reader? Thanks, Elizabeth 02:29, 10 January 2007 (UTC)

Yes in theory, provided that the absorbency you are looking for is within the range of the spectrophotometer. However, most spectrophotometers read from side to side on a cuvette, meaning that you often need at least 0.5mL to get an accurate reading (depending on the volume of the cuvette and the build of the spectro), whereas microplates usually don't hold that much fluid (since the microplate analyzer reads top to bottom through the plate). Remember, the precision of the experiment won't be as good if you use two different tools, so you should choose one metod for all the data in the experiment if possible, and certainly only use one machine for all data in a data set. 24.241.20.246 16:44, 10 January 2007 (UTC)
It should be noted that "Ultramicro" cuvettes are readily available from standard suppliers (reusable for ~US$300 ea), and can hold 0.05 - 0.1 mL (50-100 microliter) volumes, about the same as most microtiter plates. I'd also recommend brushing up on Beer's Law. -- 128.104.112.15 19:21, 12 January 2007 (UTC)

Corrupted page edit

The Thermal Conductivity page has been vandalized - various expletives see http://en.wikipedia.org/wiki/Thermal_conductivity —The preceding unsigned comment was added by Agsmyth (talkcontribs).

In the future, you can revert vandalism yourself by selecting an earlier revision and saving. Splintercellguy 07:18, 10 January 2007 (UTC)


Huntington's disease edit

I have read the article Huntington's disease. Some of the symptoms are Depression and Anxiety.

Another symptom is Chorea. Chorea is cused by overactivity of the Neurotransmitter Dopamine.

Dopamine is a neurotransmitter that is involved in depression.

The treatments for Chorea are antipshycotics, which are antagonists of dopamine.

--Jones2 23:59, 8 January 2007 (UTC)

You won't like my answer but, you'd have to ask a doctor;). I'm not one, but I do know that when the label says tell your doctor if you are on any of this other medication, they MEAN IT! Drugs can have surprising and NON intuitive side effects when taken with other drugs. brain chemistry is very complex, a normal person taking the medication of a schizophrenic may get completely different effects from the medication. For example, Antagonising OVERACTIVE dopamine to treat chorea may simply bring it to normal levels, not causing further depression at all. A doctor's job is to determine and weigh up all the positives and negatives of the treatments, there may be alternative treatments to chorea, partly why most people believe medical advice shouldn't be given out over the internet. Vespine 02:52, 9 January 2007 (UTC)
  • I don't have Huntington's disease. My question is of scientific interest. I don't understand. --Jones2 05:31, 9 January 2007 (UTC)
  • It's not the best of sources, but according to a 1990s-2000s fiction television show there is no cure for Huntington's. That means you need medication to treat the symptoms and suppress the progression of the disease. That would require at least 2 different meds. - Mgm|(talk) 11:24, 9 January 2007 (UTC)
I didn't assume you had Huntington's, but your question is still asking for something that could vary greatly depending on each individual case and could only be answered by a doctor who has evaluated the individual. If someone gave you an answer, even if it was well educated, someone else could come across this site and read the answer to be more authoritative then it could possibly be. Vespine 21:43, 9 January 2007 (UTC)
One further thing, the article states it is a genetic disorder so there is no cure, we can't currently cure genetic disorders. It also states There are treatments available to help control the chorea, although these may have the side effect of aggravating bradykinesia or dystonia. So the mechanism is known at least to some degree. On the chorea article it further states There is no standard course of treatment for chorea so I think you are barking up the wrong tree. A doctor would take all that into account, including if depression or any other conditions are a concern and would tailor treatments and controls suitable for the individual. A doctor should not give you medication for one affliction if the side effects outweigh the benefits..Vespine 21:50, 9 January 2007 (UTC)
Q1: There is no treatment for the underlying cause of HD, but there are good treatments for relieving symptoms. Q2: An antidepressant may or may not cause problems, there is no "scientific" answer, only individualised therapy. Without going into details, the principles would be: Initially choose medications that have been shown to help the specific symptom, and then watch the person for side-effects. Then the choice is then to either treat the side-effect, or try a different drug. There are numerous ways of treating the symptoms of depression, conventional anti-depressants not necessarily being the best, or giving better results. Whatever the doctor chooses, it is really just a matter of accurate follow-up and changing medicines appropriately. It is a matter of individualised poly-pharmacy, and not working straight from a textbook about normal people, and the doctor knowing what she is doing, and the patient (or family) learning to observe and report positive and negative effects, and it being a mutual responsibility (doctor, therapist, patient and family) to find ways of improving the situation - there is no "theoretically right treatment". Clear goals and honest communication is the only way to go. --Seejyb 01:26, 10 January 2007 (UTC)

  • I read the article Tardive dyskinesia. Tardive dyskinesia is characterized by repetitive, involuntary, purposeless movements. Tardive dyskinesia most commonly occurs in patients with psychiatric conditions who are treated with antipsychotic medications for many years.
  • I have read the article Chorea. Chorea is characterized by brief, irregular contractions that are not repetitive or rhythmic, but appear to flow from one muscle to the next, it also adds twisting and writhing movements. Chorea is caused by overactivity of the Neurotransmitter Dopamine.
  • Now Chorea is caused by excess Dopamine.
  • I am sure you can infer my above question now. --Jones2 06:31, 10 January 2007 (UTC)
I'm not an expert at all, and this is not a definative answer, but perhaps a pointer towards an explanation. Yesterday I had the pleasure of meeting Paul Greengard, a Nobel Prize winner for work on dopamine mediated signal transduction in the nervous system. He talked at some length about his work and, what I learned from him, was how a multitude of neuro-stimuli including antipsychotic drugs, alcohol, caffiene, LSD, PCP, cocaine, marijuana, amphetamines, ritalin and prozac - many of which produce contrasting physiological effects and act through different transductions pathways - all converge at a common protein called DARPP-32. It at first seems impossible to imagine how a single protein can mediate such different neurochemical effects. But - and I couldn't do the complexity or elegance of the system justice here - an incredibly sophisticated network of positive and negative feedback loops via kinases and phosphatases interact to give the protein a remarkable scope of action. Indeed DARPP-32 itself can be phosphorylated on up to seven different serine sites. Combinations of phosphorylation at least four of them are known to regulate its activity, and thus the proteins that DARPP-32 itself phosphorylates.
Now here is how this is related your your query: these sites are phosphorylated independently by downstream cascades of different dopamine receptor classes (D1-like or D2-like receptors). Moreover, other neurotransmitter signalling pathways have a huge, and differential, influence on DARPP-32 phosphorylaytion downstream of, and at the level of, both receptors. In other words, the same "dose" of dopamine at synapse can give very different responses via DARPP-32, depending on the receptor it signals through. And the local signaling environment for a given neuron - which can change from second to second - can effect both the type of dompamine receptor that is active and the effect of either receptor's signaling at DARPP-32.
So, while I don't have a specific answer for you, I believe Greengard's body of work - coupled with the consideration of signaling through different dopamine receptors - provides a mechanism that can explain why modulation of dopaminergic signaling can result in such seemingly paradoxical effects. Rockpocket 02:47, 12 January 2007 (UTC)
Though I thought the questioner was simply trolling in a rather unsophisticated way, I liked your answer, Rocketpocket, and envy the opportunity you had. It is wonderful to see how the basic scientists are working out why real life health problems, such as drug responses, are as tricky as are. --Seejyb 15:44, 12 January 2007 (UTC)
Thank you. I was very lucky. Greengard is a remarkable character (what he did with his Nobel Prize winnings speaks volumes about the man) and his work is incredibly inspiring. I heard today that I'm going to have the opportunity to have dinner with another Nobel winner, Sydney Brenner in a few weeks. So if anyone has any good RD questions on Caenorhabditis elegans or genomics I'll put them to him. I'm hopeful that if I meet enough of these people, some of their genius will rub off! ;) Rockpocket 19:02, 12 January 2007 (UTC)

force constant edit

A spring of force constant 'k'is cut into 3 equal parts.What is th force constant of each

Well, imagine that your big spring is made of three identical springs. The formula for three springs connected in parallel is:

 

Where kbig is the original k and ksmall is the new k. As all three ks are the same,

 

Therefore

 

It should be easy to solve from here. Laïka 14:31, 10 January 2007 (UTC)
Laïka, you mean connected in series, right? Springs add like capacitors, and "uncutting" a spring is certainly connecting them that way. --Tardis 16:31, 10 January 2007 (UTC)
Opps! Yes, sorry. Laïka 22:10, 10 January 2007 (UTC)

Circuit with two switches edit

In my kitchen there are two light switches. When one is up and the other is down, the lights turn on. When both are down or both are up, the lights turn off. Can someone show me with a simple diagram or a lucid explanation how this circuit works? I have only high school level Physics in me, so simpler is better. Thanks, J. Finkelstein 08:24, 10 January 2007 (UTC)

See light switch. Here is the circuit diagram. In your case, one of the switches is upside-down, so just imagine the central double wire having been twisted.--Shantavira 09:07, 10 January 2007 (UTC)

supply voltage    to light

Just for "completeness", I'll note that there was an older way of wiring these switches called the Carter system. It was used a lot when "wiring" meant actual, individual single-conductor wires in the walls (see knob and tube wiring), but it's no longer considered acceptable because, in some switch positions, it makes the "shell" of the light bulb base carry line voltage, potentially electrocuting a person attempting to change the light bulb. But you might well encounter this scheme in an older house.
Atlant 14:21, 10 January 2007 (UTC)

That seems a weird system. Does it have any advantages over a simple SPST switch? 80.169.64.22 16:46, 10 January 2007 (UTC)

Put one switch by each of two entrances to a room so one can turn the light on or off when entering or leaving by either door. DMacks 18:03, 10 January 2007 (UTC)
Just imagine you are using a pitch black stairwell. You need to switch the light on at the bottom of the stairwell. Now, once you reach the top, how would you switch off the light? 202.168.50.40 23:59, 10 January 2007 (UTC)
A linguistic curiosity here. The two switches in the diagram are formally "single-pole double-throw" (SPDT) switches, but in everyday hardware-store vocabulary they have two other names -- which are "two-way switch" and "three-way switch"! Apparently "two-way" is because they have two active positions, and "three-way" because they have three terminals. There is also a "four-way switch", which has four terminals that it connects in pairs, either straight-through or criss-crossed depending on the switch position. (Internally it's a DPDT switch with two pairs of terminals joined up.) One or more of these can be interposed along the two wires between the two three-way switches in the diagram above, to allow the same light to be controlled from additional points. --Anonymous, January 11, 2007, 04:00 (UTC).
--Of course I wrote this before reading the following question. Oh well. --Anon, 04:01 (UTC).

Rider question 8-) edit

Can you tell me the wiring diagram to be able to turn a light on or off from 3 or more locations?--Light current 21:46, 10 January 2007 (UTC)

You mean the one at the end of the light switch article? DMacks 21:51, 10 January 2007 (UTC)
Yes THanks--Light current 21:57, 10 January 2007 (UTC)
I'm guessing this is rhetorical, nevertheless, interesting. Google is useful too. David D. (Talk) 22:12, 10 January 2007 (UTC)

Will there be a mass die-off of my local gulls? edit

I've just heard that they're planning to close down my local landfill site this year. The area will be covered with earth and landscaped. I find myself wondering what will become of the hundreds (thousands?) of gulls that feed on the landfill on a daily basis. I'm not certain but I think that the gulls that feed on the rubbish there are a different group of gulls to the ones that feed in the urban areas. In the short term, it may be the case that the gulls from the landfill descend into town in search of food - a neighbour told me that when they closed the landfill in the place he used to live, the gull population in his town tripled for a time, with some of the gulls becoming very aggressive towards humans, presumably due to hunger (they'd attack anyone carrying anything edible as there simply wasn't enough discarded food in the streets to go around). So, what will happen to the gulls in the long term? Will they have the sense to move on eventually, or will the people of my town be subjected to the sight of sickly, starving gulls and corpses in the streets? I'd be lying if I said that I wasn't concerned about this... --Kurt Shaped Box 09:09, 10 January 2007 (UTC)

It could depend on how big the gulls are. The huge aquilline gulls we get in Thanet would probably eat smaller animals if they couldn't get enough food. And they could also try to eat out of bins &c, like foxes. Gulls die all the time, but there aren't usually lots liying around the streets here.Hidden secret 7 13:54, 10 January 2007 (UTC)
Gulls are pretty resourceful. Like most birds, their natural food supplies vary throughout the year so they will be happy to look elsewhere. In fact, they will probably find a healthier supply than scraps of plastic and rotting food, so the closure will probably be good for them in the long term. Whether they invade the town will depend what other resources are available to them.--Shantavira 14:01, 10 January 2007 (UTC)
Do you live near the coast? If so, I'd imagine that the gulls could still get more than enough by trailing the trawlers, but you'd see less of them, as they would spend more time at sea. This document states that the North Sea alone could support over 2,000,000 seabirds on fishing wastes alone. Given that this fascinating RSPB document about seabirds states that there are fewer than 2,000,000 birds in British waters, my guess is that there is more than enough fish waste out there to sustain a few thousand extra gulls. Laïka 14:08, 10 January 2007 (UTC)
(edit conflicted) Incidently, where I live, gulls have learned that the most profitable way to survive is to hang around schools around break and lunchtimes, and then swoop in to pick up all the dropped crisps and sandwiches as soon as the playground clears. It's always amusing to watch them (and also squirrels) perching on the rims of bins to try and retrieve leftovers, only to fall in themselves! Laïka 14:16, 10 January 2007 (UTC)
Yeah, they used to do that when I was at school too. They'd start hanging around on the roof of the kitchens when they started cooking the dinners in the mid-morning - and wait. All the waste food went into a big skip thing for pigswill and the gulls would climb inside to eat - some kids thought it was funny to try and knock the lid down when there were gulls inside. When the bin was next opened, they would attack the dinner ladies. --Kurt Shaped Box 21:30, 10 January 2007 (UTC)

I think your fears are justified. Unless a new source of food exists nearby, which can feed as many gulls, their future is bleak. The particular concern is that they only know how to get food that's lying on the ground, not having any experience in hunting. This type of dependency should not have been allowed to be created in the first place, perhaps ultrasonic speakers could have kept them away from the dump all those years ? StuRat 14:14, 10 January 2007 (UTC)

Apparently (I was told this when we visited the landfill on a school trip) they've been trying for years to find a way to keep the gulls away. AFAIK, ultrasound doesn't work on them, they ignore fireworks and flares after they've seen them a few times and they seem to understand the concept of 'shotgun' and 'rifle' once they've seen them in action (they apparently know the difference too - they fly high, out of the range of the pellets when the men have shotguns and stay low to the ground and fly away at top speed when they see rifles). On the other hand, I was told by someone else who used to work there that the local council don't actually mind the gulls being there too much because they eat quite a few rats and are good at locating the nests and eating the babies. --Kurt Shaped Box 17:43, 10 January 2007 (UTC)

Judging from this article their aggressiveness is probably nothing to do with their hunger and more to do with their proximity to humans. David D. (Talk) 21:40, 10 January 2007 (UTC)

They attacked people at my school too :( Lots of people I knew were late to lessons as seagulls wouldn't let them close enough to the nest to get a football &c back :)Hidden secret 7 19:05, 11 January 2007 (UTC)

Ah. That's your typical 'angry mother gull' response. Pound for pound, I'd say that a sitting hen gull is the most dangerous creature on the planet. :) --Kurt Shaped Box 23:26, 11 January 2007 (UTC)

geophysics edit

application of geophysical survey to the solution of a geofeatures eg salt dome.

Exploration geophysics is not a great article, but it does have some links. --Zeizmic 12:50, 10 January 2007 (UTC)

Frozen Corona edit

How does this work? Dismas|(talk) 13:40, 10 January 2007 (UTC)

The beer was first cooled to well below the freezing temp, without actually freezing, due to a lack of nucleation sites. This is called a supercooled liquid. In such a state, almost any disturbance can allow nucleation sites to be created, in this case air bubbles (note how it freezes from the top, where the air bubbles are). Other methods are to add small particles or release pressure. StuRat 13:59, 10 January 2007 (UTC)
It's also related to a topic we were discussing here about a week ago. In a closed, inelastic container (like a beer bottle), the freezing of water would imply expansion (and therefore, massive pressure increases). Carried far enough, it might rupture the bottle explosively, but up to that point, the rise in pressure just inhibits further freezing (thus providing another mechanism to allow the supercooling that StuRat mentioned). But uncap the bottle (and remove the excess pressure) and suddenly... - Atlant 14:26, 10 January 2007 (UTC)

work done by frictional force edit

Please give me a hint to solve this problem: a uniform chain of length L and mass M overhangs a horizontal table with two-thirds part on the table.The friction coefficient between the table and chain is 'u'.Find the work done by friction during the period the chain slips off the table.

Start with the definitional functions - what is work, and the equation for it? What is friction and the equation for it? Hipocrite - «Talk» 15:46, 10 January 2007 (UTC)
It presumably involves an integral over time or (more cleverly) over the length of the chain, because different parts of the chain will experience different amounts of force over different distances. (Using the length of the chain is my hint.) --Tardis 16:42, 10 January 2007 (UTC)

Images with eyes closed edit

When you press against your eyes while there closed, how come after 3-4 seconds you "see" colors and patterns? nicholassayshi216.253.128.27 16:40, 10 January 2007 (UTC)

See phosphene and closed-eye hallucinations. X [Mac Davis] (DESK|How's my driving?) 17:38, 10 January 2007 (UTC)

Hey I get that too. :( I can see stage 1 CEV, but to me they seem mostly pink and yellow. And if I close my eyes really tightly there is a very bright yellow patch that moves and pulsates randomly. I think this is stage 3 CEV. Once it was blue instead, and much smaller. :) I think I read somewhere that pressure on someones eyes appears as light, as the eyes can only detect things as images. Also could the first one be caused by the less sensitive cells of the retina? :?Hidden secret 7 19:00, 10 January 2007 (UTC)

Phosphenes from pressure come from mechanical stimulation of the retina—the eyeball is pressed against it. Pink and yellow! For me it is green and shades of black when I close my eyes. In the dark it is like television static, but I also see phosphenes (but not the hallucinations) in the center of vision appearing and disappearing in the same way as the static, like bacteria in an optimal environment reproducing. X [Mac Davis] (DESK|How's my driving?) 20:24, 10 January 2007 (UTC)

At high altitude / in space, you can get noticable sparkles from cosmic rays passing through your eye. Dragons flight 19:59, 10 January 2007 (UTC)

Sometimes when taking marijuana one encounters effects like this even though this is not written in the article. --helohe (talk) 00:16, 11 January 2007 (UTC)

Another one I forgot, sometimes I can see pages of writing floating past, but I can't see what any of the words are, possibly as they don't really exist :(Hidden secret 7 19:02, 11 January 2007 (UTC)

Ink edit

How do you separate two colours in ink--82.19.94.30 16:50, 10 January 2007 (UTC)

There are many ways, however I expect what you are looking for is Paper chromatography. --Neo 16:58, 10 January 2007 (UTC)

Blonde haired blue-eyed cavemen edit

As a person with brown eyes and hair, I am fed up with seeing cavemen always depicted with those characteristics, as though blue-eyed blondes were a more advanced form of life. Surely there must have been blue-eyed blonde cavemen especially as I understand blue eyes are determined by a regressive gene? Also, am I correct in thinking that when the mummy of Pharoah Rameses the Second was taken to Paris for 'repairs'the hair-dye experts from Loreal found that he had naturally red/ginger/auburn hair? Or is this just a tale? Thank you.

Depending on what you mean by cavemen, the answer may largely be no. Our article on blonde hair suggests it became prevalent only ~10,000 years ago. The near total absense of blonde hair and blue eyes outside of European populations suggests that the prevalance of both are likely to be relatively recent quirks of evolution. Dragons flight 19:57, 10 January 2007 (UTC)
Also that guy you see in the commercials... it's not makeup ...that's what he really looks like. Barringa 20:13, 10 January 2007 (UTC)
However the effect of presenting blonde-haired blue-eyed as the "advanced" group in film is interesting. See the Clan of the Cave Bear; neaderthals were all dark hair, the modern human (played by Daryl Hannah) was blonde and blue eyed. --TeaDrinker 21:51, 10 January 2007 (UTC)
In The Complete Valley of the Kings, New York, Thames & Hudson, 1997, it is mentioned that a study done in 1975 in Paris where Ramses II was taken, the mummy's hair was not just dyed red from henna but had originally been red. It would appear the white nationalist community likes this idea, as they have written about him, and other "white" Pharoahs here.
There is very little known about he evolution of blonde hair because the genetics of it are not understood, however red hair (and the strongly associated blue/green eyes, pale skin and freckles) has been studied. Variation in the gene MC1R accounts for a large proportion of redhair (in humans and pretty much every red coloured animal). A group of scientists analyzed the evolution of MC1R variants in human populations (primarly to determine how and why red-hair appeared in European populations) however, in their study they note:

Our estimates of 80,000 years for the red hair–associated Arg151Cys and Arg160Trp variants [of MC1R] likewise suggest a distant ancestral contribution from Paleolithic Eurasians to the western European populations of today.

In other words, the earliest modern humans in Europe had the genetic variants that we know cause red-hair, but certainly not fixed in the population at any great proportion, and since the alleles are recessive, it would have taken a long time until red hair began to appear with any frequency in Northern Europe. The same scientists estimate that it was another 40,000 - 60,000 years before red-heads became commonplace (sometime between 20,000 and 40,000 years ago). This would mean that late Cro-Magnon "cavemen" may have had red-heads among them, through probably not Neanderthals. Rockpocket 07:28, 11 January 2007 (UTC)

Interaction and placement of magnetic fields in a DC permanent magnet motor...? edit

In a 3 phase DC permanent magnet motor where magnet poles in the rotor are in alternating sequence per pole and the number of stator teeth are greater than the number of rotor poles does the magnetic field in the stator alternate sequentially from tooth to tooth and is the stator magnetic field designed to race ahead of the magnetic field in the rotor? 71.100.10.48 20:08, 10 January 2007 (UTC)

Is there such a thing as a 3 phase DC motor? I'm only aware of 3 phase AC motors, have you read electric motor?Vespine 09:46, 11 January 2007 (UTC)
Further, in any PERMANENT magnet motor, the stator magnetic field does nothing, it's permanent. Vespine 09:53, 11 January 2007 (UTC)

medical term for eye crust edit

What is the medical term for eye crust (aka: eye gunk, eye booger, sleep, schmutz...) Whatever it is, the common terms should redirect to the proper article name. --Kainaw (talk) 20:31, 10 January 2007 (UTC)

Oddly, all the Notes & Queriesesque things I've read say that doesn't have a medical name; I can't find any sources right now, but they seemed to agree that since neither it nor its absense is a sign of disease, the medical community ignored it. Incidently, if the eye does become infected, the discharge is known as Mucopurulent discharge, due to the charming combination of mucous and pus that it hence contains. Laïka 22:21, 10 January 2007 (UTC)
A similar question has been asked twice previously without any satisfying answers. Perhaps we should make up a name! BenC7 01:19, 11 January 2007 (UTC)
I believe its salt. But thats only my opinion. OTOH see Tears--Light current 01:42, 11 January 2007 (UTC)
Some people have already done this! See here. It's "Mucopurulent mattering in the punctum." Also, interestingly, Spanish has a perfectly good word for this: lagañas, which the Oxford Spanish Dictionary translates as "sleep" or, in a British familiarity, "sleepy-dust." zafiroblue05 | Talk 21:51, 11 January 2007 (UTC)

Parts of an automobile edit

In my technology class, we've learned about the engine block, the transmission, and the differential. Is that the order in which the energy flows, so to speak? Thanks, anon.

I would tend to replace 'engine block' with 'crankshaft' in the above list, but otherwise that is correct. (The engine block is arguably just a stationary lump in which the pistons move. The pistons are tied to the crankshaft, which converts the reciprocating (up-and-down) motion of the pistons into smooth rotation.) TenOfAllTrades(talk) 21:24, 10 January 2007 (UTC)
And you can add wheels then road to the end, and foot -> accelerator pedal at the start. Vespine 21:33, 10 January 2007 (UTC)
Well, no, the power comes from the fuel burned, not from the foot of the driver. The gas pedal is just a control. Friday (talk) 22:13, 10 January 2007 (UTC)
Sorry, should have added a ';)'. Vespine 23:11, 10 January 2007 (UTC)
Unless you're Fred Flintstone. anonymous6494 06:45, 12 January 2007 (UTC)

Furniture materials and health effects edit

With out getting into too much chemestry, how can I tell if a piece of furniture has a material or finish that is off-gassing or creating other potentially harmful effects in my home? 67.94.101.67 21:55, 10 January 2007 (UTC)

Good question... you could try smelling it. Benzene and those sorts of chemicals will not smell very nice. Vranak

Water Displacement in Ferries edit

Could you please explain in language understandable for a 10 year old, why a ferry does not sink when cars are loaded onto it and why the cars must be balanced when they load? Thank you so much for taking the time to answer this. Linda Battler216.183.133.181 23:26, 10 January 2007 (UTC)

Does it make sense to you why boats float at all? See the "Why boats float" section of our boat article if it's not clear. Now, even with all the extra weight of the cars, the whole thing is still lighter than the water it displaces, so it still floats. One could make a boat of any weight simply by making it very large...a ferry is just a boat that is large enough to float with the cars. As for even loading, if there's more weight on one side than another, the boat will tilt that way. Tilt too far and the boat could capsize. DMacks 00:03, 11 January 2007 (UTC)
There have to be enough air-filled compartments low in the ship to keep the ship riding high enough in the water that when the cargo doors are opened, they are above the waterline. Ferries aren't unique in that if enough water gets inside (as has happened when cargo doors were either: left open, not completely closed or opened en route and either rough water or a rogue wave allowed ingress) the vessel will capsize. Ship designers and builders carefully calculate the maximum acceptable load to keep the cargo doors a safe distance above the waterline, that's fairly basic load tolerance mathematics. When empty, the ferries ride high in the water (like other cargo vessels), and they are filled to a carefully-calculated capacity to ensure that (under normal conditions) the ferry doesn't risk being swamped. Anchoress 01:38, 11 January 2007 (UTC)
You can even make ships and boats from concrete - see concrete ship and concrete canoe. Gandalf61 13:05, 11 January 2007 (UTC)
The physics of exactly when a boat will tip over are complicated (because of the effects of the hull shape), but it's related to the idea of center of mass. --Robert Merkel 02:12, 12 January 2007 (UTC)

January 11 edit

Sweet potatoes don't grow on trees edit

Or at least that is what I still believe, despite my friend in Israel just telling me via IM that they do. Is there a food available in Israel that might be called a sweet potato that does grow on trees? Or is someone just pulling her leg? Anchoress 01:29, 11 January 2007 (UTC)

First, make sure it really is a sweet potato she is talking about. In the US sweet potato is synonymous with yam. But there is another true yam that is unrelated to sweet potato. It also has edible tubers, however, one, the vine Dioscorea bulbifera, also has an above ground tuber (bulbil) known as an air potato. It too can be eaten so it is possible that this is what your friend is describing. David D. (Talk) 04:44, 11 January 2007 (UTC)
Good information! I'm assuming, since she grew up in Canada, that when she says 'sweet potato', she's referring to what we both were familiar with from our years as friends and co-chefs. ;-) I think the above-ground tuber you mentioned is probably the source of confusion, and I suspect it is the person who told my friend about it who is confused, not my friend. Thanks for the info! Anchoress 21:00, 11 January 2007 (UTC)
Or the person told your friend they were yams, (no confusion there), your friend then told you sweet potato (translating to common usage) not realising there is a different yam? The thing that makes me doubt that Dioscorea is the source of this story is the tree part. these are definitely vines and could not be confused with trees. I suppose the vines might grow in trees. David D. (Talk) 06:23, 12 January 2007 (UTC)
Possible, but unlikely. My friend is definitely familiar with the difference between sweet potatoes and yams, but the person who told her might not be. There's also a whole lot of hydroponically grown food in Israel; maybe that's part of the confusion. ;-)) Anchoress 06:41, 12 January 2007 (UTC)
OMFG American's call sweet potatoes/kumara yam? I always assumed whenever I hear people talking about yams in Americans books and TVs they were really referring to yams, not sweet potatoes. Silly Americans. You learn something new every day I guess... Speaking of which, I just realised the yams I purchased once or twice before to try and make a yam ring ([8]) weren't real yams either but ocas which we silly kiwis call yams evidently (I've lived here for 5 years and didn't know that!). No wonder it didn't work. Nil Einne 14:44, 13 January 2007 (UTC)
The most fascinating thing about yams is how they revolutionised medicine. The secondary metabolite of diosgenin caused the price of medicinal corticosteroids to drop massively. [9] David D. (Talk) 21:43, 13 January 2007 (UTC)

Red Blood Cells edit

Why is that red cells in humans do not show a nucleus

Because they don't have nuclei. :-) See red blood cell. Basically, an RBC is a packet of hemoglobin. --David Iberri (talk) 01:45, 11 January 2007 (UTC)

They are prokariots, like bacteria, and the lack of a nucleus means they only live for a few months :(Hidden secret 7 18:57, 11 January 2007 (UTC)

They are not prokaryotes they are just very specialised. David D. (Talk) 19:19, 11 January 2007 (UTC)

A similar question came up before so unless the article has significantly changed, you should find BTW it mentions that RBCs in some animals do have nuclei Nil Einne 14:33, 13 January 2007 (UTC)

Chemistry - What is a plasma edit

What is a plasma

Plasma is an ionised gas or substance; it's also a blood product excluding red blood cells. Read up on different types of plasma and let us know if you have any specific questions. Anchoress 02:21, 11 January 2007 (UTC)
Plasma is the 4th state of substances and is reached when someting reaches a few hundred degrees C
Well, not necessarily fourth, else you'd need to number other states like Bose-Einstein condensates, quark matter, etc. But that's just a point of pedantry. GeeJo (t)(c) • 22:40, 11 January 2007 (UTC)
It is not accurate to cite a temperature requirement. A plasma is simply a gas which is ionized to the point where interactions with electromagnetic fields are significant. Much interstellar gas is a plasma, but it is extremely low temperature - around 4 degrees kelvin. --bmk

What is this psychological phenomenon called ("illusion" seems too general) edit

I'm curious as to what the name of this is -- for example, when you spot a distant rock in the forest, and you are convinced it's a resting bear or a stationary wolf. Or, at night, you see some white pipes at a construction site and they appear as a small group of people in white t-shirts. Basically, when your mind/imagination makes up decisive assumptions about a vague, uncertain shape with little information (but the assumption usually keeping in line with the context of the surrounding area and its theme). I think this generally fits into "illusion", but is there a more specific, narrower name for this? My original guess was mirage, but reading the article on that, a mirage is something very specifically different. --67.161.84.158 02:27, 11 January 2007 (UTC)

It's a misperception, more specifically a pareidolia. - Nunh-huh 02:33, 11 January 2007 (UTC)
Reminds me of Gestalt! I think pareidolia comes from our instincts for face recognition and anthropomorphization to try and understand [and copy] other humans to learn. X [Mac Davis] (DESK|How's my driving?) 05:26, 11 January 2007 (UTC)
It's definitely a useful paranoia to have; if you're alone in the dark, it's better for your brain to err on the side of caution, and assume that you're surrounded by wolves and bears. Vranak

This sounds almost similar to what I see :) Actually I don't see it, but that is still the same :) Could it be because we want to see something, or caused by something we think we should be seeing :? What about seeing clouds as shapes, is that paraidolia too :?Hidden secret 7 18:55, 11 January 2007 (UTC)

This was once termed "perceptual set" as at [10]. Your expectation condition how you perceive a sensory stimulus. If a policeman thinks someone he has stopped is a fleeing felon, he is more likely to perceive the celphone in her hand as a gun and shoot her. Edison 20:56, 11 January 2007 (UTC)

Volume of Atmosphere edit

How would one calculate the volume of Earth's atmosphere? Maybe this page will help, but I don't know how. Thanks.--68.205.179.107 03:45, 11 January 2007 (UTC)

Since the atmosphere doesn't have a definite upper limit, it doesn't have a definite volume either. If you want to compute the volume below a certain altitude above ground level, just multiply that altitude by the surface area of the Earth. --Anon, January 11, 04:05 (UTC).
I don't think you can simply multiply the surface area by the height. To do that, you would have to integrate the surface area equation with respect to radius of a sphere from the surface of the earth to the end of the atmosphere (anyone want to back me up on that?). I think to properly do it, assume the earth and its atmosphere are spheres and subtract the volume of the earth from the volume of the earth + atmosphere. Treat them as concentric spheres. :-) --Bennybp 04:09, 11 January 2007 (UTC)
If R stands for the radius, V the volume and A the area of a sphere, for a small increase of R by an amount of ΔR you can approximate ΔV = (ΔV/ΔR)·ΔR by (dV/dR)·ΔR = AΔR. The relative error in doing so is ΔR/R. For the Earth R ≈ 6400 km. Even if the thickness of the atmosphere is arbitrarily set at 100 km, that is less than 2% – rather good for something that depends on an arbitrary choice to start with.  --LambiamTalk 04:35, 11 January 2007 (UTC)
Still, why not just calculate the difference between the volumes of the spheres with the atmosphere radius and the surface radius? — Kieff 04:56, 11 January 2007 (UTC)
Good point. Although if I had to do it I'd still do the difference of two volumes method. Plus, the volume of the earth may be on the Earth page, although it doesn't say if that's the volume of everything or just the (solid) earth. --Bennybp 05:02, 11 January 2007 (UTC)
The volume of the two is essentially the same - the Earth's atmosphere has very negligible height (however you reckon it) with the depth of the earth ... so surface area times height probably induces very little error compared to the error of where you apply the cutoff. The atmosphere probably "ends" around 100 Km, or ~1% of the total distance from the centre of the earth. WilyD 14:19, 11 January 2007 (UTC)
I would say that attempting to calculate the volume of the atmosphere should probably be done by first determining the mass of the atmosphere (i.e. taking into account that the atmosphere tapers off) and then determine how much volume that equates to under standard temperature and pressure. That said, you would also have to remember that the atmosphere is composed of different gases at different altitudes. Andrewjuren(talk) 07:29, 11 January 2007 (UTC)

Names of the toes edit

The above question about medical terms reminded me that I once heard on the radio that there were no medical names for the different toes, so a doctor had invented some Latin (or they might have been Greek) names based on the nursery rhyme. For example, the big toe was called whatever is Latin for "market piglet" ("this little piggy went to market"). I would love to know the complete list of names he invented but have had no success in Googling this. Perhaps they are lost for ever? (If nobody knows the answer, I'll take this to the language desk and see if somebody there would like to recreate them.)--Shantavira 09:42, 11 January 2007 (UTC)

I Googled on "toe" "market", "piglet", and "Latin", and found the answer here on the second hit. (For fun I also looked in Gray's Anatomy, 1958 edition, to see about the statement that there are no medical names for the toes. Indeed it simply numbers them from "first toe" through "fifth toe", also using "big toe", "great toe", and "little toe" as synonyms for the first and fifth. It doesn't even use the word "hallux" that that web page mentions, at least not on the pages I looked at.) --Anonymous, January 11, 11:11 plus 8 minutes (UTC).
Oh well done! Now my challenge is to wheedle pocellus fori etc into the appropriate articles! ;-) --Shantavira 13:32, 11 January 2007 (UTC)

free engineering handbook downloads edit

On which web-sites can i download free engineer's handbooks legally?

Tatenda

  • If they usually cost money, any way to get them for free is illegal. You might try Wikibooks, but you'd have to be more specific about which field in engineering you're referring to. - Mgm|(talk) 12:14, 11 January 2007 (UTC)
    So are there any that "usually" don't cost money and can be downloaded?  --LambiamTalk 19:55, 11 January 2007 (UTC)
Your best bet would be to search for U.S. government publications, which are usually free. A search like this in Google will pick up only US government sites, and seems to have gotten a few hits, though they are fairly specific (Naval Engineering, Hydrologic Engineering, etc.). --24.147.86.187 15:30, 12 January 2007 (UTC)
Some universities now publish their course books online. These aren't exactly handbooks. There are also a number of textbooks available from pubmed. Dunno if they cover engineering. They aren't usually downloadable tho Nil Einne 14:30, 13 January 2007 (UTC)

Minimal diet edit

I have been asking myself for quite a long time what ingredients should have a human diet a person could follow without future health problems by the premise of beign as less diverse as possible. Is possible to survive only eating one kind of food? Maybe with two combined? I am not going to follow the task but theoretically I think the question es interesting and not easy to answer. Thank you for your valuable help!

  • Survive for how long and with what quality of life? 100% probably doesn't allow a very restricted diet, but my Grandfather only eats about 12 food items and is not "noticably sick" --- but I doubt he's 100%. And where food comes from can make a difference too ... people in China are often Selenium deficient while North Americans may be exceeding their recommended daily allowance just from the differences in it's concentration in soil. WilyD 14:45, 11 January 2007 (UTC)
  • I guess something like "normal health". For example, someone who is almost living in Mc Donald's can not have a healthy life and perhaps he will have cancer or heart disease in the long term, but he is not going to have a serious short-term vitamin deficit disease like Beriberi or Scurvy.I think a man on their fourties beign able to get confortably into seventy years old.

I will say this: if you go on a very limited diet, you will eventually begin to crave foods that contain nutrients that you've been missing out on. So if you eat nothing but grains and fruit, one day you might wake up with an incredible appetite for calamari. Our bodies are smart that way. Vranak

I usualy eat almost nothing except noodles ommelette and pasta but I am still reasonably healthy :) I suppose it also depends on what the food is made of, some food may have a good mixture of nutrientsa &c, whilst other food is almost entirely one thing :)Hidden secret 7 18:50, 11 January 2007 (UTC)

You say you're still reasonably healthy, but how long will you stay reasonably healthy with such an unhealthy diet? That is the relevant question here. How long have you been following this diet? My guess is, not very long, at most a couple of years.  --LambiamTalk 20:03, 11 January 2007 (UTC)
Also, what do you mean 'resonably healthy'. Have you had a recent medical checkup with a blood work up? If not, I question whether your really resonably healthly. IMHO, appearing healthy doesn't mean your healthy. If e.g. your cholesterol is way out of wack are you really healhty even if you don't notice any ill effects? You may say yes, but some people with cancer also seem healthy and then are dead in 2 months. I myself very rarely get sick, and feel quite healthy (except perhaps when I try to exercise/exert myself too much). But I definitely have concerns about my health given that I know I have a poor diet, eat too much, and don't exercise much. Nil Einne 15:32, 13 January 2007 (UTC)

All I know is, if they ever sell a tasteless gray paste with all nutrients I'd ever need for a good living, I'd live off the stuff. — Kieff 20:23, 11 January 2007 (UTC)

Wasn't there a urban myth that you could have a healthy diet purely on Guiness and bananas? But the caveat was that you'd have to dring 15 pints of guiness a day or something.Vespine 21:48, 11 January 2007 (UTC)

I bet you could live off human flesh pretty well, assuming that the person you were eating was in good health -- apart from being dead I mean (see Soylent Green). Seriously though, there are several lab animal "diets" that might work (monkey chow). It would probably need to be slightly modified for people, but I'm sure that will happen someday (the opening sequence of Futurama advertises bachelor chow). --Cody.Pope 04:40, 12 January 2007 (UTC)

physical exercise and aching after edit

Hi. I went to a circuit training class on tuesday (for the first time in 7 years, so I expected to ache after). why though do my muscles ache more now on thursday than I did yesterday? Why does it get worse over time (and then obviously gets better after a few more days)? thanks, Spiggy 16:04, 11 January 2007 (UTC)


found it! thanks anyway guys. here's the url http://en.wikipedia.org/wiki/Delayed_onset_muscle_soreness

spiggy 17:01, 11 January 2007 (UTC)

Heh, I stumbled onto that article the morning after the I got my Wii. Cyraan 19:37, 13 January 2007 (UTC)

Aloneaphobia? edit

Ok, I searched List of phobias, Loneliness, and Solitude and haven't been able to find it. I haven't tried google yet as I don't know what I'll find searching google.

Is there a name for an unreasonable fear of being alone? Any one term I can search for to find information? ~ ONUnicorn(Talk|Contribs)problem solving 16:22, 11 January 2007 (UTC)

Fear of being alone is called autophobia - see here and here. Gandalf61 16:43, 11 January 2007 (UTC)
The first link's mildly helpful; the second is exactly the sort of thing I did not want to find by searching Google. CTRN (change thats right now) claims to be able to cure fear of anything, and if there's not a word for it they make it up (usually several of them). Thank you though. ~ ONUnicorn(Talk|Contribs)problem solving 17:08, 11 January 2007 (UTC)
Links were more or less picked at random just to show I wasn't making the term up. At least you maybe have a starting point for your research now. Gandalf61 17:33, 11 January 2007 (UTC)

Ernst Ruska edit

I can not find out any information about: Was Ernst Ruska ever married and did he have any kids? I am doing a project on him and need to know this information.


Thanks70.17.236.192 17:55, 11 January 2007 (UTC)

If you visit our article Ernst Ruska, you will see a link at the bottom, "Homepage of the Ruska family". If you follow that link, you can find the answers to your questions – although I had to apply some combination and deduction to the information given before I figured out his wife was called Irmela.  --LambiamTalk 19:20, 11 January 2007 (UTC)

Peanuts!!! edit

Is there any benefits or dangers to eating peanut shells?216.253.128.27 18:51, 11 January 2007 (UTC)nicholassayshi

On the positive side it is low calorie and high fiber. On the negative side it is non-nutritious, unappetizing, and an extra risk of ingesting aflatoxin.  --LambiamTalk 19:28, 11 January 2007 (UTC)
I used to hate peanut shells and deshell peanuts. Nowadays I prefer peanuts with them. So I wouldn't say they're unappetizing Nil Einne 14:27, 13 January 2007 (UTC)

Questions regarding a Rainbow edit

I have been looking at rainbows since I was a kid. I was also able to creat rainbows using our garden hose. I questioned why people didn't know that a rainbow was a full circle. I also have wondered when reading information regarding rainbows if the reason that we see different shapes of a rainbows caused by the rainbow being perpendicular to the sun?

I have learned from reading and I try to get more interested in the facts about rainbows. That is why I give friends and family members questions I create to get them to wonder about things more themselves.

  1. What is the shape of a rainbow? —A circle
  2. What is the white ring that can sometimes be seen around the moon during the winter? —A Lunar Rainbow
  3. When light from the Sun enters a raindrop what happens? —It is both Reflected and Refracted and is broken down into its primary colors
  4. What are the primary colors of a light? —Red, Green and Blue (True Blue)
  5. The reflected and refracted light from one raindrop creates how many colors? —Only one
  6. How many raindrops does it take to create a rainbow? —Millions
  7. The center of a rainbow is in-line with the center of the sun, which is in line with what point of your body? —Between your two eyes
  8. A rainbow can sometimes look like the shape of an ellipse but it is actually perpendicular to what? —The Sun
  9. A rainbow can only be seen when the Sun is from the horizon to what degrees? —42 degrees
  10. The colors of a secondary rainbow is what? —Inverted
  11. When light comes out of a raindrop is comes out at what angle? —42 degrees
  12. When up in an airplane with clouds down below you sometimes can see a total rainbow. What is in the middle? —You are
  13. Forms of a rainbow can be seen: a. In the morning dew; b. When the sky is almost black; c. High in the cirrus clouds; d. All of the above; e. None of the above? —d: All of the above
  14. What three colors are the primary colors of the Positive Spectrum? —Red, Green and Blue (True Blue)
  15. What three colors are the primary colors of the Negative Spectrum? —Red, Yellow and Blue
  16. When Green light and Blue light is combined what color is created? —Cyan (Sky blue)
  17. What causes us not to see a full rainbow? —The Earth
  18. Light is both reflected and refracted by each raindrop. How many angles can the light be seen? —Only one
  19. When you and a friend are looking at a rainbow, can your friend see the same rainbow as you? —No
  20. When light enters the lower part of a raindrop a secondary rainbow is created. At what angle? —52 degrees
  21. When Red light, Green light and Blue light are combined what color light does it produce? —White Light

—The preceding unsigned comment was added by 76.174.186.108 (talk)

Neat stuff, rainbows are! Your #14-16 aren't self-consistent on their face though...maybe you've oversimplified a bit? How can the same color be part of both the positive spectrum and negative spectrum ("if you take something away, it's not still there")? See also subtractive color. Also, #5 is wrong (or at least poorly worded along with #11)...each raindrop acts as a prism and creates the full spectrum over a (perhaps narrow) range of angles. Perhaps what you mean is that an observer is only situated to see a single color from each raindrop? DMacks 19:56, 11 January 2007 (UTC)

Just adding a few comments:

4) Not quite. Red, green and blue are only the primaries of our color perception. See also: visible spectrum
5) No. Each raindrop refracts light entirely. The reason the transition between colors is gradual is because of the angle we're seeing the rain drops at.
15) If you mean the primary colors for a subtractive color space, they're actually cyan, magenta and yellow. See CMYK.
16) Cyan is not really the color of the sky.
18) Several. The incident light is refracted to several different angles.
19) Well, that'd depend of how you define "rainbow" as an entity — Kieff 20:21, 11 January 2007 (UTC)
(after edit conflict) Have you read our article Rainbow? I hope you enjoyed it. In the lsit you posted there were two questions numbered 18. I've taken the freedom to renumber them, using the inbuilt numbering of the wiki markup, so that all now have different numbers.
If you allow me to nitpick a bit, I have a few remarks.
Re 3. Rather than breaking white light down into primary colours, the refraction breaks it down into spectral colours, which form a continuum; see also Prism.
Re 4 (and also 14 & 15). See our article Primary colour.
Re 5. It is true that the rays of light reaching your eye from any single raindrop are (almost) the same colour. But rays of all spectral colours emerge from each raindrop, and while your eye may be hit by blue rays from some raindrop, the eye of your friend, who is standing next to you, may be hit by red rays from the same drop of rain.
Re 7. Actually each eye receives the image of a slightly different rainbow. For each eye, the centre of that rainbow lies on a straight line coming from the sun and passing through the eye. Since these lines are parallel for all practical purposes, the images are seen as coming from the same direction, which our brain interprets as coming from very far (recall that parallel lines "meet" at infinity). That is why a rainbow looks as if it is far away while it is actually quite close -- or, more precisely, the drops contributing to the illusion are usually not far away.
Re 8. The way this is formulated is a bit puzzling. The image always forms (part of) a neat circle. An ellipse, looked at under an angle, may also appear as a circle. If our brain is somehow misled into thinking the image is on a plane that is not perpendicular to the straight line from the Sun through your head, it may interpret the image as being that of an ellipse.
Re 12. You can also sometimes see this when you are on the top of a mountain. I've seen it. More precisely, it is the shadow of your head that is in the middle – except when looking while enclosed by a plane you only see the shadow of the plane at the spot where your head's shadow would have been.
Re 18. This question is not very clear.
Anyways, I agree that rainbows are fascinating stuff, and a good way to pick up some science. Cheerio.  --LambiamTalk 21:07, 11 January 2007 (UTC)

The optic nerve edit

last year, when I was doing an internship at a neuroscience institute, a question popped into my mind. Namely, why do the optic nerves (partially) cross over eachother and why is the central visual processing unit of the brain, the ocipital lobe, to the back of the brain? Wouldn't it be smarter to have the ocipital lobe to he front of the brain so the incoming signals can be processed more rapidly? I asked one of the researchers I worked with and he just told me I should know the answer to that by now. Sadly I had no previous history in neurosci let alone any knowledge of neuro anatomy or evolutionairy biology. So can any of you guys give me a clue? PvT 20:58, 11 January 2007 (UTC)

In the same vein, why do the nerves coming from the retina pass in front of it, so that they need to go through it somewhere to pass to the other side, causing a blind spot. The design of the eye of an octopus is very similar but doesn't have this blooper. I think such bloopers (problems with teeth and child birth come to mind as other examples) can best be explained by the theory of Dumb Design: our body plans are not the result of evolution, which would have optimized this away, but the result of a homework assignment in bio design, unfortunately executed by not-the-brightest kid in the class (if you allow me to call a juvenile extraterrestrial being a "kid").  --LambiamTalk 21:24, 11 January 2007 (UTC)
I think the standard term is "unintelligent design" actually. For some intertesting reading about the design of the human eye, try Evolution of the Eye. DMacks 22:28, 11 January 2007 (UTC)
A better theory would be just that evolution isn't an intelligent process at all, and mistakes happen. The fact is that our blind spot never gave us much trouble for survival in the past (it was never a significant handicap,) so the trait was passed on. Perhaps the same can be said about the occipital lobe and everything else: they worked well that way, and so that's how they were kept. — Kieff 22:31, 11 January 2007 (UTC)
Not clear that it's even a "mistake" or harmless chance, or that there aren't specific benefits to this layout. See Bergman, Jerry (2000). "Inverted Human Eye a Poor Design?" (reprint). Perspectives on Science and Christian Faith. 52: 18–30. {{cite journal}}: Unknown parameter |month= ignored (help) DMacks 22:49, 11 January 2007 (UTC)
Note that even if there is a benefit to this design, the actual principle in regards to evolution still holds since other organisms don't have our 'better' eye [11] Nil Einne 14:26, 13 January 2007 (UTC)
I asked this to a neurosurgeon, and this is his reply to the best I can remember it: Much of the body grows outward. So, the cells that eventually become the optic nerves develop from the middle of the brain outward. It would be possible for them to develop as non-crossing paths, like ><, but it is just as easy to develop in a crossing X. The main question isn't why they cross in the middle of the brain - it is why do they need to develop from the start. If they developed very late in growth, the entire optic part of the brain could be consolidated right behind the eyes. --Kainaw (talk) 22:32, 11 January 2007 (UTC)
During embryonic development the retina forms from the brain. The part of the neural tube that forms the retina folds over another nearby part that forms the retinal pigment epithelium. The rods and cones are metabolically very active and they shed a large part of their structure every day. The adjacent cells of the pigment epithelium act like a garbage disposal to phagocytize the discarded chunks of the visual cells. Thus, it is very useful to have the photo-detector cells "inside". The other cells of the retina are essentially transparent, so it hurts nothing to have them on the "outside", with light having to pass through most of the retina to get to the light-sensitive part. It is a simple matter for axons to follow the "stalk" of the retina back to the rest of the brain. In an animal like humans with over-lap in the visual fields of the two eyes, not all of the axons in the optic nerves cross to the other side of the brain. This allows information from the two eyes about the same object to be combined in the brain. The target for the axons of the optic nerves is not the back of the brain. The lateral geniculate nucleus is a major target for retinal axons in humans. In evolutionary terms, it was a later "invention" to send additional axons to the back of the brain and use for vision the vast amount of cerebral cortex that is available there. --JWSchmidt 05:31, 12 January 2007 (UTC)

Is it possible to convert random solid objects into liquid? edit

Is it possible to convert random solid objects into liquid? Feel free to laugh at me if this is a ridiculous question. For instance, can you take something like an iPod, fingernails, a pen, etc and convert them into a liquid by methods of grinding, melting, boiling, something along those lines?

Basically, what if you put an iPod into an extremely powerful juicer? NIRVANA2764 22:04, 11 January 2007 (UTC)

Grinding up something solid won't magically change it into liquid. It'll just make many small, solid pieces. Melting it would help, of course. Putting something like a tomato into a juicer is another story- the tomato was largely liquid to begin with. Maybe try reading melting or liquid for more info. Friday (talk) 22:12, 11 January 2007 (UTC)


If you were to heat an iPod, most of it would liquify, but at different temperatures, and by the time the last of it had liquified, the plastic parts would have done something other than liquify (say, burn). Fingernails cannot really be liquified. They would undergo destructive distillation. Eventually you would be left with a mineral residue that would probably melt (rather than sublime), but you couldn't really call it "fingernail" anymore. --Trovatore 22:15, 11 January 2007 (UTC)
There may be ways to liquify a variety of objects or substances. Things which would burn could be heated in a reducing atmosphere, or they could be ground up and dissolved in a solvent, creating a liquid solution. Metals can be melted and plastics, bone, minerals or ceramics can be placed in solution. Metals can also be (in general) placed in solution. Edison 22:40, 11 January 2007 (UTC)

Will it Blend? That is the question ≈Eh-Steve 01:03, 12 January 2007 (UTC)

Chemistry Naming question edit

Is there another name for this:

2,2,3,4-tetramethlyhexane

Like a proper nomenclature name? Thanks.

That seems like the proper (systematic) name to me. Are you looking for a trivial name? --Bennybp 00:42, 12 January 2007 (UTC)
Oh, except for the typo (2,2,3,4-tetramethYLhexane). --Bennybp 00:49, 12 January 2007 (UTC)

Plans for building a spaghetti bridge edit

Are there any tips for building a spaghetti bridge? Any engineering help on design ideas for added strength without compromising weight? Do share!

Most modern bridges are suspended bridges - unless you are using spaghetti and string, that design will not be of use to you. Look at older bridges made completely of steel girders (or wood planks). You will see how they built strong bridges with the least material possible. Copy their design. --Kainaw (talk) 23:44, 11 January 2007 (UTC)
Most? I think not. Suspension bridges are used primarily for long spans over deep water or perhaps other deep drops. Most bridges are concrete-and-steel affairs with intermittent supporting piers as necessary. See Girder Bridge. Dfeuer 00:05, 12 January 2007 (UTC)
Correct. I already jumped to the conclusion that this is a school assignment (a very common one in physics and engineering classes). The assignment is to build a long bridge span using a small and weak material. The teacher puts weights on the bridges. The bridge that supports the most weight gets the highest grade. --Kainaw (talk) 00:55, 12 January 2007 (UTC)

Do not boil the spaghetti. Paul Silverman 23:55, 11 January 2007 (UTC)

You could boil the spaghetti to make it soft and sticky. Then, twist a large group of them together. Wait for them to dry (put them in a dehumidifier if you have one). When they dry back out, you'll have a thick spaghetti rod that is stronger than a single strand. --Kainaw (talk) 23:59, 11 January 2007 (UTC)
I rather like the idea of a suspension bridge built on Kainaw's idea of boiling, forming, and drying.
Atlant 01:15, 12 January 2007 (UTC)
Yeah but how brittle is dried spaghetti?--Light current 02:50, 12 January 2007 (UTC)

You can also mold or extrude wet spaghetti into beams and even add fibers to eliminate the effect of its brittleness but don't store your finished bridge in the garage or it may become food for mice and rats. Barringa 03:17, 12 January 2007 (UTC)

The rats use the nutrients to grow, so could you then sacrifice them and use their bones to build the bridge (or better yet, feed it to your bratty little brother, etc:)? The pasta was used to make the bones were used to make the bridge, so sneak in on a technicality and the transitive property? DMacks 03:53, 12 January 2007 (UTC)
I particularly suggest the femurs for this technique. X [Mac Davis] (DESK|How's my driving?) 12:14, 12 January 2007 (UTC)
Remember that the triangle is the strongest polygon; wherever you have any squares or rectangles, add some crossbracing to turn the shape into a series of triangles. In addition, I'd guess that spaghetti has far more tensile strength than compressive strength; although a suspension bridge might would be too complicated, it would be far stronger than a simple beam or girder bridge. Laïka 15:06, 12 January 2007 (UTC)

January 12 edit

Car crash edit

There is a 2000 Kg car going 14 m/s north, and it crashs into a 4500 Kg truck that is going 22 m/s east. what is the resultant magnitude and its direction.24.215.147.206 00:10, 12 January 2007 (UTC)

You can calculate the momentum of both bodies (m × v) and add the vectors using the top-and-tail method. (You find the magnitude by using Pythagoras' theorem to find the momentum of both bodies combined, then dividing the figure you get by the sum of masses of both bodies (i.e., 6500 kg) to give you the velocity; then work out the angle.) You need to assume that the bodies stick together. BenC7 00:28, 12 January 2007 (UTC)
I am assuming that at this time of year that that was not a homework question. BenC7 09:25, 12 January 2007 (UTC)
Where are you located that doesn't have schooling at every level this time of year? --66.195.232.121 19:09, 12 January 2007 (UTC)
Dear anon, might I suggest that ben is located somewhere in the Southern Hemisphere? --Robert Merkel 02:10, 13 January 2007 (UTC)
Whoops. BenC7 03:10, 13 January 2007 (UTC)

Farting between sexes edit

Is it true that women actually fart as much gas as men but pretend thay dont and try to hide it by coughing and blaming the smell on the dog etc?

Yes. Poor dogs. --jpgordon∇∆∇∆ 03:20, 12 January 2007 (UTC)
I don't believe there are any scientific studies on the subject. X [Mac Davis] (DESK|How's my driving?) 12:13, 12 January 2007 (UTC)
But there's no real reason to assume that females have less or more flatulence than males. Though I could imagine that if male and female diets differed by much, which is plausible, that there could be different rates. But no physiological reason. --24.147.86.187 15:26, 12 January 2007 (UTC)
Quite the opposite. The male and female bodies exhibit numerous physical differences (e.g. anatomy, average height, hormonal levels, physiological parameters, ...); there is no reason to assume that the incidence of flatulence is exactly the same in both sexes. Believing otherwise in the absence of evidence is to base a conclusion on ignorance. --71.175.23.226 18:33, 12 January 2007 (UTC)

Farting is a function of a few things: 1, amount of swallowed gas/carbonated beverage; 2, amounts of certain carbohydrates in the diet; 3, type of gut flora; 4, amount of gut flora; 5, gut motility. I would think that the only likely significant differences would be in 2 and 4 (because women are generally smaller and thus would both eat less and have shorter intestines). Note that 2 and 3 are linked. Different people have different kinds of bacteria in their guts, which use different kinds of carbon. Almost everyone has hydrogen sulfide-producing bacteria, only about 1 out of 3 people has methane-producing bacteria. Tuckerekcut 22:03, 12 January 2007 (UTC)

Didn't Mythbusters did that? ...here you go youtube link

How does light slow down in glass? edit

Why is light slower in glass than in vacuum? Do the photons actually slow down, or do they bump into atoms, or do they curve around the atoms? Why is red light faster than blue light? Thanks, AxelBoldt 03:45, 12 January 2007 (UTC)

See Permittivity --Tbeatty 03:47, 12 January 2007 (UTC)
Couldn't find the word "photon" in that article, nor the dependency of the speed of light on the wavelength. AxelBoldt 04:00, 12 January 2007 (UTC)
Light is a form of electromagnetic radiation, so all the discussion of electric fields is relevant. Regarding wavelength dependence, see the "Complex permittivity" section. DMacks 04:07, 12 January 2007 (UTC)
Sorry, I just read the article again, but couldn't find information as to whether photons slow down in glass, nor could I extract from the complex permittivity section an explanation of why red light is faster than blue. Maybe it's in there, I just couldn't get it out. AxelBoldt 22:48, 12 January 2007 (UTC)
Blue light doesn't travel faster than red. All wavelengths of electromagnetic radiation move at the same speed in the same medium. --Cody.Pope 04:51, 12 January 2007 (UTC)
I don't believe this is true. Nonlinear dielectrics can have permittivity dependant on frequency. Also, the speed of light in a dielectric does not have same "speed limit" as light in free space does. Namely other particles can travel faster than light in that medium (see Cherenkov radiation. Tbeatty 05:06, 12 January 2007 (UTC)
In vacuum all wavelengths have the same speed, but in other media the speeds typically differ. A prism separates the colors because of their different speeds.AxelBoldt 22:59, 12 January 2007 (UTC)
Also see refractive index. Tbeatty 05:06, 12 January 2007 (UTC)
Also see Dispersion (optics). The material will have a different permittivity for different wavelengths of light. The Photon 05:25, 12 January 2007 (UTC)
The actual photons must travel at c individually, as much as any one speed and energy can be assigned to objects that may exist for so short a time over so short a space. The usual interpretation is that the waves of polarization and magnetization in the material (which are responsible for regenerating the wave, albeit at a lower speed, causing transparency) result from absorbing the incident batch of photons and that a new batch, somewhat behind where the originals would be if they had been unimpeded, is created shortly thereafter (although of course it's not in coherent bunches but rather on an atom-by-atom basis). --Tardis 06:40, 12 January 2007 (UTC)
Yes, that makes sense, thanks. Does this model also somehow explain why red light is faster than blue light? AxelBoldt 22:48, 12 January 2007 (UTC)
Different colors of light have different frequencies and so the polarization and magnetization of the particles in the medium proceed somehwat differently. (In an ideal linear medium the differences are nil.) See dispersion. --Tardis 23:57, 13 January 2007 (UTC)

Light speed and time edit

It is my understanding that one appears to slow down (to an outside observer) the faster one moves.

If I was travelling close enough to the speed of light so that my motion looked to another observer like it had ceased, but headed for the other side of the room, what would the observer get when he reached over and touched me? A light "echo"? Would his hand just go through? 05:06, 12 January 2007 (UTC) — Preceding unsigned comment added by 164.11.204.52 (talkcontribs)

The room distance also contract so it works out. See Lorentz contraction and Relativity of simultaneity --Tbeatty 05:11, 12 January 2007 (UTC)
The below answer is probably closer to the target, but your response reminded me of a wonderful physics problem I came across in high school: Suppose a pole with a rest length of twenty feet is traveling fast enough that it is relativistically shortened to 15 feet. Now imagine that it enters a 15-foot-deep garage. It will, of course, fit (briefly) in the garage. The problem, of course, is to explain how this is consistent with what happens in the frame of reference of the pole, in which the pole is 20 feet long, and the garage is only 11.25 feet long. Dfeuer 07:46, 12 January 2007 (UTC)
IIRC, that problem is addressed in some article over here at Wikipedia. The article mentions a fly inside hole when something
Yeah, I decided to look that up too. Here it is: Wikipedia:Reference_desk/Archives/Science/2006_October_19. The page linked has a another page on the specific pole example at [12]. — Kieff 03:32, 13 January 2007 (UTC)
Actually, Kieff, the page you pointed to contains errors, and misses the interesting point about what happens if you do close both garage doors while the pole is inside. (The answer to that question has to do with the finite speed with which the information that the front has hit a door can propagate back along the pole to tell the rear end to stop: the front end of the pole by necessity scrunches up (for real, not by Lorentz contraction!) while the rear end keeps going, and in the pole's own frame this compression is enough to allow the rear end to pass through the front door before it closes.) I edited the Wikibooks page about this [13] to explain it better. --mglg(talk) 22:40, 12 January 2007 (UTC)
I wouldn't know, I never really checked it (I just had it on my "to check later" bookmark folder along with some other 50 thousand pages I'll probably never look at.) That Wikibooks page is pretty good, though. — Kieff 03:07, 13 January 2007 (UTC)
I think what you're saying is that you expect your friend to see you stationary, "frozen". While it's true that he will see you as experiencing not much time (your watch will rarely tick, for instance), you will still be seen as moving at the proper speed: there's no confusion about where you are, merely how much time you experience. --Tardis 06:35, 12 January 2007 (UTC)


Tardis is absolutely right. Although time in your frame of reference, as measured by a stationary observer, appears to have slowed down that does not mean that "my motion looked to another observer like it had ceased". You still appear to be mvoing very quickly in the observer's frame of refernce - but your watch appears to be running slowly. Relativistic particles do not stand still - they just last longer before they decay. See time dilation for more details. Gandalf61 11:36, 12 January 2007 (UTC)
For a refreshingly different (and stimulating) view of light, space and time, try Alice in Quantumland by Peter Russell. --Seejyb 16:30, 12 January 2007 (UTC)

Tell me about the physical substance of memory and personality edit

I'm sorry if my question is unclear but I don't know how else to articulate it. Are there any accessible books on the subject?

Thanks.

66.91.225.188 10:08, 12 January 2007 (UTC)

You might try to articulate what you mean by "physical substance" — it is a rather vague term. One approach to articulating it might be to try and articulate what sort of answer you are looking for — neurological? genetic? psychological? physiological? etc. If you are talking about the neuroscience of memory, our article on memory does a relatively good and understandable introduction to it. Personality is a much more ambiguous term, referring to a number of different types of mental activity, but our article on that has some links as well. --24.147.86.187 15:24, 12 January 2007 (UTC)
You seem to be looking for something much more than encyclopedia content. I assume you mean "hard coding", i.e. where exactly is memory (or personality) stored, and what are the structures or molecules involved? You are asking for books, not articles, so you are likely to get a more satisfactory response at a sites designed to point you to suitable books. For memory you would get quite a few reviews by searching for "memory and learning" literature, e.g. this very helpful thread. At sites such as BrainConnection you can use the search box to find articles and related books. --Seejyb 16:19, 12 January 2007 (UTC)
You might find this online textbook to be useful. The book "Memory: From mind to molecules" by Squire and Kandel is fairly easy to read. --JWSchmidt 18:08, 12 January 2007 (UTC)
The Scientifically Correct expression to use here is physical substrate, as in this Google search query.  --LambiamTalk 20:17, 12 January 2007 (UTC)

Quantum Physics edit

Can two observers detect one photon and by extension,would an object emitting only one million particles exist for two million observers?208.101.159.59 14:17, 12 January 2007 (UTC)Giles

My instinct would be that the answer should be "no" — "detection" of a photon should mean that another "observer" should not be able to detect it again in strict terms (in semi-practical terms, it would be the equivalent of saying that once a photon has triggered a detection in a photomultiplier tube it cannot trigger any further detection in another tube; at least, not in a way which would associate it with the object in question). But I am not a physicist, and this could be totally wrong. --24.147.86.187 16:35, 12 January 2007 (UTC)
Certainly - here's a thought experiment proof-of-concept: if a single photon is detected by a photomultiplier tube, anyone standing around the output readout will notice the single photon; i.e. multiple observers have detected one photon. I actually think your question is not well defined; what do you mean by two observers detecting one photon? Do you mean simultaneously? I believe that a single particle can only interact with one other particle at a time (Feynman's idea, I think), but I don't know much about quantum field theory. -bmk
What you've done there is taken the detection of the one photon and used a machine to generate more photons to describe it. I'm pretty sure that's not what is being asked about — it is not a quantum physics problem in any case if that is all it is. I'm pretty sure the question is whether or not more than one observer (in these examples, the tube, which uses the photoelectric effect to determine whether a photon has hit it or not) can detect the same photon. Transforming a single photon into a bit of information and then having other people see that piece of information is not the same thing as having multiple observers detecting the same photon—detecting here should have a pretty well-defined meaning. --140.247.249.100 18:13, 12 January 2007 (UTC)
Through quantum entanglement two observers can obtain information about the same photon in independently conducted measurements. Some source must have emitted two photons for that, though.  --LambiamTalk 20:24, 12 January 2007 (UTC)
I still maintain that the question is ill-defined. Is the question, "can two observers simultaneously observe a single photon", or is it "can one observer observe a photon, then another observer observe the same photon?". I'm not really sure what the answer would be to the first question, but the second question is somewhat trivial; photons have no identities. They are just quanta of the electromagnetic field. They are identical bosons, and are fundamentally indistinguishable. --bmk
... and does "observe" only mean "interact with the photon directly" or can it mean "observe effects of the photon" or "observe a record of an interatcion" ? And if "observing" the photon changes its energy/frequency/wavelength, is it still the same photon after the interaction, thus allowing it to be "observed" multiple times ? Agree that question as it stands is not well defined. Gandalf61 11:12, 13 January 2007 (UTC)

Tungsten edit

What are some of the chemical properties of tunsten? The article isn't much help... 209.81.119.178 15:01, 12 January 2007 (UTC)

Try WebElements, following the links in the menus. --Seejyb 16:59, 12 January 2007 (UTC)

Thermal Conductivity? edit

What materials have high thermal conductivity, but are not metals? (other than diamonds) —The preceding unsigned comment was added by 216.182.149.254 (talk)

Probably not what you were thinking of (single materials), but heat pipes.
Atlant 23:46, 12 January 2007 (UTC)

January 13 edit

Conservation of energy? edit

when we push a wall and are unable to move it where does the energy goes —The preceding unsigned comment was added by 59.161.1.40 (talk)

  1. Ceramic
  2. your muscles heat up — Vranak
You are not expending any energy if there is no movement! Work = force x distance. So no distance --> no work. No work = no energy.

--Light current 00:19, 13 January 2007 (UTC)

You certainly are expending energy -- the energy just isn't doing work on the wall. As the previous poster said, your muscles heat up. --Anonymous, January 13, 00:44 (UTC).
Complete rubbish! —The preceding comment was added by Light current (talkcontribs) 01:15, January 13, 2007 (UTC).
So according to LC's theory, in climbing Mount Everest and returning you are not expending any energy, since the net distance = 0. And remember kids, no distance, no work!  --LambiamTalk 02:29, 13 January 2007 (UTC)
Pushing a wall is the same as leaning on the wall, same as sitting on a seat. No energy is expended. --Light current 02:48, 13 January 2007 (UTC)
Pushing a wall is not the same as leaning on it. Pushing is an active process that, assuming the wall does not move, produces an isometric contraction in muscles doing the pushing. At a microscopic level, crossbridges are cycling like mad to build up the tension to push the wall. Of course, you never build up the tension required to push the wall (hence it remains stationary), but the mere fact that you've begun pushing means that crossbridges are forming. Crossbridge cycling in skeletal muscle uses loads of energy in the form of ATP, so saying that pushing on a wall requires no energy is simply false. Care must be taken when attempting to interpret gross physical observations at the physiological level. The problem here is that you're assuming a human pushing a wall is equivalent to one point mass exerting a force against another more massive point mass. And that's just not the case. Cheers, David Iberri (talk) 03:08, 13 January 2007 (UTC)
Work is accomplished through small changes in position of cells in the muscle. This motion is converted to heat energy. It takes work and therefore energy to deform the cells of the muscle. --Tbeatty 03:00, 13 January 2007 (UTC)
If your muscles were not vibrating when exerting a force, there would be no heat produced. Energy is NOT expended by a force that does no work!--Light current 03:08, 13 January 2007 (UTC)
The person may not do work on the wall, but work is being done inside the person's body. A person pushing on a wall all day will certainly get tired; chemical energy is being used up. They would get hot & sweaty, etc. Anyway, I also think that perhaps (if you could imagine a wall made of something soft - say, rubber) that the space between molecules becomes compressed, albeit by a very small amount. So while it is being "pushed on", energy is stored in the wall. When the person lets go, the molecules pop back into their equilibrium position. All this would probably be negligible, however, compared to the energy converted to heat in the person's body. BenC7 03:20, 13 January 2007 (UTC)
As it happens, LC, I've addressed this precise question before. There doesn't have to be any macroscopic motion at all in order for work to be done: consider melting butter in a dish that is already exactly the right size to hold it. --Tardis 03:40, 13 January 2007 (UTC)

edcon

You say:

if you could imagine a wall made of something soft - say, rubber) that the space between molecules becomes compressed, albeit by a very small amount.

So in this case, movement is involved and (external) energy is indeed stored in the wall (just like you have stored a lot of potential energy by your body being at the top of everest) 8-)--Light current 03:44, 13 January 2007 (UTC)
All that energy storage in deforming the wall happens within a few milliseconds of you pushing on it — your muscles continue to become hot from doing work on themselves for however long you push. The latter effect quite dominates the former in total work done for any reasonable length of pushing. --Tardis 05:36, 13 January 2007 (UTC)
Well, clearly, everyone's right. Let's use a less ambiguous example than pushing on/leaning on a wall. A pillar supporting an object does no work on the object because the force it exerts on the object does not move along its line of action. A person holding the same object above their head does do work in order to keep their muscles extended. As a result, their muscles turn chemical energy into heat energy (remember, energy is not consumed, only transformed from one form into another). But still no work is done on the object - its energy is still unchanged. Beginners in physics/mehcanics are sometimes confused by this, and assume that a body that is doing work must be doing work on something, which is not always the case. Gandalf61 09:46, 13 January 2007 (UTC)

Gallium Reactions edit

Is there a list of materials that gallium combines with or corrodes?

Our article, gallium, mentions several gallium compounds including gallium arsenide, magnesium gallate, gallium citrate, and gallium nitrate. We also have a category, gallium compounds. - Nunh-huh 20:58, 12 January 2007 (UTC)
I am looking for a supplier that can give me the best price on a certain metal. I live in the US and have looked at Sigma and Aldrich but I wanted to shop around a little to see what other purities/prices others are offering.

Chemical supplier? edit

What are good sources to order chemical supplies from? Suppliers similar to sigma aldrich, fisher sci, etc.

Uh, Sigma and Fisher would be good places to order chemicals from... So what is your question, precisely? Do you want a long list, or are you looking for the best (or any) source of a particular chemical, or a very high purity, or somebody that will sell to hobbyists, or what? --mglg(talk) 21:29, 12 January 2007 (UTC)
And of course the answer might depend on what country you're in. --Anon, Jan. 13, 2007, 00:45 (UTC).

I am looking for a supplier that can give me the best price on a certain metal. I live in the US and have looked at Sigma and Aldrich but I wanted to shop around a little to see what other purities/prices others are offering.

Consider Wikipedia:Chemical sources --Mdwyer 06:40, 13 January 2007 (UTC)

Question 1 about my eyes edit

Two unrelated questions about my eyes: 1. Is there anything I can do to make the white part whiter?

It's best to keep unrelated questions separate, so I'm taking the liberty of splitting this item into two. --Anonymous, January 13, 00:52 (UTC).
If you think your sclera is unusually colored, then please, go to an opthamologist and let them answer your questions. — Kieff 03:20, 13 January 2007 (UTC)
Digital photo editing is how eye whitening is done. :) Otherwise, the only way I know of the change the color of your sclera is strictly related to the blood vessels. That is, vasoconstrictors (Visine) make the vessels shrink, and so the redness of your eyes is lessened. The effects are only temporary, and if you have concerns, you should really contact your eye doc. --Mdwyer 06:32, 13 January 2007 (UTC)
Sleeping well might help too Nil Einne 14:14, 13 January 2007 (UTC)

Question 2 about my eyes edit

2. How does squinting help me read a little farther when I don't wear my glasses?

Light rays from any particular point on the thing you're looking at spread out in all directions. When you see clearly, it's because rays from the same point that spread out and hit different parts of the pupil of your eye are properly focused by the lens of the eye to hit the same point on the retina.
If you need glasses, it's because the lens is not able to adjust to the right shape to achieve this focus. Then the rays from the same point hitting different points on the pupil will spread out to hit different points making a circle on the retina (the circle of confusion). If you look at a small bright light without your glasses and it's at a distance that you can't focus on, you will see that it spreads out into a circle.
When you squint, this obstructs part of the pupil, so the rays that get through to the retina are starting from a smaller part of the pupil. This reduces the circle of confusion. If the focusing defect wasn't too bad to start with, this may be enough to help you pick out the sharp edges you need to read. Incidentally, you may also notice that in bright light you can read without glasses better than in dimmer light even though you can still see to get around. This is for the same reason: your pupil shrinks in the bright light and again the circle of confusion is reduced.
--Anonymous, January 13, 2007, 00:55 (UTC).
You might want to research the pin-hole camera to see more about that whole circle of confusion thing. It is really pretty neat! The other reason squinting might help is because the force against your eye changes the shape of it. By changing the shape, you change the focus, and for a bare few seconds, you can see better. This is often why the first sign of eye trouble in children is rubbing their eyes in school. --Mdwyer 06:38, 13 January 2007 (UTC)
See also pinhole glasses. (There was also a classic Far Side cartoon featuring "early microbiologists" squinting through their fingers (in lieu of microscopes) at their primitive stone petri dishes, but I can't seem to find a copy on the web to link to.) —Steve Summit (talk) 14:48, 13 January 2007 (UTC)

Frog in boiling water edit

I first read about the analogy of the frog in boiling water in the book The Story of B, then I saw the analogy again in the film An Inconvenient Truth. The analogy is that if the frog is placed in a pot of boiling water, it will jump right out. However, if the frog is placed into a pot of room-temperature water, and the water is slowly increased to its boiling point, the frog will boil to death. The environmental plight of humanity has been compared to this. What is the origin of this analogy? If this does not apply to science, please move wherever necessary. —Erik (talkcontrib) - 02:45, 13 January 2007 (UTC)

According to Frogs in popular culture, it came from the book: "Life and Death in the Executive Fast Lane" by Manfred Kets de Vries. It is a myth, as stated in that article. My guess is that they completely made it up. If I put my frogs in cold water, they tend to jump out straight away. --liquidGhoul 03:05, 13 January 2007 (UTC)
The book you mentioned was published in 1995, but The Story of B was published shortly after in 1996. Is it possible that the comparison (however false, apparently) originated from something earlier? —Erik (talkcontrib) - 03:08, 13 January 2007 (UTC)
I can't find any more info on it. It is mostly used to explain to lay-persons about environmentalism. So there is a good chance that it is a recent story. However, I don't really know. If we can find an earlier book about it, I may look it up more. Thanks. --liquidGhoul 03:21, 13 January 2007 (UTC)
As spoken folklore it dates back much farther than that, as I have been aware of the analogy for maybe 3 decades. I doubt it is based on fact, but it certainly a picturesque and useful analogy. alteripse 04:17, 13 January 2007 (UTC)
Per the above, it is apparently untrue. BTW, anyone else remember any other recent TV shows (movies or whatever) where the frog analogy occured? I seem to remember hearing it (possibly reading it). But it wasn't either of the above since I haven't see or read either of them (it could have been from a trailer but I doubt it) Nil Einne 14:12, 13 January 2007 (UTC)
I can't see how it's a useful analogy when it's false to begin with. I suppose it's useful if your intent is to mislead. Vranak
What would be a more appropriate and accurate analogy to reflect that people tend not to react to slow changes as potently as quick changes? —Erik (talkcontrib) - 17:13, 13 January 2007 (UTC)
I've found at least one reference to it dating from 1990 (via JSTOR), so it must be earlier than both of those. Crispin Tickell, "Human Effects of Climate Change: Excerpts from a Lecture Given to the Society on 26 March 1990", The Geographical Journal, Vol. 156, No. 3. (Nov., 1990), pp. 325-329, on 325. --24.147.86.187 18:07, 13 January 2007 (UTC)
Hey — we have a boiling frog article. Whoa. --24.147.86.187 18:11, 13 January 2007 (UTC)
Via Lexis Nexis, I found a reference dating from 1980 from an article about survivalists who use the analogy to describe how they think society is going to collapse. Cammille Recchia, "Area Survivalists Circle Wagons for Coming Armageddon; Survivalists Prepare to Ride Out Armageddon; Fearing Economic Chaos, Advocates Store Food, Buy Gold, Silver", Washington Post (25 August 1980): C1. I'll keep poking around... (Can you tell that I do historical research for a living?) --24.147.86.187 18:16, 13 January 2007 (UTC)
OK, wow, I got a reference back to at least 1960. It's pretty humorous. It is an article about a certain J. Hannibal Rustbugle (who is likely fictional) who has gone into a reporter's office and demonstrate said frog experiment. "Behold the moral. The frog dropped into boiling water water has sense to leap out, but the frog dropped into cold water can be cooked to death before he realizes he is in serious trouble. So it is with us Americans and our civilization in this mounting crisis. We must beware of those who want to thaw the cold war out at any cost. We may be cooked before we realize what has happened. Every day in every way, enemies, dupes, and some well intentioned persons are busily stoking fires under the water. They are seeking to boil our freedom into nothingness. They can only be stopped if we keep a close eye on the situation." From Walter Trohan, "Report from Washington," Chicago Tribune (6 June 1960): 2. I doubt this is the first instance of its being committed to print, though it is the earliest I was able to find with all of my electronic databases. --24.147.86.187 18:26, 13 January 2007 (UTC)
Wow, that's incredible. Very interesting to know. Thanks for doing the research! —Erik (talkcontrib) - 18:28, 13 January 2007 (UTC)
A previous query seems to trace it back to at least 1952. My uninformed guess is that this became part of some sort of campaign speech that some candidate or another gave and made the rounds that way, but I don't know. I haven't found any earlier references. I would not be entirely surprised if it went back to the late 19th century, when researches into things like the physiology of boiling frogs was all the rage (I exaggerate, but only a bit), though as of yet I see no evidence. --24.147.86.187 18:45, 13 January 2007 (UTC)
I updated the boiling frog article with those references and a few choice quotes. A nice way to spend 20 minutes on a Saturday afternoon! --24.147.86.187 18:58, 13 January 2007 (UTC)
It is very old indeed! I have found a reference to it dating from 1893... will keep looking... --24.147.86.187 19:14, 13 January 2007 (UTC)
OK — here is the earliest I could find, and now I give up and continue my day:
By very gradual increase or decrease of temperature, a frog may be boiled or frozen to death without making the smallest movement.
From Harald Höffding Outlines of Psychology, Mary E. Lowndes, trans. (New York: Macmillan and Co., 1892), on p. 108. The entire paragraph (about temperature sensation) references some books in German from the 1860s, which I do not at the moment have at my disposal to check (nor is my German really up to par for it). It is available through Google Books. --24.147.86.187 19:23, 13 January 2007 (UTC)
Again, thank you very much for the research. Your answer was more than enough. Cheers! —Erik (talkcontrib) - 19:44, 13 January 2007 (UTC)
This is "science" from the era of Aristotle, who did "thought experiments" but did not bother to test the theories empirically. Having raised frogs, I claim that when the temperature becomes uncomfortable, they move. I have not attempted to boil frogs, however. If i did, I expect they would taste like chicken. Edison 23:19, 13 January 2007 (UTC)
I had a feeling before I read the popular culture arrticle (which obviously needs to be updated), that it would come from Asitotle's (I would have said Socrates') time. I thought this was a dead disucussion, and came back to it being huge! Great work everyone. --liquidGhoul 11:18, 14 January 2007 (UTC)
Galileo did thought experiments that he didn't test. Einstein did thought experiments that he didn't test. Doing thought experiments is not what distinguishes one from doing good science or not. In any case it seems to have its origins in early physiology work, as I suspected. As to whether it would work or not, I imagine it would depend a lot on the conditions — obviously just dropped frogs into buckets is not going to work, but you could imagine finding other ways to tell their reaction to temperatures. --24.147.86.187 17:26, 14 January 2007 (UTC)

DVD Vs Real Life clarity edit

Question - Would watching a good quality DVD movie be clearer in seeing things around us in real life? If you understand what i'm talking about. I was just wondering because when you look around a room or something you have a small intereference of colour or patterns from looking at something bright to looking at something darker but we're so used to it that we don't really notice it, but it's still there. But if you look at a T.V. screen you don't get that interference. So does that mean watching something clear like a DVD would appear clearer than just looking around in real life? It's a hard question to explain, but I hope most people understand what i'm getting at.

No, it wouldn't be clearer, although it might seem that way if you were looking at a DVD of something magnified larger than it is in real life, or if the lighting was better than it was in real life. BenC7 06:04, 13 January 2007 (UTC)

Ampules edit

Why are ampules still used for some pharmaceuticals? They are very inconvenient - the drug has to be filtered before use, and sometimes they don't crack open the way they are supposed to. The vast majority of drugs use a vial with a rubber stopper, which is convenient for single or multi-use. --Joelmills 04:49, 13 January 2007 (UTC)

Just a guess, but I assume it is because they can be heat-sterilized using hotter temperatures than rubber stoppers. They're also probably cheaper to manufacture. --Mdwyer 06:28, 13 January 2007 (UTC)
Another reason is to protect individuals who have latex allergies: [14]. Glass is also less permeable that virtually any conceivable stopper material; this can extend shelf life of drugs and protect them from spoilage due to premature exposure to air. Finally, a glass ampoule is about as tamper-resistant as packaging can get. You can't add something to an ampoule without breaking it open, and you can't reseal it. TenOfAllTrades(talk) 16:00, 13 January 2007 (UTC)
Also, some substances are not stable on air over a longer time so are shipped in ampules containing inert gases like nitrogen. -- 85.179.8.121 18:49, 13 January 2007 (UTC)

Thanks for all the responses. I will refer my annoyed technicians to this page. --Joelmills 22:21, 13 January 2007 (UTC)

Astronomical Distance edit

This must be an exceptionally easy question for somebody here: how is the distance and age of stars calculated? I understand the red shift can measure the speed at which a star is moving away (speed of universe's expansion) but can it also be used to measure the stars distance from us? I thought maybe it was measured using parallax, but apparently the interstellar distances involved are so huge that this doesn't work. -Kelly

There is no single answer. See the article Cosmic distance ladder for information on various techniques (including both redshift and parallax) used to determine distances to various stars. A very common method is to identify a particular star as being of a certain class, knowing that such stars vary in brightness between two figures, comparing that known range to the observed brightness, and then computing a distance which would cause such a variation in brightness assuming various factors in the intervening space. While often giving a fairly accurate range this also sometimes produces incorrect results when the observed 'star' turns out to be two stars in close visual proximity, if there are higher intervening concentrations of interstellar dust than assumed, or due to other factors. Most of the methods of measuring stellar distances involve such uncertainties and it is only through cross-checking via different methodologies that accurate values may eventually be determined. --CBD 13:00, 13 January 2007 (UTC)
Parallax is the primary method for determining stellar distances - they are not too far away, the cosmic distance ladder is used to measure the distances to galaxies. Redshift is not used to determine distances to stars because they are not caught up in the expansion of the Universe.
For determining the distance to galaxies, see Hubble's law.
Age is estimated by spectroscopy. Basically, the intensity of the star's light is measured for each wavelength, and at some wavelengths the light will be much more (or less) intense than at the surrounding wavelengths. These peaks and troughs are spectral lines and they indicate the presence of different elements. Older stars have more metals, which are elements heavier than helium. --Bowlhover 21:28, 13 January 2007 (UTC)

Resistance edit

—carry out investigation of the relationship between lenght of a metallic conductor e.g nichrome wire and its resistance — Preceding unsigned comment added by 89.101.116.77 (talk)

Okay - I've done it - can I have a prize now? :P
Seriously - this sounds like a homework assignment - you really should work out what you are going to do yourself. If you have any specific questions then feel free to ask them here. --Neo 13:08, 13 January 2007 (UTC)
*nudge nudge* Electrical resistance *hint hint* -Obli (Talk)? 13:13, 13 January 2007 (UTC)
O.K., I finished the investigation. The longer it is, the greater the resistance, in direct proportion. Next homework question? Edison 23:22, 13 January 2007 (UTC)

Planet82 edit

Does anyone know how this SMPD (Single Carrier Modulation Photo Detector) sensor that everyone's talking about works? The planet82 FAQ [15] looks to me more like a collection of buzzwords like nanotech, quantum tech etc rather then containing much useful info. I admit I've only done very brief searches so feel free to ignore me. I didn't even read their FAQ properly just browser through it Nil Einne 13:55, 13 January 2007 (UTC)

Composition of Solar Radiation: are gamma rays present? edit

Simple enough, are gamma rays a part of solar radiation?

  • I was going to say the same thing but surprisingly, it's not actually very clear. The two spectrum graphs for example stop in the visible wavlength, hundred nm ranges. However if you take a look at gamma ray you should notice a picture with a revealing caption. I do think the articles need work tho... Nil Einne 15:22, 13 January 2007 (UTC)


Yes, the sun does emit gamma rays. It emits radiation of (almost) all wavelengths in a planckian distribution peaking in the visible.
Just out of curiousity, does the image at Solar radiation provide the necessary answer? I don't know enough about radiation to know. --jpgordon∇∆∇∆ 00:57, 14 January 2007 (UTC)
Not quite. It's cut off at 3 micrometers. Gamma rays are between 0.03 and 0.003 nanometers. If the graph was extended it should show some gamma rays (although very few)
3 micrometers? 30k cm^-1 is 333 nm unless I really screwed up my calculations. I.e. visible wavelengths as I said above (well okay near UV but close enough... :-P) Nil Einne 13:49, 14 January 2007 (UTC)

N.B. As I mentioned above, you can infer the sun emits gamma rays from a caption in that article:

The Moon as seen in gamma rays by the Compton Gamma Ray Observatory. Surprisingly, the Moon is actually brighter than the Sun at gamma ray wavelengths.

We can presume from the caption that the sun does emit gamma rays other wise it will say something like "Surprisingly, the Moon emits gamma rays even though the sun does not)Nil Einne 13:53, 14 January 2007 (UTC)

Audio file compression edit

I was raised in the dying days of vinyl, suffered through the Cassette tape, initially liked CDs, and have now begun downloading mainly classical recordings from iTunes. In that process I've seen the artwork, the drama, the visceral presence of the LP diminish into ghostly mp3 files and the occasional pdf attachment. In other words, I've been sandbagged into thinking that, despite getting less and less for the money, I'm somehow getting a superior product. And recently I learned that iTunes compresses audio files down to some certain rate, and that some aspects of the recording are lost in the process. I read through Compression artifacts but remain curious if someone can explain what, in general, may be getting lost in the journey from iTunes to my computer, and what a reasonably close listen to the CD v. the download might reveal, on, for example, a very recently released Deutsche Grammophon recording. Wolfgangus 17:28, 13 January 2007 (UTC)

Analog recordings vertically recorded on wax cylinders acoustically from the recording horn (using no amplification) obviously do not suffer the compression inherent in MP3. Slight surface noise and uneven frequency response are the only downsides of this method. Edison 23:24, 13 January 2007 (UTC)

Sounds minimal; not as drastic as I'd imagined. Thanks, Edison. Wolfgangus 00:55, 14 January 2007 (UTC)

Barring transmission errors, nothing will be lost from the journey from iTunes to your computer. A reasonably close listen may or may not reveal anything, since transparency is, by definition, very subjective. Oh, state-of-the-art compression can achieve transparency at 128 kbps for surprisingly many people (Wikipedia is kind-of outdated), so don't worry. --Kjoonlee 08:35, 14 January 2007 (UTC)

On that note, the subjectivity of transparency you're referring to will certainly favor me, if for no other reason my hearing isn't what it used to be. Thanks for the further input. Wolfgangus 09:58, 14 January 2007 (UTC)

yield stress edit

what is the prportional limit of steal48?σy — Preceding unsigned comment added by 196.205.218.36 (talkcontribs) 18:36, 13 January 2007

Are you looking for facts about steel? Our article has links to many web pages and tables. —Steve Summit (talk) 18:46, 13 January 2007 (UTC)

sonic screwdrivers edit

are they any good for undoing girls' bras? — Preceding unsigned comment added by 86.142.11.215 (talkcontribs) 19:02, 13 January 2007

Since sonic screwdrivers don't exist outside of the world of Dr Who, we can only speculate on tghe basis of what we've seen in the shows. (I'm not aware of any incident in any of the shows where the Doctor has made use of his sonic screwdriver in such a way.) On the basis that the sonic screwdriver can open almost any lock, operate and repair various pieces of equipment, and even drive screws, I would suspect that it could be used to undo a bra fastener, should the Doctor feel inclined to use it for that. -- AJR | Talk 20:12, 13 January 2007 (UTC)
Slight compression of the fastener between the thumb and first two fingers is adequate for the task and can do it in a split second. Facility will develop with practice. Edison 23:27, 13 January 2007 (UTC)

Memory foam outgassing and an air purifier edit

Would an air purifier with a HEPA filter help clean the air of the outgassing (smell and possible chemicals) caused by newly opened memory foam? Thanks. -- Win777 19:25, 13 January 2007 (UTC)

A HEPA filter, by itself, will have little to no effect on odours or chemicals. HEPA filters remove particulates, but not gases. TenOfAllTrades(talk) 19:45, 13 January 2007 (UTC)
It took a few days to get rid of the odor of my 2" thick topper (no idea how long it would take for a full mattress). No apparent side effects, but my back thanks me for it every morning. Clarityfiend 01:02, 14 January 2007 (UTC)

Entropy vs. Energy edit

If entropy is energy, what is the distinction? Is energy limitless while entropy is a word to describe a measurement over time? Looking back, was there limitless energy on earth and now we're looking at the temp of the ice in the glass becoming closer to the temp of the room in terms of entropy? How does any of this reverse global warming?68.73.2.117 21:51, 13 January 2007 (UTC)

That's a lot of questions, many of which I don't understand! Maybe Thermodynamic free energy will interest you? Melchoir 22:01, 13 January 2007 (UTC)
The energy on Earth was finite. In addition Earth is an open system and is continually being provided with energy from the Sun. David D. (Talk) 22:21, 13 January 2007 (UTC)

Introduction to entropy may have clues. --Light current 23:15, 13 January 2007 (UTC)

Don't quote me on this, but entropy is almost opposite to energy. A system with high entropy has low energy. Enthalpy is a word in the back of my mind that might have something to do with this.
Entropy is essentially the tendancy of energy towards ever less usefull heat. Imagine a universe where all energy was in the form of heat and everything was the same temperature. That situation is maximum entrophy, a situation where by the second law of thermodynamics it is impossible to do any more usefull work.

Visualization edit

Can it be practiced without thinking of a word to descsribe the image? If anxiety is word-related, wouldn't it be better to not use words... Say, if you're hearing your heart valves open and shut, can you redirect the thought to an Allis-Chalmers WD45 or the Dahli Lama and not feel panicky?68.73.2.117 21:57, 13 January 2007 (UTC)

Yes, I can. Edison 23:28, 13 January 2007 (UTC)
Most anxiety is not "word related", and visualization (depending exactly which meaning you had in mind) is the ability to think non-verbally, so you're better off without words.--Shantavira 09:22, 14 January 2007 (UTC)

It isn't always better to think without words though, I do that a lot and find it difficult to write about what I am thinking :) You could instead think about being calm :)Hidden secret 7 11:46, 14 January 2007 (UTC)

Far from home? edit

How far, in miles, can you possibly be from home whilst remaining on the Earth's surface? Stuarthill 22:37, 13 January 2007 (UTC)

If your home is on Mars, you can't get any closer to home than a little under 34 million miles while remaining on the Earth's surface. Also when your home is on Earth, it depends on where your home is, but less so. If it is on the equator, the farthest you can get measured along the earth's surface is half the equatorial circumference, about 12451 miles. If your home is on the South Pole, the farthest spot away is the North Pole; in this case the distance is half the meridional circumference, or 12430 miles. The difference is due to the fact that the Earth is not a perfect sphere. If your home is somewhere else, the answer is somewhere in between. For some pairs of places on Earth that are about as far away from each other as possible, see our article Antipodes.  --LambiamTalk 11:16, 14 January 2007 (UTC)

January 14 edit

Embryonic Stem Cells and the beginning of human life edit

My reading suggests that after an egg is fertilized in vivo, it is composed of stem cells that continue to divide at least until the egg attaches to the uterine wall. The write up under stem cells says "When given no stimuli for differentiation, ES cells will continue to divide in vitro and each daughter cell will remain pluripotent." The specific question is: Is this also true in vivo and what is the stimuli in vivo that begins the differentiation? The answer I am expecting is that it is something in the process that is initiated following attachment of the egg to the uterus that begins the next stage of embryonic development and if the egg does not attach, it degenerates and gets flushed with the next menstrual cycle.--Billxm 00:23, 14 January 2007 (UTC)

Much more is known about early mouse development than early human development, but during early human embryo development the cells start to differentiate before the embryo implants in the uterus. Two types of cells form according to their position in the embryo, Inner cell mass cells and trophectoderm cells. For mice, it is thought that signals from the uterus help the trophectoderm cells differentiate and get ready for implantation. See: Blastocysts don’t go it alone. Extrinsic signals fine-tune the intrinsic developmental program of trophoblast cells. While the cells of the inner cell mass remain as multi-potential stem cells until about the time of implantation, they quickly form two epithelial cells layers, the epiblast and the hypoblast (Wikipedia's short Epiblast article). --JWSchmidt 02:09, 14 January 2007 (UTC)

Metal Working edit

I want to construct this model gun from sheet metal for laser-tagging purposes. Given that the diameter varies along the length of the gun, how might I accurately fold the metal? And ideas for making the gun handle, which I'd like to construct from the same metal? The blue part is a plastic window. >>gun<< --Username132 (talk) 00:23, 14 January 2007 (UTC)

light pollution help edit

I am doing a science fair project related to light pollution. I plan to zoom in on 'earth at night' maps located on the Internet and count the number of pixels in a certain 'area' (e.g. mid sized city) and see if light pollution naturally occurs in areas where there is population. Unfortunately, few (or no) maps on the Internet are reliable enough for me to look at (because of low res, no state or country boundaries, chance of being Photoshopped to make the image look 'better', etc...) and the process seems too tedious (city boundaries hard to 'distinguish', time in counting pixels, etc...). Can anyone help me, like introduced me to some special software or a good image (ahem...I mean really good, and better than the IDA stuff or the World Atlas of Night Brightness)? And if you can find the latter (an image) please give me a suggestion on how I can make the project less tedious. If you think I should have a new project, please keep it so I don't have to go outside during the nighttime or have to buy too many things. Thank you! —The preceding unsigned comment was added by Member (talkcontribs).

You need GIS software to do this right. What you need is a georeferenced image of the night area you want to study -- finding high-resolution ones may require some digging, but with a little Googling I was able to find evidence of 5km resolution images -- and then you need to load it into ESRI's ArcGIS, using an extension called Spatial Analyst. You define a grid with the cell size equivalent to a pixel in your input image, and then fill each cell with a numerical value based on a color value from the original image (probably whiteness). Then you define another grid of the same size, filling its cells with population data (which you would pull from, say, the US Census 2000 data, if you are doing this for the US: note there are a lot of processing steps I'm leaving out). It would take several iterations to get it right. Then you can compare the numbers between the two grids and see what you find, most likely by producing a third grid which gives, in each cell, the ratio between the cells in grids 1 and 2; it would be interesting to see the outliers, i.e. what bright areas have little population, and what dark areas are heavily populated, and then to try to figure out what causes them. Here is an example of a paper I found where an agency has attempted to correlate population with night light [16] -- go to the very bottom of the page to see the reference. Do you have a geography lab or computer lab at your school with ArcGIS? (the software is not cheap: current ArcGIS plus Spatial Analyst will set you back about USD $4000). Antandrus (talk) 02:44, 14 January 2007 (UTC)

Shaking a bottle of ice and water edit

If I have a bottle of ice and water and want to drink the water -- will it help if I violently shake the bottle and splash the ice/water around?

Some things to think about:
You're putting kinetic energy into the system, and energy = heat. But, some of this energy will escape as sound, bugger all, though. Convection between the bottle and the surrounding air would also increase. However, shaken water is whiter and presumably reflects more light (energy) so in this respect shaking may hinder the melting. Breaking up the lumps of ice into smaller pieces may help by increasing surface area available to melt - also, if you break it up small enough, you can get it out the top and melt it in your mouth or stomach. Holding the bottle to shake it could also have an effect on conduction to the bottle. Your hand is warmer than the surrounding air (presumably) and a better conductor. 121.72.14.221 03:43, 14 January 2007 (UTC)

Hearing problems edit

I'm 14 years old but I seem to have hearing problems. I've always had the problem of hearing people talk, but never understanding what they are saying, especially when there is background noise or I am talking on the phone. This is obviously annoying because I either say what 5 times before understanding, or I just nod and say yes, not ever knowing what they just said. I also just listened to these high frequency "mosquito noises" on this website [17]. A teenager like me is supposed to be able to hear from 21.1 khz to 22.4 khz. I can only hear up to 14.9 khz, which is what people 39 and under are supposed to hear! I have never listened to music on earphones very loud for long periods of time and I don't go to loud concerts. However, I did have many ear infections and had to have ear tubes put in when I was young. Is my problem just going to get worse as as I get older? Maybe as far as though I will go deaf? Hopefully this doesn't count as medical advice. Thanks for your responses. :-). Imaninjapiratetalk to me 02:05, 14 January 2007 (UTC)

I went partially deaf once due to ear wax in one of my ears and flushing them helped a lot. I used a wax softener and a syringe that came with it to flush out tons of wax. You could also have a nurse do it at a hospital if you're feeling uncertain about it, just make sure your ear is healthy (no very recent or current infections). -Obli (Talk)? 02:20, 14 January 2007 (UTC)
Warm water (not too hot!) works as well as a softener (or even better) and is cheaper and less messy.  --LambiamTalk 11:32, 14 January 2007 (UTC)
I would think that getting a pro to look at the problem makes sense: 1. The poor hearing may be fixable (it probably can be improved), and it may be easier to fix than if you waited for say another five years. 2. There may be something you need to be doing - or avoid doing - to prevent it from getting worse. 3. There are some pretty important implications for social activities, sport and career choices. 4. Think of what fun it would be if you could hear all the sounds that your friends do. --Seejyb 04:20, 14 January 2007 (UTC)
You think YOU have problems? I'm fifteen and can only just hear the 12khz sound. Vitriol 04:30, 14 January 2007 (UTC)

Stop! Don't panic! Never use the computer to diagnose yourself or others. You didn't go into detail about how you listened to the tones. Unless you have a fantastic computer/audio system, you can't reliably test your hearing. The high frequencies won't get through most speakers. There's a good chance that you don't have a problem, but the only way to make sure is to have your hearing tested by a professional.Bunthorne 06:31, 14 January 2007 (UTC)

Fire Bottle edit

What sort of chemicals might these smart kids be using to create the fire in the bottle? Thanks for the help. --72.69.146.116 03:08, 14 January 2007 (UTC)

Any sort of "flammable" gas or volatile liquid after evaporating would probably work, but I'm not sure what they're using here. The trick is to get the ratio of oxygen and the chemical right. As long as there is no oxygen in the bottle, their won't be ignition until they're mixed outside of the bottle. But I wouldn't recommend trying it, unless you really know what you're doing. --Cody.Pope 04:15, 14 January 2007 (UTC)
It has to be a gas that's heavier than air; many of the common flammable gases, like carbon monoxide, natural gas, and methane, aren't. I'd go for lighter fluid vapour, since it is very accessible.
By the way, I think there has to be some oxygen in the bottle. The person holding it squeezed it, and then let go, sucking air in. --Bowlhover 04:33, 14 January 2007 (UTC)

Why do cats love fish? edit

Of course, fish is nutritious. But why is the common stereotype that cats, rather than dogs love fish? Our article on cats says that domestic cats successfully hunt for fish. I have seen many swimming dogs, but never a swimming cat (despite the external link in our article). — Sebastian 03:31, 14 January 2007 (UTC)

Where did these animals evolve? Dogs = wolves from the woods, maybe not many places to practice catching fish. Cats... er... Egypt? You wouldn't think there'd be many fishing holes in a desert. Maybe Africa like lions, there'd be a fair few watering holes to dabble in there. There, how helpful was that! 121.72.14.221 03:50, 14 January 2007 (UTC)
actually land mammals (not stuff like whales porpoises etc which have a more fish like body shape and only come onto land occasionally if at all) don't fish by swimming, fishing from outside the water has many advantages: you can move much faster, you don't get anywhere near as wet (and wet tends to lead to cold which is bad) and you are unlikely to be noticed by the fish.
also lets not forget that cats have far better claws than dogs (this certainly applies to the domestic variants but i think it applies to thier wild variants as well), maybe dog claws are simply not good enough to get fish out of water. Plugwash 04:07, 14 January 2007 (UTC)
Thanks, that makes sense! — Sebastian 04:24, 14 January 2007 (UTC)

Simple microcontroller stepper control edit

What solutions would you suggest for a simple stepper motor controller? Ideally, the microcontroller should only have to send two bits: the direction and a pulse. The motor controller would then move the motor according to the input.

Most websites recommend the Allegro UCN5804B for this type of application, but it has been discontinued. The replacement (A3967) is unfortunately only available in a SOIC package (which is much harder to prototype).

Do you have any recommandations or suggestions for this project?

A PIC microcontroller might do this simple job--Light current 03:51, 14 January 2007 (UTC)

it would but you'd still have to build the drive switching cuircuit from scratch. BTW you can buy dip-soic converter boards which you can solder your soic chip to, solder two rows of pins in and then drop into your breadboard or stripboard prototype. Plugwash 03:58, 14 January 2007 (UTC)

Why Mercury? edit

Why mercury is used in a thermometer?Vichu satheesh 04:25, 14 January 2007 (UTC)

We use mercury becasue it is a very heat sensitive substance. Compared to other elements, it expands very significantly due to temperature.Rya Min 05:53, 14 January 2007 (UTC)
Did you try reading mercury and thermometer?--Shantavira 09:29, 14 January 2007 (UTC)

manufacture of petorl edit

hello,

I would like to know the manufacturing process of petrol with more details than the normal one paragrph process. being a chemical engineer and gonna pursue my studies in oil and i would like to have a strong backgroud. waiting for your reply.


thank you for your co-operation .

You can start with gasoline and then look at [18] and [19]. Superm401 - Talk 06:20, 14 January 2007 (UTC)

Perpetual Motion edit

How is orbit not a perpetual motion device? The object is continuously falling, so therefore gravity should keep accelerating it, right?

  • Yes -- until you actually try to extract any of the energy from it. In an isolated system consisting of two objects orbiting each other gravitationally, the two objects will remain in orbiting motion forever, as long as they are not disturbed. However any attempt to extract useful work from this system would lead to a loss of energy. This would result in the objects slowing down and getting closer to each other, until at some point the objects would collapse together and no more energy would remain to extract. (from Perpetual motion.) --jpgordon∇∆∇∆ 06:08, 14 January 2007 (UTC)
That's not quite true. Two orbiting bodies will only remain in orbit forever if you're using newtonian gravity. In GR they slowley get close together.
    • I think this stems mainly from a problem in the conception of perpetual motion. Physicists only use that label for machines that are designed to do work. Superm401 - Talk 06:21, 14 January 2007 (UTC)

Forget frogs. How about crabs? edit

...as a metaphor. A few hundred centimeters up, we spoke about boiling frogs as a metaphor. Another metaphor has been going around for a while (see Crab mentality, [20], or [21]

Ok, so if you're too lazy to click on the above, in short it is claimed that if there is an open bucket of live crabs, none would be able to escape, since if one tried to climb out, the others would pull it back. I've heard this used as a metaphor for someone who tries to get out of the 'hood, that the other members will do all they can to keep people from bettering themselves.

So, any fisherfolk out there that can vouch for this supposed behavio(u)r of crabs? Bunthorne 07:46, 14 January 2007 (UTC)

Only crabby people would ascribe this to the crabs' mentality. A crab trying to claw its way out that is not positioned at the side of the bucket has no other leverage than nearby crabs. Those in the second layer of crabs (second-nearest to the side) will, mindlessly trying to clamber out, attempt to find leverage on the first layer. Because of Newton's third law they pull the crabs in that layer down. And because of Archimedes' Principle, they would have to push them well under water in order to get out themselves.  --LambiamTalk 11:51, 14 January 2007 (UTC)

physics-infrared photographic optics question edit

I am a photographer and have been doing infrared film photography since 1987. I was contributing to the Infrared photography article, and I came upon an assertion under the Film Cameras section that I could not confirm and that did not sound correct to me.

"Catadioptric lenses do not require this adjustment because mirrors do not suffer from chromatic aberration".

"This adjustment" refers to the refocus of a camera lens to compensate between visible light focus and infrared light focus. Is this correct? I just wanted to check since I teach infrared photography workshops and this would be good to know definitively. There are other optical assertions under Infrared photography that are so far out of my league I wouldn't know it they are right or wrong, but this one sounded odd. Thanks--Schafphoto 08:12, 14 January 2007 (UTC)schafphoto

Can woodpeckers manipulate objects with their feet? edit

 
Like so...

I notice from reading the woodpecker article that some species have zygodactyl feet (i.e. two toes forward and two toes back). Birds in the parrot family also have this configuration and are able to use their claws like hands to manipulate objects. Can woodpeckers do the same? --Kurt Shaped Box 11:46, 14 January 2007 (UTC)

From my (limited) experience watching Old World woodpeckers, no. Woodpeckers normally do not perch on branches like parrots do, but rather cling to the bark. That's what piciform zygodactyl feet evolved for in the first place. I presume clinging to the tree trunk makes it much harder to manipulate objects. BTW, birds do not have to have zygodactyl feet to manipulate things. I often see hooded crows using feet to manipulate objects. Cheers, Dr_Dima
Interesting. What have you seen the crows do? We have European Magpies, Carrion Crows and the odd Raven around here but I've never actually seen them 'manipulate' anything with their feet - unless you count putting a foot on a piece of food to hold it down whilst eating as 'manipulating'? --Kurt Shaped Box 16:07, 14 January 2007 (UTC)

q edit

what is the actual fullform of Rx in medical sciences?—

Are you talking about Medical prescriptions? (check the article) — Kieff 17:20, 14 January 2007 (UTC)

NORTH POLE TO SINGAPORE edit

how would an animal such as a polar bear adapt to the environment if it was taken fm the northpole and put in singapore??

It wouldn't. It would just die. Adapting to a new environment is a slow process that take many generations through many years. — Kieff 17:16, 14 January 2007 (UTC)

effect of total silence on human hearing edit

What are the effects on the human sense of hearing in total silence?Rick42774 14:13, 14 January 2007 (UTC)

From 4'33":
In 1951, Cage visited the anechoic chamber at Harvard University. An anechoic chamber is a room designed in such a way that the walls, ceiling and floor will absorb all sounds made in the room, rather than bouncing them back as echoes. They are also generally sound-proofed. Cage entered the chamber expecting to hear silence, but he wrote later, "I heard two sounds, one high and one low. When I described them to the engineer in charge, he informed me that the high one was my nervous system in operation, the low one my blood in circulation."
It's the best I can give you on that topic. — Kieff 17:13, 14 January 2007 (UTC)

Working Mechanism-SRE edit

Can you explain how the recently launched SRE-satellite vehicle by ISRO works and return back to earth without any damage by the viscosity of the air?59.92.82.44 15:32, 14 January 2007 (UTC)

Zebra subspecies identification edit

 
Some subspecies of Plain's Zebra

I have recently created an edit of Zebra Botswana.jpg for its featured picture attempt, and one of the major problems I have with the image is that the subspecies of Plain's Zebra (at least, I think that's the right species) isn't identified. It seems to match Burchell's Zebra, but I would like a second (and perhaps more knowledgeable) opinion. --Pharaoh Hound (talk) 16:19, 14 January 2007 (UTC)

casimir effect and gravity mix up edit

i was asked a while back and could not answer, so could someone tell me how they knew the casimir effect was caused by the zero point field and not by the gravity between the two plates. i know they did somehow but not how they distinguished this. can anyone help me? curious

how can digital radios auotmatically jump over noisy channels? edit

Do they compute the spectrum width on the fly with some kinda DFT? tia --Ulisse0 14 January 2007 (UTC)